PDA

Archiv verlassen und diese Seite im Standarddesign anzeigen : Kollision trotz parallelem Kurs?


Seiten : [1] 2

SCR
26.10.09, 08:07
Hallo zusammen,

können eigentlich zwei Raumschiffe in unserem realen Universum antriebslos einen eigentlich parallelen Kurs fliegen ohne nicht irgendwann einmal zusammenzustoßen (G-Felder außen vor gelassen)?

Ergänzend:
Falls ja (Achtung: doppelte Verneinung ;)): Ruhen sie trotzdem zueinander?
:rolleyes:

JoAx
26.10.09, 13:15
Hallo SCR,


können eigentlich zwei Raumschiffe in unserem realen Universum antriebslos einen eigentlich parallelen Kurs fliegen ohne nicht irgendwann einmal zusammenzustoßen (G-Felder außen vor gelassen)?


wenn wir unser reales Universum nehmen, dann können wir die g-Felder wohl kaum aussen vor lassen.

Nehmen wir an, dass die Raumzeitgeometrie auf ihrem gesamten betrachteten Weg überall flach (ungekrümmt) ist, und ihre Geschwindigkeit relativ zu einander Null ist, dann sehe ich keinen Grund, warum sie sich in einem Raumzeitpunkt treffen sollten. (imho)


Gruss, Johann

SCR
26.10.09, 13:30
Hallo JoAx,

ich hatte vor meinem geistigen Auge auf meinem Wasserplaneten (= 2D-Riemann-Geometrie; aber den kennst Du ja schon ;)) zwei Objekte parallel beschleunigt und sie dann antriebslos (und unterstellt reibungslos) treiben lassen ... Da sind die bei mir doch glatt nach einer Weile 2x zusammengestoßen (In der Realität wäre das Experiment selbstverständlich bereits nach der ersten Kollision beendet gewesen).

JoAx
26.10.09, 13:36
zwei Objekte parallel beschleunigt und sie dann antriebslos (und unterstellt reibungslos) treiben lassen ...

Was war dein Kriterium für die Parallelität? Gibt es so etwas auf deinem "Wasserplaneten" überhaupt?


Gruss, Johann

SCR
26.10.09, 14:01
Ja (bzw. Nein): Ich denke Parallelität gibt es auf meinem Wasserplaneten gar nicht.
Das ist ja auch der eigentliche Hintergrund meiner Frage:

Etwas ruht in unserem Universum zueinander. Das heißt es kann sich auch gleichförmig zueinander bewegen ... Grundsätzlich stelle ich mir da einen "Parallel-Flug" vor - Aber gibt's den überhaupt in einer 3D-Riemann-Geometrie (wenn das anscheinend schon in einer 2D-Riemann-Geometrie nicht klappt)?

Und falls Ja (bzw. Nein): was hieße das denn dann?

JoAx
26.10.09, 14:57
Hallo SCR,

falls ich jetzt was falsches sage, werde ich korregiert. :)

Es kann drei in ihrer Art unterschiedliche Metriken (?) geben.

Positiv -> oberfläche eines Planeten, eines Zylinders. (z.B.)
NULL -> flach = (pseudo-) euklidisch
Negativ -> Oberfläche eines Sattels (z.B.)

Wenn unser Universum global positiv gekrümmt wäre, dann würde es beschleunigt kollabieren, beschleunigt immer kleiner werden.

Wenn unser Universum global nichtgekrümmt wäre, dann würde es konstant moderat (langsam) wachsen. (Konstante Ausdehnung ist nicht satbil -> würde in Kontraktion übergehen, denke ich.)

Wenn unser Universum global negativ gekrümmt wäre, dann würde es beschleunigt wachsen. (War so in der inflationären Phase. Ist wieder, wie es aussieht, seit 7 Mrd. Jahren. (?))

.....


Gruss, Johann

Timm
26.10.09, 18:04
können eigentlich zwei Raumschiffe in unserem realen Universum antriebslos einen eigentlich parallelen Kurs fliegen ohne nicht irgendwann einmal zusammenzustoßen (G-Felder außen vor gelassen)?

Ergänzend:
Falls ja (Achtung: doppelte Verneinung ;)): Ruhen sie trotzdem zueinander?
:rolleyes:

Hallo SCR,

die von Dir beschriebenen Raumschiffe entfernen sich wegen der Expansion des Universums langsam voneinander. Ich habe mal hier irgendwo für abgestufte Massen die Grenzen der gravitativen Bindung kontra Expansion ausgerechnet. Denkbar ist, daß man einen Mindestabstand der Raumschiffe vorraussetzen muß,

Gruß, Timm

EMI
26.10.09, 23:18
Wenn unser Universum global negativ gekrümmt wäre, dann würde es beschleunigt wachsen.
Genau JoAx,

das folgt aus der ART.
Richtig sicher ist man sich mit der Materiedichte im Universum nocht nicht.
Aber diese, nur diese entscheidet welche der 3 Möglichkeiten zutreffend ist!

Beobachtet wird zur Zeit die 3. Möglichkeit(dein Zitat oben), ergo wird die Materiedichte etwas unterhalb des "kritischen" Wertes liegen.

Gruß EMI

PS: das hatte ich aber schon öfters hier im Forum angemerkt.

SCR
27.10.09, 06:41
Die zwei Raumschiffe beschreiben bei paralleler Beschleunigung immer "Großkreise".

Was daraus folgt hängt von der Krümmung des sie umgebenden Raumes statt.

Mein Wasserplanet ist positiv gekrümmt: Die Winkelsumme eines Quadrats ist dort > 360°. In einem positiven gekrümmten Raum können die Raumschiffe nicht parallel fliegen, sie stoßen irgendwann zusammen.

In einem ungekrümmten Raum beträgt die Winkelsumme eines Quadrats genau 360° -> Die Raumschiffe können parallel fliegen.

In einem negativ gekrümmten Raum ist die Winkelsumme eines Quadrats < 360°.
In negativ gekrümmten Räumen entfernen sich im Gegensatz zu positiv gekrümmten Räumen die Raumschiffe voneinander: Ihre "Großkreise" schneiden sich nie.

Falls unser Universum negativ gekrümmt ist ist das festgestellte Raumwachstum deshalb möglicherweise nur ein Trugschluss ... zumindest das beschleunigte (?) :rolleyes:

JoAx
28.10.09, 02:40
Hallo EMI,


... , ergo wird die Materiedichte etwas unterhalb des "kritischen" Wertes liegen.


du meinst es im Sinne - auch ohne dunkle Energie?


Gruss, Johann

JoAx
29.10.09, 00:37
Hallo SCR,


Die zwei Raumschiffe beschreiben bei paralleler Beschleunigung immer "Großkreise".


ich würde nicht immer von Beschleunigungen als Anfang einer Betrachtung ausgehen. Das verkompliziert manchmal die Sache unnötig. Einfach eine Parallelbewegung als Anfangsbedingung reicht vorerst aus.


Mein Wasserplanet ist positiv gekrümmt: Die Winkelsumme eines Quadrats ist dort > 360°. In einem positiven gekrümmten Raum können die Raumschiffe nicht parallel fliegen, sie stoßen irgendwann zusammen.


Ja. Aber der Kurs der Raumschiffe ist nicht einfach nichtparallel. Der Winkel zwischen ihren Geschwindigkeitsvektoren verändert sich ständig so, dass sie beschleunigt aufeinander zu steuern.


In einem ungekrümmten Raum beträgt die Winkelsumme eines Quadrats genau 360° -> Die Raumschiffe können parallel fliegen.


Ja. Das bedeutet aber nicht, dass der Raum nicht wachsen(/schrumpfen) kann. Bei einer konstanten Raumveränerungsrate würde man ihre Bewegung einfach als auf- bzw. voneinander interpretieren. Das kann man aber durch eine einmalige Kurskorrektur kompensieren. (Oder von Anfang an "nichtparallel" beschleunigen. ;))


In einem negativ gekrümmten Raum ist die Winkelsumme eines Quadrats < 360°. In negativ gekrümmten Räumen entfernen sich im Gegensatz zu positiv gekrümmten Räumen die Raumschiffe voneinander: Ihre "Großkreise" schneiden sich nie.


Ja. Mit dem analogen "aber", wie für positive Krümmung.
_______________________________________________

Nimm einfach einen dritten "Schiedsrichter"-Raumschiff hinzu, der genau in der Mitte zwischen den anderen Raumschiffen startet. Jetzt denk dir das "Wasser"-Planet, mit welcher Krümmung auch immer, weg und betrachte nur noch diese drei Schiffe (das ist der Sinn der Relativität, imho), mit dem "Schiedsrichter"-R. als Bezugssystem.
_______________________________________________


Falls unser Universum negativ gekrümmt ist ist das festgestellte Raumwachstum deshalb möglicherweise nur ein Trugschluss ... zumindest das beschleunigte (?) :rolleyes:


Oder ein Laie (wie auch isch) versteht etwas anderes darunter, wenn er/sie davon hört. "Beschleunigte" Veränderung ist für eine Krümmung aber Voraussetztung (imho).


Gruss, Johann

SCR
29.10.09, 09:45
"Beschleunigte" Veränderung ist für eine Krümmung aber Voraussetztung (imho).
Das hat ein anderer Laie (sprich "isch") nicht verstanden.

JoAx
29.10.09, 10:06
Nur das Zitierte, oder alles?


Gruss, Johann

SCR
29.10.09, 11:04
Nur das Zitierte - das andere sehe ich so ;):

ich würde nicht immer von Beschleunigungen als Anfang einer Betrachtung ausgehen. Das verkompliziert manchmal die Sache unnötig.
Da kann ich aber doch gar nix für: Damit sich was bewegt ist es nun einmal zwingend erforderlich ... ;)
Ja. Aber der Kurs der Raumschiffe ist nicht einfach nichtparallel. Der Winkel zwischen ihren Geschwindigkeitsvektoren verändert sich ständig so, dass sie beschleunigt aufeinander zu steuern.
Diese "Beschleunigung" ist - im Gegensatz zu anderen! - entweder nur "Illusion" (da rein krümmungsbedingt - Da vertrete ich ja schon fast eine "richy-Argumentation") oder man verkompliziert eben die ganze Angelegenheit damit es wieder zusammenpasst (Muß sich wieder irgendwo die Energie dafür "herholen" etc. :rolleyes:) ...
Ja. Das bedeutet aber nicht, dass der Raum nicht wachsen(/schrumpfen) kann. Bei einer konstanten Raumveränderungsrate würde man ihre Bewegung einfach als auf- bzw. voneinander interpretieren. Das kann man aber durch eine einmalige Kurskorrektur kompensieren. (Oder von Anfang an "nichtparallel" beschleunigen.;)
Ja: Bin voll dabei :).
Nimm einfach einen dritten "Schiedsrichter"-Raumschiff hinzu, der genau in der Mitte zwischen den anderen Raumschiffen startet. Jetzt denk dir das "Wasser"-Planet, mit welcher Krümmung auch immer, weg und betrachte nur noch diese drei Schiffe (das ist der Sinn der Relativität, imho), mit dem "Schiedsrichter"-R. als Bezugssystem.
Das ist mir schon zu kompliziert: Zur Nachweisführung genügt es mir, wenn zwei Schiffe zusammenstoßen. Ein drittes hierbei mit zu verwenden ist in meinen Augen reine Verschwendung von Steuergeldern. ;)

JoAx
29.10.09, 13:49
Hallo SCR,


Da kann ich aber doch gar nix für: Damit sich was bewegt ist es nun einmal zwingend erforderlich ...


IMHO.
Nach aktuellem Standardmodell ist für eine "Bewegung" nur der Urknall erforderlich. Ob und in wie weit dieser mit (unserer) Beschleunigung verglichen werden kann/darf - ich weiss nicht. Gibt es ein Mal relative Bewegungen, dann braucht es keine weitere Bewegungsveränderungen, um Raumzeitkrümmung festzustellen. Speziell braucht es keine parallele "Bewegung" mit gleicher Geschwindigkeit (=relative Ruhe), als Ausgangsbedingungen. Wie untersucht man die Krümmung unseren Universums? Bestummt nicht mit zwei parallel gestarteten Raumschiffen, oder?


(Muß sich wieder irgendwo die Energie dafür "herholen" etc. :rolleyes:) ...


Dazu fällt mir der Beitrag von Uli und Nachfolgediskussion ein:

http://www.quanten.de/forum/showthread.php5?p=39934#post39934


Das ist mir schon zu kompliziert: Zur Nachweisführung genügt es mir, wenn zwei Schiffe zusammenstoßen. Ein drittes hierbei mit zu verwenden ist in meinen Augen reine Verschwendung von Steuergeldern. ;)

Ok. Dann betrachte die Situation aus der Sicht eines der beiden Raumschiffe, aber nicht von "Oben", nicht vom Standpunkt des "Wasserplaneten". Wie sieht es dann aus?


Gruss, Johann

SCR
29.10.09, 16:25
Hallo JoAx,
Dazu fällt mir der Beitrag von Uli und Nachfolgediskussion ein: http://www.quanten.de/forum/showthread.php5?p=39934#post39934
Nur zur Klarstellung: Ich bezog mich bei meinem Statement rein auf die beschleunigte Raumexpansion (Ansonsten bin ich bezüglich "Energie" grundsätzlich noch am brüten und noch nicht aussagefähig - Kann Ulis Argumentation aber absolut folgen, evtl. geht es nur noch etwas "universeller" ...).
Ok. Dann betrachte die Situation aus der Sicht eines der beiden Raumschiffe, aber nicht von "Oben", nicht vom Standpunkt des "Wasserplaneten". Wie sieht es dann aus?
Ok: Dann eben drei Schiffe ;). Aber es bringt in meinen Augen nichts wenn alle Drei kollidieren :D -> Das Schiedsrichter-Schiff ruht (?).

(Ansonsten: Keine weiteren Anmerkungen ;))

Timm
29.10.09, 16:32
Wie untersucht man die Krümmung unseren Universums? Bestummt nicht mit zwei parallel gestarteten Raumschiffen, oder?


Nein, sondern mit WMAP:

http://map.gsfc.nasa.gov/
WMAP nailed down the curvature of space to within 1% of "flat" Euclidean, improving on the precision of previous award-winning measurements by over an order of magnitude


Gruß, Timm

SCR
29.10.09, 20:36
WMAP nailed down the curvature of space to within 1% of "flat" Euclidean, improving on the precision of previous award-winning measurements by over an order of magnitude.
Können wir damit ("1% Krümmung") nicht prüfen ob das in etwa mit der Raumexpansion hinkäme?
Dazu bräuchten wir vermutlich noch ein exemplarisches Beispiel einer entsprechend rotverschobenen Galaxie ...
Aber so oder so: Das übersteigt bei weitem meine mathematischen Fähigkeiten - Ich kann das leider nicht :(.

Timm
30.10.09, 18:34
Können wir damit ("1% Krümmung") nicht prüfen ob das in etwa mit der Raumexpansion hinkäme?


Ich glaube nicht, SCR,

wenn unser sichtbares Universum nahezu flach ist, wie es die Daten nahelegen, dann heißt das erst mal nur, daß dieses sichtbare Universum ein entsprechend winziger Bruchteil des "ganzen" Universums ist, wie man am Ballon-Modell sieht. Dabei muß man allerdings die gefundene Isotropie auch großräumig, also über den sichtbaren Teil weit hinausgehend unterstellen.

Mir ist von einem Zusammenhang der Expansionsgeschwindigkeit mit der Geometrie des Universums nichts bekannt.

Gruß, Timm

SCR
31.10.09, 00:16
Hallo Timm,
Mir ist von einem Zusammenhang der Expansionsgeschwindigkeit mit der Geometrie des Universums nichts bekannt.
Mir auch nicht - Ich gehe aber inzwischen davon aus: http://www.quanten.de/forum/showthread.php5?p=43104&postcount=165.

Timm
31.10.09, 11:03
Hallo SCR,

ich sollte noch klarstellen, um ein mögliches Mißverständnis zu vermeiden, daß dieses statement sich auf die derzeit beobachtete Expansion des Universums bezieht:

Zitat von Timm:
Mir ist von einem Zusammenhang der Expansionsgeschwindigkeit mit der Geometrie des Universums nichts bekannt.


Und nicht auf die inflationäre Expansion in der Frühphase des Universums. In dieser sehen Kosmologen die Ursache dafür, daß das sichtbare Universum zum Zeitpunkt der ungehinderten Verbreitung der Hintergrund-Photonen ca. 380000 Jahre nach dem Urknall, nahezu flach war. Das wurde ja schon häufig gepostet.

Gruß, Timm

Bauhof
31.10.09, 11:31
Mir ist von einem Zusammenhang der Expansionsgeschwindigkeit mit der Geometrie des Universums nichts bekannt.
Hallo Timm,

beim geschlossenen Universum-Modell [1] (überall positive Krümmung) gibt es einen Zusammenhang zwischen der Expansionsrate und der Geometrie des Universums. Die in diesem Modell mögliche globale Krümmung nimmt zeitlich um so schneller ab, je größer die Expansionsrate des Universums ist.

M.f.G. Eugen Bauhof

[1] Ob die globale Krümmung exakt Null ist (bei globaler euklidischer Geometrie) oder etwas größer Null ist (bei positiver Krümmung), kann aufgrund eines vierdimensionalen Krümmungsradius von rund 13 Milliarden Lichtjahren mit den heutigen Messmethoden noch nicht entschieden werden.

SCR
31.10.09, 12:11
z.B. http://www.aei.mpg.de/einsteinOnline/de/vertiefung/kosmischerSchall/index.html
Es wird oft so argumentiert "Wenn unser Universum so und so gekrümmt ist dann endlose Expansion / Big Crunch / ...".
Hier wird in meinen Augen Ursache und Wirkung verwechselt:
Raumexpansion führt zu einer negativen Raumkrümmung
Raumverkleinerung würde zu einer positiven Raumkrümmung führen
Ein statischer Raum wäre ungekrümmt
...
(Begründung siehe "Modelle von Raum und Zeit")

Timm
31.10.09, 17:52
Hallo Timm,

beim geschlossenen Universum-Modell [1] (überall positive Krümmung) gibt es einen Zusammenhang zwischen der Expansionsrate und der Geometrie des Universums. Die in diesem Modell mögliche globale Krümmung nimmt zeitlich um so schneller ab, je größer die Expansionsrate des Universums ist.

M.f.G. Eugen Bauhof

[1] Ob die globale Krümmung exakt Null ist (bei globaler euklidischer Geometrie) oder etwas größer Null ist (bei positiver Krümmung), kann aufgrund eines vierdimensionalen Krümmungsradius von rund 13 Milliarden Lichtjahren mit den heutigen Messmethoden noch nicht entschieden werden.

Hallo Eugen,

im Rahmen der Meßgenauigkeit (WMAP) war zum Zeitpunkt des Durchsichtig werdens, also 380000 Jahre nach dem Urknall, die Krümmung =0. Die Expansion sollte sich nach den Friedmann-Gleichungen also immer mehr verlangsamen (K=0). Darum scheint das Universum sich aber nicht zu scheren, denn die Expansionsrate nimmt zu, nicht nur das, sie nimmt beschleunigt zu. Und dies, wenn ich es richtig im Gedächtnis habe, seit ca. 5 Milliarden Jahren.

Ob Quintessenz oder kosmologische Konstante, niemand weiß heute, was dahinter steckt. Deshalb bin ich der Meinung, daß es keinen klaren Zusammenhang von Geometrie und Expansionsrate gibt. Ich weiß auch nicht, ob man die Friedmann-Gleichungen so einfach an die neuen Erkenntnisse anpassen darf. Wenn ja, müßte man jetzt von negativer Krümmung ausgehen. Deine Meinung dazu würde mich interessieren.


Ob die globale Krümmung exakt Null ist (bei globaler euklidischer Geometrie) oder etwas größer Null ist (bei positiver Krümmung), kann aufgrund eines vierdimensionalen Krümmungsradius von rund 13 Milliarden Lichtjahren mit den heutigen Messmethoden noch nicht entschieden werden.

Heute nicht, aber gibt es solche Durchmusterungen nicht schon? Man stelle sich vor, künftige Resultate bestätigen den WMAP'schen euklidischen Raum.

Gruß, Timm

Bauhof
31.10.09, 18:41
Heute nicht, aber gibt es solche Durchmusterungen nicht schon? Man stelle sich vor, künftige Resultate bestätigen den WMAP'schen euklidischen Raum.
Hallo Timm,

ich bezweifle, dass man durch WMAP einen exakt euklidischen Raum bestätigen kann. Die mögliche globale Gauss'sche positive Krümmung des Raumes beträgt höchstens 1/(13 Milliarden Lichtjahre)², also fast Null.

Außerdem halte ich aus wahrscheinlichkeitstheoretischen Gründen einen Raum mit der Krümmung exakt Null für extrem unwahrscheinlich.

Alle anderen Topologien sind viel wahrscheinlicher, da es unendlich mehr Möglichkeiten für negative Krümmungen und positive Krümmungen des Raumes gibt.

M.f.G. Eugen Bauhof

Timm
31.10.09, 19:56
Hallo Eugen,

auch bei Wiki findet sich der Befund:

Die Datenlage lässt auf ein Universum mit flacher (Euklidischer) Geometrie schließen.


Dabei darf man natürlich nicht übersehen, daß dieser sich, wie schon angemerkt, nicht auf das "heutige" Universum bezieht.


Außerdem halte ich aus wahrscheinlichkeitstheoretischen Gründen einen Raum mit der Krümmung exakt Null für extrem unwahrscheinlich.

Alle anderen Topologien sind viel wahrscheinlicher, da es unendlich mehr Möglichkeiten für negative Krümmungen und positive Krümmungen des Raumes gibt.


Diese Ansicht gibt es durchaus. Aber gerade deshalb halten es Physiker für keinen Zufall, daß die derzeitigen Daten eine außerordentlich gute Übereinstimmung von kritischer Dichte und Energiedichte nahelegen. Sondern vermuten eher eine tiefere Ursache dafür. Ich kann es Die im Moment nicht belegen, aber wenn Du daran interessiert bist, suche ich gerne danach.

Gruß, Timm

SCR
31.10.09, 20:35
Hallo Timm,
Deshalb bin ich der Meinung, daß es keinen klaren Zusammenhang von Geometrie und Expansionsrate gibt.
Kannst Du bitte einmal definieren was Du unter Krümmung verstehst? Vielen Dank!

EMI
31.10.09, 20:45
Aber gerade deshalb halten es Physiker für keinen Zufall, daß die derzeitigen Daten eine außerordentlich gute Übereinstimmung von kritischer Dichte und Energiedichte nahelegen. Sondern vermuten eher eine tiefere Ursache dafür.
Hallo Timm,

ist das Universum euklidisch flach müsste es sich nach der ART ewig immer langsamer werdend ausdehnen.
Beobachtet wird aber zur Zeit eine offensichtlich zunehmende beschleunigte Expansion.
Hierzu ist laut der ART ein Energiedichtewert der unter der kritischen Dichte liegt notwendig.
Der ermittelte Dichtewert liegt zur Zeit mit dem Faktor +/-10 um den kritischen Wert. Dieser ermittelte Wert bezieht sich NUR auf die sichtbare baryonische Materie.
Um den ermittelten Wert "zu drücken" wurde die dunkle Energie erfunden, die bekanntlich negativ sein muss um ein beschleunigt expandierendes Universum, wie beobachtet, ART konform "zu machen".

Gruß EMI

SCR
01.11.09, 10:49
Ein statischer Raum wäre ungekrümmt
Das muß ich korrigieren - besser gesagt ergänzen (Denn die Aussage an sich stimmt):
Allerdings kann auch ein expandierender Raum umgekrümmt sein - sofern das Wachstum sich auf eine reine Verlängerung seiner Geodäten beschränkt.

Uli
01.11.09, 15:39
Das muß ich korrigieren - besser gesagt ergänzen (Denn die Aussage an sich stimmt):
Allerdings kann auch ein expandierender Raum umgekrümmt sein - sofern das Wachstum sich auf eine reine Verlängerung seiner Geodäten beschränkt.

Ein ungekrümmter Raum wäre ja euklidisch und Gedäten sind dort Geraden.

Das Wachstum soll sich also auf das Wachsen des Abstandes zweier Punkte voneinander beschränken. Gibt es denn weitere Arten von Wachstum als diese ?

Wieso soll ein statischer Raum ungekrümmt sein ?
Betrachte die Oberfläche eines Apfels. Sie ist statisch und dennoch gekrümmt.

Mir kommt vor, dass du eine Menge "Stuss" von dir gibst.

Timm
01.11.09, 17:53
Hallo EMI,

ist das Universum euklidisch flach müsste es sich nach der ART ewig immer langsamer werdend ausdehnen.



im Rahmen der Meßgenauigkeit (WMAP) war zum Zeitpunkt des Durchsichtig werdens, also 380000 Jahre nach dem Urknall, die Krümmung =0. Die Expansion sollte sich nach den Friedmann-Gleichungen also immer mehr verlangsamen (K=0).

Mir scheint, wir sind uns in dieser Frage einig.


Um den ermittelten Wert "zu drücken" wurde die dunkle Energie erfunden, die bekanntlich negativ sein muss um ein beschleunigt expandierendes Universum, wie beobachtet, ART konform "zu machen".


Ich weiß nicht, ob hier ein Mißverständnis vorliegt. Die dunkle Energie macht man für die beschleunigte Expansion verantwortlich. Und diese schließt man aus der Auswertung von Supernovae Helligkeiten. Hat also mit dem Mikrowellen Hintergrund und den WMAP Daten nichts zu tun.

Die Frage nach der dunklen Energie ist offen. Da sie offensichtlich abstoßend wirkt, könnte es sich um die viel beschworene kosmologische Konstante handeln, die positv sein müßte und die nicht zeitabhängig ist. Oder um eine zeitabhängige Vakuum Energie unbekannter Art, die auch Quintessenz genannt wird. Vielleicht können künftige noch genauere Durchmusterungen diese Frage klären helfen. Mindestens so lange ist das Schicksal des Universums offen.

Gruß, Timm

Timm
01.11.09, 18:36
Hallo Timm,

Kannst Du bitte einmal definieren was Du unter Krümmung verstehst? Vielen Dank!

Hallo SCR,

ich bin leider außerstande in die Mathmatik einzusteigen. Implizit steckt die Krümmung der Raumzeit in den einsteinschen Feldgleichungen.

Einige allgemeine Zusammenhänge findest Du hier:

http://de.wikipedia.org/wiki/Allgemeine_Relativit%C3%A4tstheorie

Bei bekannter Krümmung lassen sich Aussagen über das Schicksal des Universums machen, das wurde hier ja schon mehrmals erläutert,

Gruß, Timm

SCR
01.11.09, 18:49
Hallo Timm,
kein Ding - Danke für Dein Feedback!

Hallo Uli,
Ein ungekrümmter Raum wäre ja euklidisch und Gedäten sind dort Geraden.
Ja.
Das Wachstum soll sich also auf das Wachsen des Abstandes zweier Punkte voneinander beschränken. Gibt es denn weitere Arten von Wachstum als diese?
Eine Fläche. Objekt1(x,y) sei (1,0), Objekt2(x,y) sei (2,0).
Beide Objekte bewegen sich nun in Richtung y.
Wächst die Fläche ausschließlich in y-Richtung (= Verlängerung der Geodäten) wirkt sich das nicht auf die Flugrichtung und ihren Abstand zueinander aus -> Krümmung?
Wächst die Fläche dabei homogen in y-Richtung - Werden die Objekte mit physikalisch spürbaren Auswirkungen gedehnt -> Krümmung?
Wächst die Fläche in x-Richtung homogen, werden sich die Objekte mit zunehmendem y immer mehr voneinander entfernen -> Krümmung?
Wächst die Fläche inhomogen, d.h. z.B. nur an den Rändern ... -> Krümmung?
Wieso soll ein statischer Raum ungekrümmt sein ?
Betrachte die Oberfläche eines Apfels. Sie ist statisch und dennoch gekrümmt.
Da hast Du völlig Recht: Statisch = "Krümmung wird beibehalten" und nicht "= ungekrümmt".
Mir kommt vor, dass du eine Menge "Stuss" von dir gibst.
Dem kann durchaus so sein - Die Gefahr geht man ein, wenn man zu irgendetwas Stellung bezieht. Dem kann man nur 100%ig entgehen indem man die Klappe hält - Nur lernt man mit Klappehalten nicht dazu.
Von daher meine Aufforderung an jeden hier: Immer feste d'rauf falls ich Stuss schreibe - Schadet nie! ;)

Ich hätte noch zwei Fragen an Dich:

Wie definierst Du Krümmung?
Hängt die Aussage zuvor (Statisch = "Krümmung wird beibehalten") in Deinen Augen nicht vom Material / von der Festigkeit (sprich seiner Unveränderlichkeit) ab?
Sobald es eine Wachstums-Dynamik bezüglich der Fläche/des Raums gibt, scheint es mir so, als wären keine absoluten Krümmungsaussagen mehr möglich - nur noch relative. Was meinst Du?

EMI
01.11.09, 19:53
Die dunkle Energie macht man für die beschleunigte Expansion verantwortlich.
Hallo Timm,

richtig, sagte ich ja.
Nur erst hat man die beschleunigte Expansion entdeckt, war "erschrocken" weil das nicht sein durfte, und zur "Rettung" erfand man dann die dunkle Energie.
Diese muss aber negativ sein damit alles passt.
Nachgewiesen ist die nicht, wird sie wohl auch nicht. IMHO
Das Ganze hat nicht's mit Hintergrundstrahlung, Supernovae, Inflationsphase zu tun.
Die dunkle (negative) Energie braucht's um die beobachtete beschleunigte Expansion, trotz baryonischer Energiedichte in Nähe der kritischen Dichte, zu erklären.

Gruß EMI

Timm
01.11.09, 20:30
Das Ganze hat nicht's mit Hintergrundstrahlung, Supernovae, Inflationsphase zu tun.
Die dunkle (negative) Energie braucht's um die beobachtete beschleunigte Expansion, trotz baryonischer Energiedichte in Nähe der kritischen Dichte, zu erklären.


Hallo EMI,

jetzt hast Du mich in einen ratlosen Zustand versetzt. Wie hat man denn die beschleunigte Expansion entdeckt? Wenn ich Dich richtig verstehe, nicht durch die Auswertung der Helligkeiten von Supernovae.

Gruß, Timm

SCR
02.11.09, 00:20
Hallo Timm,
zu EMIs Ausführungen hier ein evtl. weiterführender Link: http://www.uni-heidelberg.de/presse/ruca/ruca05-2/blick.html

JoAx
02.11.09, 03:42
Hallo zusammen.

http://www.physikclub.de/aus-forschung-und-wissenschaft/berichte-aus-naturwissenschaft-und-philosophie/der-klingende-kosmos/81.jpg

Ich denke, dass ungefähr so die Analyse der WMAP Bilder aussieht.


Gruss, Johann
http://www.physikclub.de/aus-forschung-und-wissenschaft/berichte-aus-naturwissenschaft-und-philosophie/der-klingende-kosmos/81.jpg/image_view_fullscreen

EMI
02.11.09, 04:53
jetzt hast Du mich in einen ratlosen Zustand versetzt. Wie hat man denn die beschleunigte Expansion entdeckt?
Wenn ich Dich richtig verstehe, nicht durch die Auswertung der Helligkeiten von Supernovae.
Hallo Timm,

also ich würde Diese durch die tiefgründige Auswertung der Rotverschiebung/Galaxienflucht "entdecken".
Ich denke deshalb, dass Diese genau so auch von den Astronomen entdeckt wurde.
Dabei war ich aber nicht, leider.

Gruß EMI

PS: es kann natürlich sein, dass die Astronomen die Rotverschiebung der Galaxien mit dem Licht von Supernovae in den Galaxien auswerten.
Ist ja naheliegend, da dieses Licht besonders intensiv ist.
Nicht mit einer Supernova an sich kann man auf die derzeitige Expansion schließen aber mit deren Rotverschiebung, je nach dem wie weit sie von und weg ist.
Das meintest Du bestimmt denke ich.

SCR
02.11.09, 08:00
Hallo JoAx,
Ich denke, dass ungefähr so die Analyse der WMAP Bilder aussieht.
Ja, das ist ja auch "sinnvoll".

Aber dann müsste man IMHO bei den WMAP-Analysen auf ein xxxxxx negativ gekrümmtes Universum kommen: Das homogen unterstellte Raumwachstum "biegt" die Geodäten, denen die Lichtstrahlen folgen, im Zeitverlauf "auf": Das aussendende Objekt müsste uns deshalb größer erscheinen als es tatsächlich ist.
In sehe keine Möglichkeiten, die Auswirkungen des Raumwachstums von einer eventuell "statischen" Krümmung des Universums zu unterschieden - Ihr? :rolleyes:
(Anmerkung: Ich persönlich denke ja, dass bezogen auf den Raum Wachstum/Schrumpfung und Krümmung exakt dasselbe sind).

Wenn/Da die WMAP-Analysen jedoch zu dem Schluß kommen, dass Universum sei ungekrümmt, stimmt hier etwas nicht:
a) Wir haben gar kein Raumwachstum
b) Das Raumwachstum liegt unterhalb der WMAP-Messgrenze
c) Die Effekte des Raumwachstums werden durch Effekte einer (partiellen?) Raumschrumpfung auf dem Weg der Photonen wieder aufgehoben.

Oder d): Ich sehe wieder einmal etwas falsch. :D

EDIT: Sehe gerade da stimmt so oder so etwas nicht - Ich denke, die Grafiken sind falsch: Bei einer positiven Krümmung laufen "Parallelen" zusammen, bei einer negativem Krümmung auseinander (Jetzt völlig wurscht ob ein Zusammenhang mit dem Raumwachstum besteht oder nicht) -> Die Grafiken in der ersten Zeile "links" und "rechts" (Negativ <-> Positiv) gehören IMHO getauscht. Oder bin ich doof? :rolleyes:

Timm
02.11.09, 09:58
Nicht mit einer Supernova an sich kann man auf die derzeitige Expansion schließen aber mit deren Rotverschiebung, je nach dem wie weit sie von und weg ist.
Das meintest Du bestimmt denke ich.

Ja, EMI, und das funktioniert deshalb, weil die Supernovae vom Typ Ia wegen des gleichen Explosionsmechanismus praktisch alle gleich hell sind. Deshalb dienen sie den Astronomen als Standard Kerzen. Der Vergleich mit den Rotverschiebungsdaten ergab dann eben die Riesenüberraschung, daß sich das Universum seit 5 Milliarden Jahren beschleunigt ausdehnt.

Die Vorhersage der WMAP Daten ist ja eine völlig andere: Danach sollte sich die Expansion immer mehr verlangsamen! Das heißt nicht, daß die WMAP Daten falsch sind. Sie charakterisieren die Geometrie des Universums zu diesem frühen Zeitpunkt sicherlich richtig.

Trägt man den Skalenfaktor gegen die Zeit auf, so verläuft er bei t=0 extrem steil (Inflation), flacht dann immer mehr ab und wird durch den Effekt der dunklen Energie wieder steiler, er durchläuft also einen Wendepunkt.

Die wirklich spannende Frage ist, ob das Universum noch ein Friedmann-Lemaitre Universum ist.
@Eugen+all, Du hast Dich nicht mehr geäußert. Deine Meinung zu dieser Frage würde mich schon auch interessieren. Du kannst mir selbstverständlich auch widersprechen, ich habe damit kein Problem.

Nach meiner Einschätzung ist Fiedmann-Lemaitre und damit deren Vorhersagen bzgl. Geometrie und Schicksal des Universums nur aufrecht zu erhalten, wenn sich die dunkle Energie als positive kosmologische Konstante erweisen wird. Andernfalls, das wäre der Fall Quintessenz, scheinen keine Lösungen der Einsteinschen Gleichungen in Sicht, die den heutigen Stand der Erkenntnisse widerspiegeln.

Gruß, Timm

SCR
02.11.09, 10:08
Hallo Timm,
Sie charakterisieren die Geometrie des Universums zu diesem frühen Zeitpunkt sicherlich richtig.
Ich denke Du mißinterpretierst das völlig: Das ist nicht die Krümmung des damaligen Universums.
Die Hintergrundstrahlung wurde "damals" abgegeben und durchläuft "seitdem" unser Universum bis sie jetzt hier eintrifft -> Du kannst damit jetzt nur alle Raumkrümmungen "in Summe" messen, die das Photon seit dem Aussenden über den ganzen Weg hinweg ausgesetzt war.
Wie willst Du das zeitlich differenzieren? Das geht schlichtweg nicht.
Du kannst nur Messwerte von vor x Jahren mit den heutigen vergleichen und daraus das Delta bilden. Dann kannst Du daraus etwas über den "aktuellen Trend" ableiten - Anders/Mehr ist nicht drin.

(IMHO - Und gerne feste drauf ;)).

Timm
02.11.09, 10:17
Wenn/Da die WMAP-Analysen jedoch zu dem Schluß kommen, dass Universum sei ungekrümmt, stimmt hier etwas nicht:
a) Wir haben gar kein Raumwachstum
b) Das Raumwachstum liegt unterhalb der WMAP-Messgrenze
c) Die Effekte des Raumwachstums werden durch Effekte einer (partiellen?) Raumschrumpfung auf dem Weg der Photonen wieder aufgehoben.

Oder d): Ich sehe wieder einmal etwas falsch. :D


Du hast mit d) meine volle Zustimmung. Wie in diesem Thread schon mehrfach dargestellt, expandiert auch ein euklidisches Universum. Warum nimmst Du das nicht zur Kenntnis? Außerdem, schau halt auch mal gelegentlich bei Wiki nach. Und bilde Dir, wenn Du das alles gemacht hast, eine Meinung. Nur eine kleine Empfehlung von meiner Seite.

Gruß, Timm

Timm
02.11.09, 10:32
Lies das, es ist ein fach geschriebener Artikel.

http://www.wissenschaft.de/wissenschaft/news/263160.html

Gruß, Timm

SCR
02.11.09, 10:44
Alles Käse was ich geschrieben habe - und auch wieder nicht ;): Ich glaube, ich habe jetzt geschnallt, was und wie die da messen.

1. Man hat bekannte Daten z.B. einer Galaxie bezüglich der Strahlung(en), die diese aussendet (sichtbares Licht, Röngten, ...).
Daraus kann man eine "Größe"/"Ausdehnung" dieser Galaxie bestimmen. Jetzt weiß man aber noch nicht, ob/wie dieser ermittelte Wert durch eine eventuelle Raumkrümmung beeinflusst ist - Blöd.
2. Also geht man "zwecks Vergleich" invers vor: Man betrachtet eine Strahlung, die explizit nicht von dieser Galaxie herrührt sondern bei welcher die Galaxie sozusagen als "Abdeckung" dieser Strahlung fungiert - wie z.B. bei der Hintergrundstrahlung.
Man ermittelt hier die Größe der Abdeckung.

Ergibt sich zwischen den Ergebnissen von 1 und 2 eine Differenz, liegt über den Daumen "die halbe Differenz" als (abstandsbezogene) Raumkrümmung zwischen dem Objekt und dem Beobachter vor.
Von daher stimmt die WMAP-Grafik doch. Aber meine Ausführungen sind auch nicht unbedingt falsch - wenn man berücksichtigt, dass sich meine Äußerungen auf die von der Galaxie ausgesandte Strahlung beziehen.

EDIT: Hallo Timm, sehe gerade Deinen Beitrag: Schaue ich mir an - Danke!

SCR
02.11.09, 10:50
Hallo Timm,
http://www.wissenschaft.de/wissenschaft/news/263160.html
Der Artikel hat nichts mit Raumkrümmung zu tun sondern wie homogen (bzw. inhomogen) und isotrop (bzw. nicht-isotrop) die Strahlung verteilt ist (hier bezüglich Polarisation und Temperatur) -> Die Ableitungen daraus sind selbstverständlich andere -> :confused: (Selbstverständlich kann man aus denen Aussagen über die Entwicklung - und damit "frühere Zeiten" - unseres Universums ableiten)

EDIT:
Außerdem, schau halt auch mal gelegentlich bei Wiki nach. Und bilde Dir, wenn Du das alles gemacht hast, eine Meinung. Nur eine kleine Empfehlung von meiner Seite.
Danke für den Hinweis: Habe ich gemacht - Da steht z.B. zur Krümmung in der Riemannschen Geometrie:
Bemerkenswert ist, dass man zum Beispiel auf der Oberfläche eines Torus eine Metrik definieren kann, die keine Krümmung aufweist. Dies lässt sich aus der Tatsache ableiten, dass man einen Torus aus einer ebenen Fläche bilden kann. Das Koordinatensystem, welches auf der Oberfläche benutzt wird, ergibt sich durch die Abbildung der ebenen Fläche, aus der der Torus gebildet wurde.
D.h. für mich im Klartext ich kann mir jede Krümmung in der Riemann-Geometrie "auf 0 biegen" - Oder verstehe ich das jetzt schon wieder falsch? :rolleyes:
Ich denke dass hiermit gemeint ist, dass man in der Riemann-Geometrie zur Vereinfachung lokal Ungekrümmtheit annehmen darf.
Ich kann es natürlich unterlassen solch womöglich voreiligen Schlüsse zu ziehen und stattdessen einfach fragen "Was bedeutet denn das?" -
Das kommt aus meiner Sicht am Ende aber doch auf's Gleiche raus ...

Und einmal so nebenbei:
Meines Erachtens hat ein Torus "innen" auch eine positive Krümmung: Man muß den Torus (bzw. dessen innere Wölbung) nur als "Bobbahn" ansehen und diese längsfahren. Da habe ich z.B. schon nach Quellen gesucht aber leider nix gefunden. Ergo: Ich habe anscheinend einfach zu dumme Fragen ...

Bauhof
02.11.09, 12:02
(1) D.h. für mich im Klartext ich kann mir jede Krümmung in der Riemann-Geometrie "auf 0 biegen".
(2) Oder verstehe ich das jetzt schon wieder falsch? :rolleyes:

Hallo SCR,

zu (2): Ja, leider schon wieder.

zu (1): Nein, du kannst nicht jede Riemann-Geometrie "auf 0 biegen". Zum Beispiel die Raumzeit in der ART: Sie ist nicht homogen, die kannst du nicht "auf 0 biegen".

Meines Erachtens hat ein Torus "innen" auch eine positive Krümmung: Man muß den Torus (bzw. dessen innere Wölbung) nur als "Bobbahn" ansehen und diese längsfahren. Da habe ich z.B. schon nach Quellen gesucht aber leider nix gefunden. Ergo: Ich habe anscheinend einfach zu dumme Fragen ...

Du solltest mal nach den Stichworten "Innnere Geometrie" und "Gaußsche Krümmung" suchen. Vielleicht erkennst du dann, dass ein Torus keine positive Krümmung aufweist.

M.f.G. Eugen Bauhof

zeitgenosse
02.11.09, 16:19
Vielleicht erkennst du dann, dass ein Torus keine positive Krümmung aufweist.

Ergänzend könnte ich dir einen Abstecher in die Topologie empfehlen, z.B.:

O'Shea, Poincarés Vermutung (Fischer)

Gr. zg

Timm
02.11.09, 16:35
Zum Beispiel die Raumzeit in der ART: Sie ist nicht homogen, die kannst du nicht "auf 0 biegen".


Das könnte jetzt aber doch mißverständlich sein, Eugen. Du hast es wahrscheinlich anders gemeint. Die Raumzeit der ART ist homogen und isotrop, das ist vielfach nachzulesen, z.B. hier:

http://idefix.physik.uni-freiburg.de/user/aufgabe/Skripte/RaumZeit.pdf

Seite 182:
Die wesentliche Annahme, die für kosmologische Lösungen der Einstein-Gleichungen meist gemacht wird, ist die Homogenität und Isotropie des Universums.


Gruß, Timm

Bauhof
02.11.09, 17:14
Das könnte jetzt aber doch mißverständlich sein, Eugen. Du hast es wahrscheinlich anders gemeint. Die Raumzeit der ART ist homogen und isotrop, das ist vielfach nachzulesen, z.B. hier:
http://idefix.physik.uni-freiburg.de/user/aufgabe/Skripte/RaumZeit.pdfGruß, Timm

Hallo Timm,

ja, das könnte missverständlich sein, was ich geschrieben habe. An der Stelle meinte ich die Raumzeit in der Umgebung einer Masse. Und die ist nicht isotrop. Meines Wissens wird die Raumzeit in der Umgebung einer Masse mit der Riemannschen Geometrie [1] beschrieben. Einstein und Grossmann haben es Bernhard Riemann zu verdanken, dass sie darauf zurückgreifen konnten.

Global im Universum kommt noch die Weylsche Krümmung :eek: dazu. Aber das würde jetzt wohl zu weit führen.

M.f.G. Eugen Bauhof

[1] Die wurde auch von SCR erwähnt.

Bauhof
02.11.09, 18:14
Ergänzend könnte ich dir einen Abstecher in die Topologie empfehlen, z.B.: O'Shea, Poincarés Vermutung (Fischer)
Hallo Zeitgenosse,diese Buch besitze ich bereits. Poincarés Vermutung lautete wie folgt:

Poincaré hatte gefragt, ob jede einfach zusammenhängende dreidimensionale Mannigfaltigkeit, die keinen Rand hat und die nicht unendlich ist, eine dreidimensionale Sphäre ist.

Ein Russe hat das kürzlich bewiesen und den Preis dafür nicht angenommen.

Die Vermutung (und jetzige Tatsache) ist auch in kosmologischer Hinsicht interessant, weil ein Universum-Modell das Universum als eine dreidimensionale Sphäre beschreibt. Eine dreidimensionale Sphäre ist der dreidimensionale Begrenzungsraum ('Oberfläche') einer vierdimensionalen Kugel.

Mit freundlichen Grüßen
Eugen Bauhof

SCR
02.11.09, 18:24
Hallo zusammen,

In meinen Augen Nebenkriegsschauplatz - Aber was soll's.
Ich habe es jetzt doch einmal auf Basis von JoAx Bild das, was ich meine, grob eingezeichnet:
http://img509.imageshack.us/img509/1394/kruemm.jpg
Und wenn der relevante mittlere Bereich eines Torus noch etwas "breiter" ist dann können sogar mehrere Geodäten dort nebeneinander parallel verlaufen.
Hierbei handelt es sich Eurer Einschätzung nach einvernehmlich nicht um einen (mehr oder weniger breiten) Bereich positiver Krümmung im Torus?

Darum geht es mir doch eigentlich:

Ausgehend von diesem Beitrag:
http://www.quanten.de/forum/showpost.php5?p=43297&postcount=44
- Wir verzeichnen eine (beschleunigte) Raumexpansion - Widerspruch?
- Diese wird doch als homogen und isotrop unterstellt, oder?
- Dann müsste aud Basis dieser beiden Prämissen bei den WMAP-Analysen eigentlich eine (leicht) negative Krümmung festgestellt werden.
Meine schlichte Frage lautet: Warum nicht? :rolleyes:

Und damit wären wir wieder bei den bereits aufgeführten Spiegelstrichen:
Wenn/Da die WMAP-Analysen jedoch zu dem Schluß kommen, dass Universum sei ungekrümmt, stimmt hier etwas nicht:
a) Wir haben gar kein Raumwachstum
b) Das Raumwachstum liegt unterhalb der WMAP-Messgrenze
c) Die Effekte des Raumwachstums werden durch Effekte einer (partiellen?) Raumschrumpfung auf dem Weg der Photonen wieder aufgehoben.
Oder alternativ gerne eine andere Erklärung (Wir haben ein inhomogenes Wachstum, das Wachstum wird so und so herausgerechnet, ...) - Danke!

SCR
02.11.09, 22:15
http://www.rzuser.uni-heidelberg.de/~q61/sdw_torus_fac.pdf

Timm
02.11.09, 22:37
ja, das könnte missverständlich sein, was ich geschrieben habe. An der Stelle meinte ich die Raumzeit in der Umgebung einer Masse. Und die ist nicht isotrop.

Damit ist alles geklärt.

Gute Nacht, Eugen,

Gruß, Timm

zeitgenosse
03.11.09, 06:48
diese Buch besitze ich bereits.

Jede einfach zusammenhängende kompakte unberandete 3-dimensionale Mannigfaltigkeit ist homöomorph zur 3-Sphäre.Ein Satz von gewaltiger Tiefe!

Ich empfand das Buch von O'Shea als sehr nützlich.

Um gewisse Vergleiche zu tätigen, habe ich mir ein zweites über dasselbe Thema zugetan:

Szpiro, Das Poincaré Abenteuer (Piper)

Auch empfehlen - wenn gleich in einem anderen Zusammenhang - kann ich folgende Titel:

Singh, Fermats letzter Satz (dtv)

Havil, Gamma (Springer)

Das letztere Buch ist etwas komplizierter geschrieben und hat u.a. die 'Riemannsche Vermutung' zum Inhalt, die meines Wissens noch immer des Vollbeweises harrt. Im Kern handelt es sich um die Riemannsche Zeta-Funktion bzw. um deren Nullstellen und die Verteilung der Primzahlen. Die Hypothese besagt, dass alle nichttrivialen Nullstellen dieser komplexwertigen Funktion den Realteil ½ besitzen.

(Es gibt zwar auch Stimmen, die behaupten, Louis de Branges de Bourcia habe 2004 die Riemannsche Vermutung in Strenge bewiesen. Vermutlich trifft dies nicht zu.)

p.s.
Das ausgesetzte Preisgeld ist übrigens von nicht unbedeutender Summe. ;)
(Grigori 'Grischa' Perelmann würde jetzt unwillig mit dem Kopf schütteln.)

Gr. zg

SCR
03.11.09, 07:08
http://www.rzuser.uni-heidelberg.de/~q61/sdw_torus_fac.pdf
Ich lese diesen Artikel so:
Zuerst gab es die Standardmodell-Vorstellung "ungekrümmtes Universum" welches dann mit Cobe/WMAP/Planck untersucht wurde/wird (Ich dachte, erst auf Grund der dabei gemessenen Ergebnisse wäre man zu diesem Schluß gekommen).
Hier scheint es aber gewisse (geringe) Abweichungen zu geben, die ein Torus-Modell besser erklären würde.
Ein Torus ist aber nicht ungekrümmt - Ich sehe das aktuell eher als eine Bestätigung meiner hier geäußerten Vermutungen.
Man muß aber noch abwarten inwieweit zukünftige Ergebnisse genauere Aussagen erlauben ...
Oder lest Ihr den Artikel anders? :rolleyes:

SCR
03.11.09, 11:52
Oder d): Ich sehe wieder einmal etwas falsch.
Du hast mit d) meine volle Zustimmung.
Ja, leider schon wieder.
Das ist grundsätzlich in Ordnung - Aber zu:
Vielleicht erkennst du dann, dass ein Torus keine positive Krümmung aufweist.
Hierbei handelt es sich Eurer Einschätzung nach einvernehmlich nicht um einen (mehr oder weniger breiten) Bereich positiver Krümmung im Torus?
Meine schlichte Frage lautet: Warum nicht?
Oder lest Ihr den Artikel anders?
... "still ruht der See" (?). http://forum.electronicattack.de/images/smilies/gruebel.gif :(

Timm
03.11.09, 16:28
Hallo SCR,


Zuerst gab es die Standardmodell-Vorstellung "ungekrümmtes Universum"...:

Ja, das "Flachheitsproblem" wurde in den 70er Jahren heftig diskutiert. Man hatte ausgerechnet, daß die Massendichte des Universums 1 s nach dem Urknall mit der Genauigkeit von 1:10^15 mit der kritischen Massendichte übereingestimmt haben mußte. Andernfalls wäre es längst rekollabiert oder so schnell expandiert, daß keine Zeit für die Bildung von Galaxien gewesen wäre. Der Physiker Robert Dicke verglich diese extreme Anforderung an die Flachheit mit dem vielzitierten Beispiel des auf der Spitze stehenden Bleistifts, der längere Zeit nicht umfallen darf. Kein Physiker glaubte ernsthaft an einen so großen Zufall. Die Lösung kam dann von Alan Guth mit seiner Theorie des inflationären Universums. Die dann mit Cobe und WMAP grandios bestätigt wurde und bis heute Standard ist.

Hier scheint es aber gewisse (geringe) Abweichungen zu geben, die ein Torus-Modell besser erklären würde.
Ein Torus ist aber nicht ungekrümmt - Ich sehe das aktuell eher als eine Bestätigung meiner hier geäußerten Vermutungen.


Weiter oben im Thread hast Du einen hyperbolischen Körper fälschlich als Torus bezeichnet. Wahrscheinlich hast Du beim Lesen des Artikels aber bemerkt, daß ein Torus die Form eines Schwimmreifens besitzt, flache Geometrie hat und endlich ist.

Man muß aber noch abwarten inwieweit zukünftige Ergebnisse genauere Aussagen erlauben ...


So stehts geschrieben.

Georg Wolschin veröffentlich immer wieder lesenswerte Artikel im "Spektrum der Wissenschaften",

Gruß, Timm

SCR
03.11.09, 19:01
Hallo Timm,
Weiter oben im Thread hast Du einen hyperbolischen Körper fälschlich als Torus bezeichnet.
Konkret: WO und WER soll hier irgendwo etwas falsch dargestellt haben?
Ich bitte um Nutzung der Zitatfunktion - Danke!
(Evtl. schaust Du auch einmal kurz in den Thread "Modelle von Raum und Zeit" rein - so etwa ab hier (http://www.quanten.de/forum/showthread.php5?p=43092&postcount=159))
Wahrscheinlich hast Du beim Lesen des Artikels aber bemerkt, daß ein Torus die Form eines Schwimmreifens besitzt, flache Geometrie hat und endlich ist.
Aha, da habe ich also etwas bemerkt - Es geschehen also eventuell doch noch Zeichen und Wunder? :rolleyes:
Und hier habe ich dann womöglich auch etwas bemerkt:
Seite 8 des PDFs / Seite 93 des Dokuments - oben (http://www.google.de/url?sa=t&source=web&ct=res&cd=11&ved=0CAYQFjAAOAo&url=http%3A%2F%2Fwww.wissenschaft-online.de%2Fartikel%2F835346&rct=j&q=Torus+negative+Kr%C3%BCmmung&ei=lk_wSrCECdD__AbylrifBw&usg=AFQjCNEs_yefJsGgXND6r44BlOk4gUNkwQ)

Eine konkrete Bitte hätte ich an dieser Stelle an Dich:
Kannst Du mir einmal Deine Bastelanleitung schicken wie ich aus einem PHYSIKALISCHEN Stück Papier einen Torus herstelle?
Nachdem das ja MATHEMATISCH kein Problem darstellt (steht ja so auch nahezu in jeder Publikation) sollte es PHYSIKALISCH doch auch kein Problem sein, oder? Zumal wir hier in einem Physikforum sind.
Das dahinterstehende mathematische Denkmodell (Das habe ich - auch wenn Du es vielleicht nicht glauben magst - wider Erwarten verstanden) darfst Du als bekannt voraussetzen.

Also ich habe hier vor mir bereits in gespannter Erwartung ein Blatt Papier, Schere, Tesa und Klebstoff bereitgelegt - Vielen Dank für Deine Bemühungen!

P.S.: Auch wenn sie vermutlich in Deinen Augen völlig unmaßgeblich ist teile ich Dir noch einmal MEINE Einschätzung zu dem Thema mit:
In meinen Augen differenziert die Mathematik nicht zwischen Stauchen/Verzerren und krümmen - Muß sie auch nicht: Sie hat ja die Unendlichkeit zur Verfügung. In einer Physik, die raumgeometrisch auf Quanten aufbaut (Planck-Länge: Hier ist sicher eine Ansatzmöglichkeit zum Widerspruch), ist eine entsprechende Differenzierung aber meines Erachtens nach erforderlich:
Nur kurz:
- Jede Dimension ist aus (entsprechenden) Quanten aufgebaut
- Die Quanten weisen grundsätzlichen einen identischen "Planck-Abstand" (abhängig von der jeweiligen Dimension) auf
- Dimensionen haben "Richtungen"

Krümmung einer Dimension:
- positiv: in Dimensionsrichtung liegen absolut weniger Quanten und Planck-Abstände vor -> Bewegungen zweier diese Dimension benutzende Objekte (in diese Richtung) führen zwangsläufig zu einer Annäherung, die die Objekte selbst ebenfalls wahrnehmen können
- negativ: in Dimensionsrichtung liegen absolut mehr Quanten und Planck-Abstände vor -> Bewegungen zweier diese Dimension benutzende Objekte (in diese Richtung) führen zwangsläufig zu einer gegenseitigen Entfernung, die die Objekte selbst ebenfalls wahrnehmen können
- ungekrümmt: in alle Dimensionsrichtungen ist die Anzahl an Quanten und Planck-Abständen identisch
[...]
Im Gegensatz dazu Stauchung/Verzerrung einer Dimension:
Die Anzahl der Quanten und Planck-Abstände einer Dimension bleiben identisch, die Länge der Planck-Abstände wird variiert.
Diese Stauchungen/Verzerrungen bleiben einem Objekt, welches diese Dimension benutzt, verborgen.
Das ist absolut noch keine saubere Definition - aber es sollte denoch daraus ersichtlich sein, wo in etwa ich den Differenzierungsbedarf sehe.

zeitgenosse
04.11.09, 06:53
Kannst Du mir einmal Deine Bastelanleitung schicken wie ich aus einem PHYSIKALISCHEN Stück Papier einen Torus herstelle?

Zunächst rolle ich ein Blatt (Rechteck) zu einem Zylinder auf; danach biege ich den Zylinder, derart, dass sich die Stirnflächen berühren. So erhalte ich einen (Ring)-Torus. Allerdings wird der Zylinder während des Biegens aussen gedehnt und innen gestaucht. Wäre das Papierblatt vollkommen elastisch, würde sich am Volumen des Körpers nichts ändern.

Ein Beispiel dieser Art findet sich u.a. bei: Ossermann, Geometrie des Universums (Vieweg)

Es gibt auch flache Tori. Befinden sie sich innerhalb einer Hypersphäre, werden sie als Clifford-Torus bezeichnet.

In der Technik finden wir den Torus bspw. in Form der Torus- bzw. Toroidspule (bei der Farbbildröhre als Sattelspule ausgebildet) und des Torusgetriebes.

In Bezug auf das Weltall wird die Torusgeometrie auch diskutiert:

http://www.rzuser.uni-heidelberg.de/~q61/sdw_torus_fac.pdf (http://www.rzuser.uni-heidelberg.de/%7Eq61/sdw_torus_fac.pdf)

Es sind aber auch andere Topologien im Gespräch, z.B. das Dodekaeder-Universum und das Horn-Universum. Während das erstere Modell auf dem Poincaré-Dodekaeder basiert, besitzt das zweite eine Picard-Topologie mit negativer Krümmung.

Gr. zg

SCR
04.11.09, 09:05
Hallo zg,
Allerdings wird der Zylinder während des Biegens aussen gedehnt und innen gestaucht. Wäre das Papierblatt vollkommen elastisch, würde sich am Volumen des Körpers nichts ändern.
Ach nee (bitte nicht persönlich nehmen!), denn da sind wir genau an DEM Punkt: Wo bekomme ich so ein vollkommen elastisches Papierblatt her?
NIRGENDS! Es gibt eben nur "PHYSIKALISCHE" und keine "MATHEMATISCHEN" Papierblätter zu kaufen.

Denn so in etwa sähe die korrekte dazugehörige PHYSIKALISCHE Torus-Bastelanleitung aus:
http://img682.imageshack.us/img682/6246/basteltorus.jpg

Links und rechts rote Bereiche wegschneiden, stehenbleibende Kanten zusammenkleben.
Immer zwei roten Reststücke rautenartig zusammenkleben und in die Schnitte in der Mitte des Blatts einfügen.

PHYSIKALISCHE Krümmungen haben ihre PHYSIKALISCHEN Ursachen immer in mehr bzw. weniger Material in einer (Blick-)Richtung.

Auf den physikalischen Raum übertragen kommt noch der dynamische Aspekt dazu:
Raumwachstum verursacht negative Krümmungen = (Partiell) höhere Anzahl / Vermehrung Raumquanten im Wachstumsgebiet.
Gravitation verursacht positive Krümmungen = (Partiell) geringere Anzahl / Verringerung Raumquanten im Schrumpfungsgebiet.

Halten sich Gravitation und Raumwachstum die Waage hätten wir ein stationäres Universum mit einer insgesamten (= Summe) Krümmung von k=0.
Wäre die Gravitation stärker als das Raumwachstum würde das Universum sich zusammenziehen (= k>1).
Wäre das Raumwachstum stärker als die Gravitation würde das Universum immer weiter expandieren (= k<1).

So sehe ich das. Da unser Universum angeblich immer schneller expandiert müsste sich das auch in seiner Krümmung widerspiegeln: NEGATIV.

Deshalb passt auch ein (PHYSIKALISCHER!) Torus wesentlich besser zur realen Beschreibung unser Universums: Denn der hat auch an verschiedenen Stellen verschiedene Krümmungen mit Gesamt-Krümmung k=0.
Aber natürlich kann man lokal als Näherung ungekrümmte Euklidik annehmen.

(Alles IMHO, versteht sich)

In Bezug auf das Weltall wird die Torusgeometrie auch diskutiert:
http://www.rzuser.uni-heidelberg.de/~q61/sdw_torus_fac.pdf (http://www.rzuser.uni-heidelberg.de/%7Eq61/sdw_torus_fac.pdf)
[url=http://www.quanten.de/forum/showthread.php5?p=43331&postcount=52] :confused:

zeitgenosse
04.11.09, 10:09
Wo bekomme ich so ein vollkommen elastisches Papierblatt her?

Im Kontext geht es um die Unterschiede zwischen Kugel und Torus, z.B.:

Die Oberfläche eines Torus ist räumlich flach. Flach bedeutet im Kontext nicht, dass es keine Krümmungen gibt. Bei der Winkelsumme im Dreieck ist dies feststellbar. Bei einer Kugeloberfläche kommt es zum 'spärischen Exzess'. Aus diesem Grunde hat ein Torus nicht dieselbe Geometrie, ist somit auch nicht positiv gekrümmt. Topologisch ist er auch anders, weil er wie ein Donut ein Loch besitzt. Eine Henkeltasse ist topologisch homöomorph zu einem Torus. Folglich handelt es sich beim eingebetteten Torus - im Unterschied zur Kugeloberfläche - nicht um eine einfach zusammenhängende Fläche. Ein Seil, das den Torus durch das Loch hindurch umschlingt, lässt sich nicht beliebig zusammenziehen. Auf einer 2-Sphäre hingegen kann ich eine Schlinge bis auf einen verbleibenden Punkt zusammenziehen.

Gr. zg

Timm
04.11.09, 10:18
Hallo SCR,

hier hast Du ein hyperbolisches Gebilde Torus genannt, jedenfalls hatte ich das so interpretiert:

http://www.quanten.de/forum/showpost.php5?p=43315&postcount=4

Erkläre mir doch bitte, wie ich eine Graphik so kopiere, daß sie orginal, also nicht als Link erscheint.

Die anderen Fragen hat Dir schon Zg beantwortet. Man nennt das stetige Transformation. Aus der Ebene kannst Du ohne einen Zerreißvorgang alle euklidischen Räume machen: Zylinder, Torus, Möbius-Band (jetzt kommt bestimmt eine freundliche Replik von Möbius) und Kleinsche Flasche. Wenn Du solche Dinge anzweifelst, kannst Du auch den Satz von Phytagoras anzweifeln.

Gruß, Timm

SCR
04.11.09, 10:19
Hallo zg,

[1] Ein Seil, das den Torus durch das Loch hindurch umschlingt, lässt sich nicht beliebig zusammenziehen.
[2] Auf einer 2-Sphäre hingegen kann ich eine Schlinge bis auf einen verbleibenden Punkt zusammenziehen.
Dieses Beispiel verstehe ich nicht: Was hat denn das eine mit dem anderen zu tun?
Wenn ich ein Seil komplett um die 2-Sphäre lege und zusammenziehe erhalte ich das gleiche Ergebnis wie [1]: Im Knoten steckt "die Kugel" drin -> Das wäre vergleichbar.
Wenn ich auf einer Torus-Oberfläche eine Schlinge zusammenziehe erhalte ich das gleiche Ergebnis wie [2]: Im Knoten steckt nichts (von mir aus auch ein Punkt) drin -> Das wäre vergleichbar.
Ist das Beispiel deshalb nicht "Äpfel mit Birnen"?

Wenn mir das Beispiel dagegen lediglich sagen will ein Torus (oder auch eine Tasse - eben alles "mit einem Henkel") sei (zumindest topologisch) eine Kugel mit einem Loch in der Mitte - Dann ist das zwar schön, aber das kann man meines Erachtens nach auch einfacher aussagen.

Aber isch bin ja auch nur Laie. ;)

SCR
04.11.09, 10:30
Hallo Timm,
hier hast Du ein hyperbolisches Gebilde Torus genannt, jedenfalls hatte ich das so interpretiert:
Ja, aber das ist doch auch ein (genauer gesagt: DER für die Fragestellung relevante) Ausschnitt von einem Torus ("Der innere Kreis") - Was soll denn daran falsch sein? :confused:
Erkläre mir doch bitte, wie ich eine Graphik so kopiere, daß sie orginal, also nicht als Link erscheint.
Gerne:
Voraussetzung: Deine Grafik liegt bereits irgendwo im WWW und Du hast den Link dazu (http ://www. ...).
Diesen Link schließt Du jetzt in IMG-Tags in Deinen Beitrag ein:
[ IMG ]http://www. ...[ /IMG ]
(Leerzeichen in den eckigen Klammern weglassen! - Fertig. Du kannst Dir ja auch noch einmal über die Zitieren-Funktion meinen obigen Beitrag mit der enthaltenen Grafik anschauen: Dann hast Du auch ein konkretes Beispiel)
Brauchst Du noch mehr Info? :rolleyes:
Aus der Ebene kannst Du ohne einen Zerreißvorgang alle euklidischen Räume machen: Zylinder, Torus, Möbius-Band (jetzt kommt bestimmt eine freundliche Replik von Möbius) und Kleinsche Flasche.
Mathematisch: Ja. Physikalisch: Nein (zumindest nicht in allen Fällen).
Noch einmal: Mathematik und Physik sind keine zwangsläufigen Identitäten!
Wenn Du solche Dinge anzweifelst, kannst Du auch den Satz von Phytagoras anzweifeln.
Muß ich den in gekrümmten Räumen nicht sogar anzweifeln (?) :confused:

Bauhof
04.11.09, 10:37
Erkläre mir doch bitte, wie ich eine Graphik so kopiere, daß sie orginal, also nicht als Link erscheint.Gruß, Timm
Hallo Timm,

wenn du hier eine Grafik einbinden willst: dazu gibt es im Schreibfenster oben in der Leiste einen Button, der meldet sich mit "Grafik einfügen", wenn du mit der linken Maustaste hinfährst. Wenn du den Button anklickst, dann erscheint ein kleines Fenster, in dem du deinen Link zur Grafik eintippst oder reinkopierst. Oder du setzt vor dem Link und nach dem Link . IMG ist das Operationszeichen für "Image". Beispiel:

{IMG}http://www.eugen-bauhof.homepage.t-online.de/Bilder/Drilling-1.gif{/IMG}

Die Eckklammern habe ich durch geschweifte Klammern ersetzt, damit das Bild jetzt hier nicht eingestellt wird.

M.f.G. Eugen Bauhof

P.S.
Ich sehe gerade, SCR hat es auch erklärt.

zeitgenosse
04.11.09, 11:45
Dann ist das zwar schön, aber das kann man meines Erachtens nach auch einfacher aussagen.

Topologisch muss man die Objekte ja irgendwie sinnvoll klassifizieren.

So hat die Kugel das Geschlecht 0, ein Torus das Geschlecht 1, ein Doppeltorus das Geschlecht 3 usw. Gelegentlich verwendet man beim Verkleben von Henkelkörpern sog. Heegaard-Diagramme.

p.s.
Vermutlich täte es nicht schaden, wenn du bei Gelegenheit doch einmal ein Lehrbuch der Topologie zur Hand nähmest. Wir können hier nicht jeden Schritt einzeln erklären. Eine gewisse Eigenleistung inkl. einem Erkenntniswachstum sollte irgendwann erkennbar sein.

Gr. zg

SCR
04.11.09, 12:34
http://upload.wikimedia.org/wikipedia/commons/2/26/Mug_and_Torus_morph.gif
Geschlecht = "Anzahl Löcher im Käse". Ja - Und? :rolleyes: (Und noch einmal konkret: Was soll Dein oben zitiertes Beispiel denn jetzt eigentlich aussagen? Loch im Käse, oder etwa nicht?)
Und ich verstehe jetzt nicht: Welche Schritte meinst Du / fehlen denn?

Unabhängig davon wäre ich aber schon sehr dankbar über eine Literaturangabe die genau den Punkt "Was ist aus der Topologie der Mathematik nicht auf die Physik 1:1 übertragbar?" betrachtet.
Da finde ich einfach nichts.
Konkret: Die mathematische Topologie geht davon aus, dass die von ihr betrachteten Dimensionen aus unendlich vielen Punkten gebildet werden.
Wenn ich in der Physik eine Quantisierung unterstelle bestehen deren Dimensionen aber aus einer endlichen Anzahl von Quanten/Punkten.
Und - im Gegensatz zu den mir bekannten mathematischen Modellen - kann dabei die Anzahl der Quanten sich im Laufe der Zeit verändern (Stichwort Raumexpansion).
Das ist in meinen Augen ein himmelweiter Unterschied (siehe meine vorherigen Ausführungen).
Kennst Du dazu was? :rolleyes: (Gerne alternativ: Siehst Du das etwa anders?)

Aus diesem Grunde hat ein Torus nicht dieselbe Geometrie, ist somit auch nicht positiv gekrümmt.
Ein PHYSIKALISCHER Torus ist an seiner inneren Seite (überwiegend) negativ, an seiner äußeren Seite (überwiegend) positiv gekrümmt.
Handelt es sich um einen regelmäßigen/homogenen/symmetrischen Torus heben sich alle Krümmungen in Summe zu 0 auf.
Die Mathematik mag das anders sehen, eine quantenbasierte Physik kann das nicht anders sehen - Egal ob aus Sicht eines inneren oder äußeren Beobachters (IMHO).

Ansonsten eher interessehalber: Wo werden denn Körper mit ineinander verschlungenen Henkeln betrachtet ("Mit sich selbst verknotete Körper")? Da habe ich auch noch nichts gefunden. Ist aber auch nicht so wild: Ich denke nicht dass das im hier besprochenen Kontext eine große Rolle spielt.

zeitgenosse
04.11.09, 13:27
Ein PHYSIKALISCHER Torus ist an seiner inneren Seite (überwiegend) negativ, an seiner äußeren Seite (überwiegend) positiv gekrümmt.

Beim Torus wird im Grunde nur die Oberfläche betrachtet und diese ist euklidisch.

Gr. zg

Timm
04.11.09, 14:19
Danke Dir, Eugen und SCR für Eure Hilfsangebote.

Ich wollte SCR's hyperbolisches Gebilde mit der Maus kopieren, was nicht ging. Aber wie ich gerade feststellte, klappt es mit der Zitat-Funktion,

Gruß, Timm

SCR
04.11.09, 14:35
Hallo zg,
Beim Torus wird im Grunde nur die Oberfläche betrachtet und diese ist euklidisch.
Das IST - es tut mir leid - so pauschal festgestellt falsch:
1. "können" und "sein" sind von eklatant unterschiedlicher Bedeutung (In nach meiner Einschätzung "guter" Fachliteratur steht explizit "kann", in "schlechter" Literatur "ist").
2. WIE schaust Du: Mathematisch oder Physikalisch?
3. Von WO aus schaust Du: Von Außen? Von Innen?
4. Und rein interessehalber:
- Kann man bei Deiner Aussage den Begriff "im Grunde" weglassen? Ansonsten: In welchen Fällen wäre die Torus-Oberfläche denn als Nicht-Euklidisch anzusehen?
- Warum schließt Du das Innere eines Torus aus? Ich folgere daraus, dass das Torus-Innere Deiner Meinung nach anscheinend Nicht-Euklidisch ist. Könntest Du das bitte etwas präzisieren?
Seite 8 des PDFs / Seite 93 des Dokuments - oben (http://www.google.de/url?sa=t&source=web&ct=res&cd=11&ved=0CAYQFjAAOAo&url=http%3A%2F%2Fwww.wissenschaft-online.de%2Fartikel%2F835346&rct=j&q=Torus+negative+Kr%C3%BCmmung&ei=lk_wSrCECdD__AbylrifBw&usg=AFQjCNEs_yefJsGgXND6r44BlOk4gUNkwQ)
Hand auf's Herz: Hast DU das gelesen oder nicht? (von mir aus gerne auch in einer anderen Quelle: Aber das Verlinkte halte ich mit für die verständlichste Darstellung, die ich bisher dazu gelesen habe)

Und nebenbei: Ich wundere mich schon ein wenig, dass man, wenn man konkret nachhakt, doch zuweilen eine eher ausweichende Antwort bekommt.
Woran liegt's? :rolleyes:

Timm
04.11.09, 16:34
Hallo SCR,

manche, darunter auch Zg geben sich Mühe, Dir weiterzuhelfen. Nimm es mir nicht übel, wenn ich das so sage, aber Du neigst dazu, die Dinge unnötig zu verkomplizieren.

Bei Andreas Müller findet sich eine kurze Einführung unter "Topologie", die insbesondere auf die Winkelsumme abhebt. Alles was darüber hinausgeht, kann auch ich Dir nicht erklären, dafür verstehe ich viel zu wenig davon.

http://www.wissenschaft-online.de/astrowissen/lexdt_t04.html#topol

Damit will ich es zu diesem Thema gut sein lassen,

Gruß, Timm

zeitgenosse
04.11.09, 17:26
Ansonsten: In welchen Fällen wäre die Torus-Oberfläche denn als Nicht-Euklidisch anzusehen?

Ein Torus lässt sich bei Zulassung von Biegung und Stauchung aus einem Zylinder herstellen und dieser aus einem Rechteck. Das sagte ich bereits. Umgekehrt lässt sich auch ein Torus unter Zulassung obiger Randbedingungen wieder in eine Fläche zurückverwandeln. Von einer Kugeloberfläche lässt sich dies nicht behaupten. Ganz gleich, wo du die Schere ansetzt, es bleiben sog. Singularitäten zurück. Fazit des Gesagten ist, dass ein Torus die euklidische Struktur beibehält, selbst wenn er wie ein Donut gebogen ist.

Im Kern geht es um die 'innere Krümmung' einer Fläche, die Gauß mit seinem 'Theorema egregium' (1828) beschrieben hat. So ist eine Sphäre 'nomen est omen' sphärisch, ein Torus euklidisch und eine Brezel hyperbolisch. Was du beim Torus als krumm empfindest, ist die 'äussere Krümmung'. Dein Rücken ist vermutlich auch nicht gerade; doch diese Art von Krümmung entscheidet nicht darüber, ob ein Körper der elliptischen oder der hyperbolischen Geometrie zuzuordnen ist. Äussere Krümmungen lassen sich ohne Weiteres im Rahmen der vertrauten euklidischen Geometrie behandeln.

Warum schließt Du das Innere eines Torus aus? Rhetorische Frage:

Weshalb schliesse ich im Kontext das Innere einer Kugel aus? Deshalb etwa, weil wir uns bezüglich der geometrischen Struktur auf die Oberfläche eines Körpers zu beziehen haben?

Schneide ich einen Volltorus (Donut) in Scheiben, erhalte ich viele zylinderförmige Schnitten mit euklidischer Struktur. Aus diesem Grund betrachte ich in Bezug auf die Krümmungsfrage lediglich die 'Aussenhaut' eines Torus. Das Innere interessiert dazu nicht. Wenn du eine echte hyperbolische Fläche suchst, nimm bspw. die Pseudosphäre von Beltrami oder meinetwegen einen ausgedienten Pferdesattel. Darauf magst du dann Dreiecke einzeichnen, deren Winkelsumme kleiner als zwei Rechte ist.

Hand auf's Herz: Hast DU das gelesen oder nicht? Inzwischen habe ich es überflogen. Der Artikel ist i.o. Damit habe ich mich allerdings bereits früher beschäftigt.

Siehe unter 'Die Poincaré-Vermutung':

http://www.relativ-kritisch.net/forum/viewtopic.php?t=1305

Will damit nur sagen:

Die Thematik an sich ist mir nicht völlig neu.

Doch offensichtlich liest du selbst nicht, was du anderen empfiehlst. Ansonsten hättest du die von mir erwähnte Probe mit der Seilschlinge nicht in Frage gestellt.

Gr. zg

Bauhof
04.11.09, 17:54
Danke Dir, Eugen und SCR für Eure Hilfsangebote. Ich wollte SCR's hyperbolisches Gebilde mit der Maus kopieren, was nicht ging. Aber wie ich gerade feststellte, klappt es mit der Zitat-Funktion, Gruß, Timm

Hallo Timm,

da haben wir uns offenbar missverstanden. Das, was du eigentlich machen wolltest, habe ich gerade gemacht: Mit der rechten Maustaste auf die Zeichnung klicken und mit Bild speichern unter... herunterladen.

M.f.g. Eugen Bauhof

SCR
04.11.09, 20:00
manche, darunter auch Zg geben sich Mühe, Dir weiterzuhelfen.
Du hast ja Recht :o : Entschuldige zg und Danke für Deine Bemühungen! (Gilt natürlich auch für die anderen)

Und jetzt lese ich erst einmal Deinen letzten Beitrag - und werde nicht wieder sofort antworten ;).

(EDIT: Auch wenn's mich nach dem ersten Überfliegen schon wieder in den Fingern jucken würde ;).)

JoAx
04.11.09, 22:10
Wo bekomme ich so ein vollkommen elastisches Papierblatt her?


Raumzeit?
Deswegen auch nicht stofflich? ;)


Gruss, Johann

SCR
04.11.09, 23:17
Raumzeit? Deswegen auch nicht stofflich? ;)
http://94.75.224.58/forums/style_emoticons/DarksideRG/ShutUp.gif (Aber bald ;))

zeitgenosse
05.11.09, 02:22
NACHTRAG mit Präzisierungen:

Unter den geschlossenen bzw. kompakten Flächen besitzt nur der 2-Torus (T^2) eine euklidische Metrik. Die 2-Sphäre (S^2) bzw. Kugeloberfläche ist dagegen sphärisch bzw. deren Geometrie folgt der Riemannschen. Näherungsweise kann die Umgebung eines Punktes auf der Sphäre euklidisch ausfallen (empirisch um so mehr, je grösser der Weltradius ist). Trotzdem verbleibt ihre Metrik in globo nichteuklidisch.

Den 2-Torus erhält man, indem in einem Rechteck in der euklidischen Ebene gegenüberliegende Seiten paarweise identifiziert werden. In praxi werden sie natürlich zu einem offenen Zylinder verklebt.Das zweite Seitenpaar kann nicht verzerrungsfrei verklebt werden. Die ursprüngliche Zylinderlänge findet sich nur noch in der neutralen Faser. Trotzdem bleibt beim 2-Torus die euklidische Metrik erhalten, wovon man sich bspw. durch die Winkelsumme im Dreieck überzeugen mag. Abstrakt lassen sich die identischen Seitenpaare zu einem Punkt reduzieren, in welchem gewissermassen vier rechte Winkel miteinander verklebt werden. Auch deshalb spricht man beim 2-Torus von einem euklidischen Objekt. Mathematisch exakt lässt sich der 2-Torus als Quotient der Euklidischen Ebene bestimmen.

In direkter Analogie konstruiert man den 3-Torus, indem die gegenüberliegenden Flächen eines Quaders paarweise im Euklidischen Raum identifiziert werden. Die innere Geometrie des Quaders wird dabei nicht verändert. Kosmologisch würde sich dies so auswirken, dass ein Lichtstrahl, der an die rechte Grenzfläche stösst unmittelbar auf der linken Seite erschiene. Blickt ein Beobachter im 3-Torus nach rechts, begegnet ihm somit die linke Seite. Weil ein Lichtstrahl diesen Hypertorus mehrmals durchlaufen könnte, erblickt der Beobachter möglicherweise mehrere Abbilder eines Teiles der Welt, die zudem infolge der Massenverteilung etwas verzerrt würden. Ihm selbst erschiene der Raum quasi-unendlich zu sein. Optisch wäre dies vergleichbar mit einem Spiegelkabinett, mit dem Unterschied, dass im verspiegelten Innenraum die ungeradzahligen Abbilder spiegelverkehrt erschienen.

Flächen mit Geschlecht 2 et ultra wie z.B. die Brezel tragen bereits hyperbolischen Charakter. Schneidet man bspw. eine Fläche vom Geschlecht 2 (Henkelkörper) entlang bestimmter Linien auf, lässt sich die zuvor geschlossene Fläche zu einem Achteck auffächern. Auch hier kann man die Seiten paarweise identifizieren, um schliesslich in der Abstraktion sämtliche Ecken in einem einzigen Punkt zu konzentrieren. Dabei wird das Achteck jedoch verzerrt, so dass in diesem Fall keine euklidische Geometrie vorliegt. Benutzt man zum Vergleich das Poincarésche Kreisscheibenmodell, lassen sich die hyperbolischen Eigenschaften des untersuchten Objektes allmählich erkennen.

p.s.
Um tiefer in die algebraische Topologie einzudringen, benötigt man m.E. ein gut verständliches Lehrmittel, z.B.:

Ossa, Topologie (Vieweg+Teubner)

Als populärwissenschaftliche Einführung ist auch das bereits erwähnte Buch von Ossermann (Geometrie des Universums) nützlich.

Selbst befindet sich der Schreiberling auch erst am Anfang dieser zweifellos hochinteressanten Reise ins Herz der Topologie.

Gr. zg

zeitgenosse
05.11.09, 15:29
Interessant im Kontext ist auch der nächsthöhere Torus. Werden nämlich beim Tesserakt dessen acht gegenüber liegende Begrenzungswürfel paarweise miteinander verheftet, entsteht ein 4-Torus. Im 3-Raum ist solches nur schwer vorstellbar. In höheren Dimensionen wäre es kein Problem, doch fehlen uns dafür offenbar die entsprechenden Sinnesorgane.

An sich ist der Tesserakt ein Hypercubus. Dieser verhält sich zum Würfel, wie dieser zum Quadrat. Der Topologe spricht auch von einem 4-Kubus resp. 8-Zeller.

Tesserakt:

http://upload.wikimedia.org/wikipedia/commons/4/46/Hcube_fold.gif

Schlegeldiagramme des Hypercubus:

http://upload.wikimedia.org/wikipedia/commons/2/22/Hypercube.svg

http://upload.wikimedia.org/wikipedia/commons/a/a2/Schlegel_wireframe_8-cell.png

Hypercube dynamisch:

http://upload.wikimedia.org/wikipedia/commons/d/d7/8-cell.gif

Der Betrachter muss sich zunächst allerdings in Geduld üben, um dem Gebilde einen nachhaltigen Sinn abzugewinnen; doch nach und nach stellt sich im mentalen Sektor eine gewisse Befriedigung ein.

Interessant ist zudem, dass Salvador Dali den Tesserakt als 'Corpus Hypercubus' in seinem tiefgründigen Gemälde des gekreuzigten Gottes (Crucifixion) verwendet:

http://upload.wikimedia.org/wikipedia/en/0/09/Dali_Crucifixion_hypercube.jpg

Will der Künstler damit etwa die weltumfassende, Raum und Zeit überwindende Bedeutung des expiatorischen Opfers Christi ausdrücken?

Gr. zg

JoAx
05.11.09, 17:08
Hallo SCR,


Ich habe es jetzt doch einmal auf Basis von JoAx Bild das, was ich meine, grob eingezeichnet:
http://img509.imageshack.us/img509/1394/kruemm.jpg
Und wenn der relevante mittlere Bereich eines Torus noch etwas "breiter" ist dann können sogar mehrere Geodäten dort nebeneinander parallel verlaufen.

ich denke, dass wenn das Universum im "Moment" "hornartig" ist, das Bild etwas angepasst werden muss.

1. Eine der Seiten, links oder rechts von deiner Linie muss weg.
2. Die Zeit in die Mitte der Figur und mit Richtung breiterwerdend gelegt werden.

Wenn man sich jetzt den Verlauf einer Geodäte überlegt, dann wird klar, dass sie nicht deiner Zeichnung entsprechen kann. Eine Bewegung entlang der von dir eingezeichneten Linie würde einer instanten Lageveränderung entsprechen. ;)


Gruss, Johann

PS: Das Bild ist übrigens nicht von mir, im www gefunden.

Timm
05.11.09, 17:17
Hypercube dynamisch:

http://upload.wikimedia.org/wikipedia/commons/d/d7/8-cell.gif

Der Betrachter muss sich zunächst allerdings in Geduld üben, um dem Gebilde einen nachhaltigen Sinn abzugewinnen; doch nach und nach stellt sich im mentalen Sektor eine gewisse Befriedigung ein.

Diese dynamische Darstellung des Hypercubus wirkt geheimnisvoll. Man könnte jede Phase aus Draht basteln und hätte doch nur die Projektion eines 4 Kubus. Gibt es eigentlich Mathematiker, die sich einen mentalen 4 Sektor so antrainiert haben, daß zumindest eine angenäherte Vorstellung des 4 Kubus entsteht?


Interessant ist zudem, dass Salvador Dali den Tesserakt als 'Corpus Hypercubus' in seinem tiefgründigen Gemälde des gekreuzigten Gottes (Crucifixion) verwendet:

http://upload.wikimedia.org/wikipedia/en/0/09/Dali_Crucifixion_hypercube.jpg

Will der Künstler damit etwa die weltumfassende, Raum und Zeit überwindende Bedeutung des expiatorischen Opfers Christi ausdrücken?


In meinem Dali Band findet sich der 'Corpus Hypercubus' unter Mystik. Es ist aber bekannt, daß Dali gerne Elemente aus der modernen Wissenschaft aufgegriffen hat,

Gruß, Timm

zeitgenosse
06.11.09, 08:12
Und wenn der relevante mittlere Bereich eines Torus noch etwas 'breiter' ist, dann können sogar mehrere Geodäten dort nebeneinander parallel verlaufen.

Erstens ist die skizzierte Fläche kein Torus.

Zweitens muss man die Gaußsche Krümmung lokal bestimmen durch den Kehrwert der Radien der Schmiegekreise:

K = 1/(R1*R2)

Im vorliegenden Fall ergibt sich wegen +R1 und -R2 zweifellos eine negative Krümmung.

Gr. zg

SCR
06.11.09, 15:15
http://sims2marktplatz.net/forum/images/smilies/neu/fessel1.gif ;)

Bauhof
06.11.09, 15:53
http://sims2marktplatz.net/forum/images/smilies/neu/fessel1.gif ;)
Hallo SCR,

was willst du uns damit sagen? War das eine Antwort auf den Beitrag des Zeitgenossen?

M.f.G. Eugen Bauhof

SCR
06.11.09, 21:52
Hallo Bauhof,
was willst du uns damit sagen?
Das ich was zu sagen hätte.
War das eine Antwort auf den Beitrag des Zeitgenossen?
Dementsprechend: Formell gesehen Ja, inhaltlich Nein.

SCR
07.11.09, 08:12
So, jetzt habe ich mich - glaube ich - lange genug zusammengerissen :D:

Die Topologie des SCR - auch bekannt als die Topologie des langen Quadrats (in memory to EMI ;)):

Wir haben eine Kugel. Eine Kugel ist ein Batzen. Egal welche Form dieser Batzen hat: Ballförmig, Eiförmig, ausgerollt wie eine Wurst, ... - Er ist in Summe immer positiv gekrümmt.

Das Gegenteil von einem Batzen ist "kein Batzen". Da der "Kein Batzen" das Gegenteil von einem "ein Batzen" ist ist er negativ gekrümmt.

Kombiniere ich einen Batzen mit einem "Kein Batzen" erhalte ich einen Batzen mit einem Loch.
Dabei summieren sich die Krümmungen des "Ein Batzen" mit denen des "Kein Batzen" zu Null -> Ein Batzen mit ein Loch ist in Summe ungekrümmt.

Kombiniere ich einen Batzen mit zwei "Kein Batzen" erhalte ich einen Batzen mit zwei Löchern. Dabei summieren sich die Krümmungen des "Ein Batzen" mit denen der "Kein Batzen" zu einem negativen Ergebnis -> Ein Batzen mit zwei Löchern ist in Summe (einfach) negativ gekrümmt.

Kombiniere ich einen Batzen mit drei "Kein Batzen" erhalte ich einen Batzen mit drei Löchern. Dabei summieren sich die Krümmungen des "Ein Batzen" mit denen der "Kein Batzen" zu einem negativen Ergebnis -> Ein Batzen mit zwei Löchern ist in Summe (zweifach) negativ gekrümmt.

...

Und alle Batzen mit Löchern kann ich mir mit einer Schnur an die Wand hängen.

-------------------------
Ich bitte um Korrektur falls hier etwas Falsches steht.

Marco Polo
07.11.09, 08:29
Das Gegenteil von einem Batzen ist "kein Batzen".

.....:D.....

zeitgenosse
07.11.09, 08:36
Wir haben eine Kugel. Eine Kugel ist ein Batzen. Egal welche Form dieser Batzen hat: Ballförmig, Eiförmig, ausgerollt wie eine Wurst, ... - Er ist in Summe immer positiv gekrümmt.

Topologisch gesehen ist eine Wurst homöomorph, d.h. bezüglich der Form gleichwertig zu einer Kugel, einer Birne, einem Brot, einer Münze usw.

In Bezug auf die Metrik ist es allerdings so, dass - wenn die Wurst wie ein Zylinder aussähe -, in diesem Fall eine euklidische Geometrie existent wäre. Denn Zylinderoberflächen lassen sich besonders einfach auf einer Ebene abrollen und umgekehrt aus einem Rechteck herstellen. Sie erben damit die euklidische Metrik und besitzen demzufolge die Krümmung k = 0.

Der Rest deines Elaborats ist bei etwas Grosszügigkeit bezüglich der Terminologie i.o.

Um zu - im Kontext - sauberen Bergriffen zu gelangen, solltest du dich auch mit den Betti-Zahlen befassen.

Gr. zg

Marco Polo
07.11.09, 09:02
Topologisch gesehen ist eine Wurst homöomorph, d.h. bezüglich der Form gleichwertig zu einem...Brot...

Hmm...bin gespannt, ob mein Mezger das ähnlich sieht... :rolleyes:

SCR
07.11.09, 09:23
Hallo zg,
In Bezug auf die Metrik ist es allerdings so, dass - wenn die Wurst wie ein Zylinder aussähe -, in diesem Fall eine euklidische Geometrie existent wäre. Denn Zylinderoberflächen lassen sich besonders einfach auf einer Ebene abrollen und umgekehrt aus einem Rechteck herstellen. Sie erben damit die euklidische Metrik und besitzen demzufolge die Krümmung k = 0.
Interessanter Hinweis - Danke zg.
Denn da muß irgendetwas falsch sein ... :D
Ich denke, ich weiß auch was: Du unterschlägst die Seitenflächen des Zylinders.
Es handelt sich um Kreisscheiben. Ausgebeult zwei Halbkugeln. Ergeben zusammengesetzt eine Kugel = positiv gekrümmt. In Summe mit der ungekrümmten Mantelfläche sollte ein Zylinder deshalb ebenfalls insgesamt eine positive Krümmung aufweisen - IMHO.
Aber ich lasse es mir noch einmal durch den Kopf gehen.
Um zu - im Kontext - sauberen Bergriffen zu gelangen, solltest du dich auch mit den Betti-Zahlen befassen.
Meine Begriffe sind sauber da eindeutig und allgemein verständlich - Ich verstehe die Kritik nicht :p ;).
Betti-Zahlen :rolleyes: - Noch nie gehört. Mal anschauen ...

EDIT: Was gefunden
Die Homologie und Kohomologie sind Hilfsmittel zur Beschreibung komplizierter mathematischer Objekte, wie zum Beispiel topologischer Räume. Es begann mit der Theorie der Bettizahlen (benannt nach dem italienischen Mathematiker Betti), wo man Mannigfaltigkeiten gewisse charakteristische Zahlen zuordnete. Zum Beispiel sind die Bettizahlen (in Graden Null bis Zwei) eines Torus (die Oberfläche eines Ringes) 1,2,1: Der erste Wert 1 bedeutet, dass der Torus zusammenhängend ist, der dritte Wert 1 Orientierbarkeit, also dass man Innen- und Außenfläche unterscheiden kann. Die 2 kommt davon, dass es zwei linear unabhängige Wege gibt, nämlich die Umläufe um den Umfang des Ringes und um den kreisförmigen Querschnitt. Jeder andere geschlossene Weg auf dem Torus lässt sich in eine lineare Kombination dieser beiden deformieren (Abb. 1). Die Bettizahlen erlauben das Unterscheiden verschiedener topologischer Mannigfaltigkeiten: Der Torus ist äquivalent zu einer Kugel mit einem Henkel, das heißt, er kann in eine solche kontinuierlich deformiert werden. Für eine Kugel mit zwei Henkeln lauten die Bettizahlen 1,4,1, sie ist also nicht zum Torus äquivalent. Dies ist zwar anschaulich plausibel, doch reicht dies nicht, um die Möglichkeit einer eventuell sehr raffinierten Konstruktion einer Deformation auszuschließen.
Mal sehen ...

Bauhof
07.11.09, 10:55
Ich denke, ich weiß auch was: Du unterschlägst die Seitenflächen des Zylinders.
Es handelt sich um Kreisscheiben. Ausgebeult zwei Halbkugeln. Ergeben zusammengesetzt eine Kugel = positiv gekrümmt. In Summe mit der ungekrümmten Mantelfläche sollte ein Zylinder deshalb ebenfalls insgesamt eine positive Krümmung aufweisen - IMHO. Aber ich lasse es mir noch einmal durch den Kopf gehen.

Hallo SCR,

Die Seitenflächen des Zylinders gehören in topologischer Hinsicht nicht dazu. Warum? Du kannst den Zylinder nicht aufrollen, ohne dass Risse an den "Seitenflächen" entstehen.

Die Argumentation des Zeitgenossen ist in dieser Hinsicht m.E. völlig korrekt.

M.f.G. Eugen Bauhof

SCR
07.11.09, 12:47
Hallo Bauhof,
Die Seitenflächen des Zylinders gehören in topologischer Hinsicht nicht dazu. Warum? Du kannst den Zylinder nicht aufrollen, ohne dass Risse an den "Seitenflächen" entstehen.
Ich kann mir doch rissfrei aus einer Kugel einen Zylinder kneten.
Ich kann ein Blatt Papier "richtig eng" einrollen, die Blattkanten bilden dann die Stirnflächen - völlig rissfrei.
Der Mantel ist in beiden Fällen krümmungsfrei, die Stirnflächen sind positiv gekrümmt. -> :confused:
Das hier wäre ein Zylinder mit negativ gekrümmten Stirnflächen (linkes Objekt):
http://img509.imageshack.us/img509/1394/kruemm.jpg
(Die Mitte noch etwas nach links und nach rechts dehnen damit dort der Zylindermantel entsteht)
Dieser Zylinder wäre insgesamt negativ gekrümmt.
Den kann ich natürlich nicht aus dem "einen Batzen" kneten - Den negativ gekrümmten Zylinder kriege ich nur aus dem "Keinen Batzen" geformt.

EDIT: Das macht für mich so oder so recht wenig Sinn: Bei der Wurst und beim Brot nehmen wir die Zipfel mit dazu, beim Zylinder ignorieren wir sie geflissentlich. :rolleyes:

SCR
08.11.09, 09:53
Ich denke es liegt rein an begrifflichen Unschärfen:
Das IST - es tut mir leid - so pauschal festgestellt falsch:
1. "können" und "sein" sind von eklatant unterschiedlicher Bedeutung (In nach meiner Einschätzung "guter" Fachliteratur steht explizit "kann", in "schlechter" Literatur "ist").

Aus einer euklidischen Ebene kann man einen ungekrümmten ZylinderMANTEL formen.
Und aus diesem ZylinderMANTEL wiederum einen Torus (dazu brauct man keine Seitenflächen - Die würden beim Ringschluss eh "wegfallen").

Das trifft aber nicht auf einen Zylinder zu: Der ist positiv gekrümmt.
Ich denke, in unserem Fall wird in den entsprechenden Ausführungen fahrlässigerweise Zylinder synomym zu ZylinderMANTEL verwendet - was (dementsprechend dann auch in der jeweiligen Fachliteratur) absolut falsch ist.

Da bleibe ich dann noch lieber beim eindeutig definierten Batzen - Da kann es keine Mißverständnisse geben. :D

Bauhof
08.11.09, 10:11
Aus einer euklidischen Ebene kann man einen ungekrümmten ZylinderMANTEL formen.
Und aus diesem ZylinderMANTEL wiederum einen Torus (dazu brauct man keine Seitenflächen - Die würden beim Ringschluss eh "wegfallen").

Das trifft aber nicht auf einen Zylinder zu: Der ist positiv gekrümmt.
Ich denke, in unserem Fall wird in den entsprechenden Ausführungen fahrlässigerweise Zylinder synomym zu ZylinderMANTEL verwendet - was (dementsprechend dann auch in der jeweiligen Fachliteratur) absolut falsch ist.
Hallo SCR,

wer hier wirklich fahrlässig Zylinder synonym zu ZylinderMANTEL verwendet, möchte ich mal dahingestellt sein lassen. Der Zeitgenosse schrieb:

In Bezug auf die Metrik ist es allerdings so, dass - wenn die Wurst wie ein Zylinder aussähe -, in diesem Fall eine euklidische Geometrie existent wäre. Denn Zylinderoberflächen lassen sich besonders einfach auf einer Ebene abrollen und umgekehrt aus einem Rechteck herstellen. Sie erben damit die euklidische Metrik und besitzen demzufolge die Krümmung k = 0.

Siehe hierzu http://www.quanten.de/forum/showpost.php5?p=43524&postcount=87

Ich meine, er hat hier mit Zylinder nur den Zylindermantel gemeint. Aber der Zeitgenosse sollte selber klarstellen, was er gemeint hat, damit hier nicht ständig aneinander vorbeigeredet wird.

M.f.G. Eugen Bauhof

SCR
08.11.09, 10:20
Hallo Bauhof,
nur um das klar zu stellen: Das sollte absolut keinen Angriff auf zg darstellen - Das steht "so verkehrt" auch in gängiger Standard-Fachliteratur.

So ist die korrekte Darstellung:
http://img41.imageshack.us/img41/6030/zylinder.jpg

Und für die Herstellung eines Torus "genügt" eben "die ungekrümmte Mitte" -> Torus ungekrümmt.

Selbstverständlich: IMHO.

wer hier wirklich fahrlässig Zylinder synonym zu ZylinderMANTEL verwendet, möchte ich mal dahingestellt sein lassen.
Wie darf/soll ich das verstehen?
Werde bitte konkret - Und das ist eine ernstgemeinte Bitte meinerseits.
Ansonsten ist das in meinen Augen nämlich eher "untere Schublade", Herr Bauhof.

möbius
08.11.09, 10:22
...

Aber der Zeitgenosse sollte selber klarstellen, was er gemeint hat, damit hier nicht ständig aneinander vorbeigeredet wird.

M.f.G. Eugen Bauhof

Guten (Sonn)Tag, Eugen Bauhof!
Ja ja - auch unter Mathematikern, Physikern usw. scheint ein wirklicher Diskurs, bei dem es um Verständigung geht, nicht ganz einfach zu sein...;)
Das macht aber nichts ...:D
Gruß, möbius

Bauhof
08.11.09, 11:25
Guten (Sonn)Tag, Eugen Bauhof! Ja ja - auch unter Mathematikern, Physikern usw. scheint ein wirklicher Diskurs, bei dem es um Verständigung geht, nicht ganz einfach zu sein...;) Das macht aber nichts ...:D Gruß, möbius

Hallo Möbius,

wem unterstellst du den hier im Forum, wer ein gelernter Physiker oder ein gelernter Mathematiker ist? Ich weiß es definitiv zur Zeit nur bei einem, nämlich bei Uli.

M.f.G. Eugen Bauhof

P.S.
Mit "gelernt" meine ich, mindestens mit Diplom. Ich selbst besitze keines dieser Diplome.

Bauhof
08.11.09, 11:34
Werde bitte konkret - Und das ist eine ernstgemeinte Bitte meinerseits.

Hallo SCR,

bitte geh nicht gleich hoch. Mit "...möchte ich mal dahingestellt sein lassen" meinte ich lediglich, dass ich kein Urteil darüber abgeben möchte, wer hier "fahrlässig" (dein Ausdruck) formulierte. Das klärst du bitte mit dem Zeitgenossen.

M.f.G. Eugen Bauhof

möbius
08.11.09, 15:54
Hallo Möbius,

wem unterstellst du den hier im Forum, wer ein gelernter Physiker oder ein gelernter Mathematiker ist? Ich weiß es definitiv zur Zeit nur bei einem, nämlich bei Uli.

M.f.G. Eugen Bauhof

P.S.
Mit "gelernt" meine ich, mindestens mit Diplom. Ich selbst besitze keines dieser Diplome.

Hallo Eugen Bauhof!
Ich weiss definitiv noch weniger;) , aber zumindest bei den Moderatoren (und einigen anderen Forumsteilnehmern) habe ich doch den Eindruck, dass durchaus erstaunliche mathematische und physikalische Kompetenzen vorhanden sind. Einige scheinen Ingenieure zu sein - und einem Ingeniör ist bekanntlich nichts zu schwör!;)
Ich besitze auch kein Mathematik- oder Physik-Diplom - und bin nur ein armer nichts-wissender Dr. phil.!
Aber auch damit kann man(n) durchs Leben kommen...:D
Grüsse, möbius

Bauhof
08.11.09, 16:31
Ich besitze auch kein Mathematik- oder Physik-Diplom - und bin nur ein armer nichts-wissender Dr. phil.! Aber auch damit kann man(n) durchs Leben kommen...:D Grüsse, möbius
Hallo Möbius,

ein armer nichts-wissender Dr. phil.! :D

Sei nicht so bescheiden. Wenn alle so bescheiden wären wie du, hätten wir Moderatoren
weniger Probleme... :rolleyes:

M.f.G. Eugen Bauhof

SCR
08.11.09, 18:28
Hallo Bauhof,
bitte geh nicht gleich hoch.
Dann war das ein Missverständnis :o - Sorry! :)

zeitgenosse
08.11.09, 20:23
Das sollte absolut keinen Angriff auf zg darstellen

Keine Bange, dermassen empfindlich bin ich nun auch wieder nicht.

Die innere Krümmung berechnet sich wie folgt:

K = k1 * k2 = (1/R1)(1/R2)

(R1, R2 sind die Radien der Schmiegekreise am betrefffenden Punkt der Krümmung)

Das im Kontext massgebende 'Theorema egregium' besagt;

Die Gaußsche Krümmung einer differenzierbaren Mannigfaltigkeit hängt nur von der Weg- und Winkelmessung in der Mannigfaltigkeit ab.Somit:

Relativ zum umgebenden euklidischen Ortsraum besitzt ein (Kreis)-Zylinder zwar eine gekrümmte Mantelfläche; doch diese gehört der äusseren Krümmung an.

Die äussere Krümmung kann nur festgestellt werden, indem die Lage des Raums im umgebenden, höherdimensionalen Raum, die so genannte Einbettung, betrachtet wird. Entscheidend bezüglich der Geometrie ist nur die innere Krümmung. Rollt man bspw. die Mantelfläche eines Kreiszylinders auf einer euklidischen Ebene ab, bleibt die Isometrie erhalten. Es verändern sich weder Längen- noch Winkelverhältnisse. Deshalb haben wir eine euklidische Metrik vorliegen. Objekte dieser Art haben eine verschwindende innere Krümmung k = 0.

Bei einer Kugeloberfläche ist das anders. Man denke dabei nur einmal an die Winkelsumme im Dreieck oder an Parallelen. Auf einem Zylinder ist das Parallenaxiom erfüllt, auf der Sphäre hingegen nicht. Ein anderer Test, um herauszufinden, ob es sich um eine euklidische Metrik handelt oder nicht, ist der Paralleltransport eines Vektors entlang eines geschlossenen Weges.

Der Vektor bleibt dabei immer tangential zur Oberfläche:

http://sites.google.com/site/futurephysics/Home/paralleltransport.jpg

Auf einer Kugeloberfläche gehen wir bspw. vom Nordpol entlang des Nullmeridians bis zum Aequator, wandern dort eine zeitlang in östlicher Richtung, um schliesslich auf einem beliebigen Meridian zurück zum Nordpol zu gelangen; dabei entsteht eine Drehung des Vektors. Die Sphäre ist somit nichteuklidisch. Misst man zudem die Winkelsumme eines Dreiecks, wird schnell klar, dass es sich um eine positiv gekrümmte Fläche handeln muss.

Wenn du den Paralleltransport auf einem Zylinder durchführst, wirst du verstehen, worin der Unterschied liegt. ;)

In medias res:

Du solltest dich zuerst mit der Gaußschen Flächentheorie vertraut machen, bevor du dich an höherdimensionale Mannigfaltigkeiten heranwagst. Das würde dir möglicherweise einige Sackgassen ersparen.

Gr. zg

SCR
08.11.09, 22:12
Hallo zg,
Keine Bange, dermassen empfindlich bin ich nun auch wieder nicht.
Das freut mich.
Es steht außer Frage dass der ZylinderMANTEL euklidisch ist.

möbius
08.11.09, 22:41
Hallo Möbius,

ein armer nichts-wissender Dr. phil.! :D

Sei nicht so bescheiden. Wenn alle so bescheiden wären wie du, hätten wir Moderatoren
weniger Probleme... :rolleyes:

M.f.G. Eugen Bauhof

Hallo Eugen Bauhof!

Ernsthaftes Philosophieren führt nun mal zur Bescheidenheit...:D - aber ernsthaftes Physikalisieren auch!
In einem Brief vom 15. März 1922 schrieb Meister EINSTEIN seinem Freund Paul EHRENFEST:
"Wie armselig steht der theoretische Physiker vor der Natur und vor - seinen Studenten!"

Gruß, möbius

EMI
09.11.09, 01:37
Es steht außer Frage dass der ZylinderMANTEL euklidisch ist.
Die ganze Zylinderoberfläche SCR!

Man schneide die Oberfläche eines Körpers auf und "walzt" diese glatt.

1. Reisst diese dabei ein, war sie positiv gekrümmt. (z.B. Kugel)
2. Knittert sie dabei, war sie negativ gekrümmt. (z.B. Sattel)
3. Bleibt sie glatt, war sie ungekrümmt. (z.B. Zylinder)

Der aufgeschnittene und "gewalzte" Zylinder ergibt ein langes Quadrat:) mit recht und links je ein Kreis (rundes Quadtat):D dran.

Gruß EMI

SCR
09.11.09, 06:46
mit recht und links je ein Kreis (rundes Quadtat)dran.
-> 2x :D

Der/Die Kreis(e) bewirken aber eine positive Gesamtkrümmung eines Zylinders - Guckst Du:
http://img4.imageshack.us/img4/9397/zylinder2p.jpg
-> :confused:
@EMI: Du darfst die Kreise nicht einfach links und rechts neben die Mantelfläche legen - Sonst würdest Du ja oben und unten einreissen und die Kreise hingen nur noch an einem Punkt am Mantel (siehe mittleres Bild).
Sollen die Kreise rissfrei berücksichtigt werden kommt man nicht umhin zu "kneten" (unteres Bild).
Laut der Topologie von SCR ist nur eine leere Klopapier-Rolle ungekrümmt.
Selbstverständlich alles IMHO - Widerspricht ja auch fast jedem Lehrbuch ;).

Ich besitze auch kein Mathematik- oder Physik-Diplom - und bin nur ein armer nichts-wissender Dr. phil.! Aber auch damit kann man(n) durchs Leben kommen...
Ja - Und das trifft sich gut: Ich bin AUCH Taxi-Fahrer ... http://www.topfield-europe.com/forum/images/smilies/und_weg.gif ;)

zeitgenosse
09.11.09, 06:55
Es steht außer Frage dass der ZylinderMANTEL euklidisch ist.

Der ganze Zylinder, notabene. Parallelen bleiben Parallelen, egal wo sie gezogen werden. Das Innere eines massiven Körpers, das Zylindervolumen in diesem speziellen Fall, ist dazu nicht massgebend. Etwas anders sähe es bei einem Hohlzylinder aus, wo ich zudem die Innenfläche berücksichtigen müsste. Einem 'Beltramischen Flächenwesen' mit elastischen Körperteilen wäre dies zwar weitestgehend egal. Es würde sich zwar wundern, dass eine Wanderung in seiner Welt zuweilen einen unliebsamen Knick beinhaltet; doch ob es daraus die richtigen Schlüsse ziehen könnte, bleibt eine offene Frage. Seine Welt ist nun einmal die Fläche. Ungeachtet dessen bliebe die Gesamtfläche aus bekannten Gründen euklidisch.

Krümmungen misst man entlang einer geschlossenen Fläche (Kugel, Torus etc.) oder auch bei Flächen mit Rand (näherungsweise ein Reitsattel oder eine Zeltbespannung). Bei der uns vertrauten Kugel ist die massgebende Entität die Sphäre. Gewissermassen die Haut des Körpers. Das nächsthöhere Analogon ist die Hypersphäre (siehe dazu die Poincaré-Vermutung). Eine 'dreidimensionale Haut' sozusagen, in der wir uns wie im gewohnten 3-Raum bewegen können.

Insgesamt geht es bei der Krümmungsfrage auch um Physik. Auf der Erde bewegen wir uns für gewöhnlich auf ihrer Oberfläche. Gingen wir ins Innere, wäre überhaupt keine Krümmung definierbar. Adäquates gilt im Kosmos. Die in Frage kommenden Geometrien betreffen die 'Oberfläche der Welt'. Ihr Inneres ist uns prinzipiell verschlossen. Sollten wir uns tatsächlich in einer Hypersphäre befinden - wie z.B. Hawkings lehrt -, bewegten wir uns nur in der 'Umrandung einer 4-Kugel', nie aber in deren Innerem. Die 'Unterwelt' selbst könnten wir nicht erforschen. Damit muss man sich abfinden können.

In summe habe ich somit genug zum Thema gesagt, um mich allmählich auszuklinken.

Gr. zg

SCR
09.11.09, 07:54
Hallo zg,
Der ganze Zylinder, notabene. Parallelen bleiben Parallelen, egal wo sie gezogen werden. [...] In summe habe ich somit genug zum Thema gesagt, um mich allmählich auszuklinken.
Wenn dem so sein sollte: Dann kannst Du mir aber zuvor sicher noch abschließend erklären was ich bei der Zeichnung in meinem letzten Beitrag falsch gemacht habe -
Ich wäre Dir sehr zu Dank verpflichtet!

Ich habe hier einmal einige exemplarische Links zusammengestellt:
Als einfaches Beispiel kann man sich ein Quadrat vorstellen. Klebt man die beiden vertikalen Kanten zusammen, entsteht ein Zylinder. Werden nun obere und untere Kante noch verklebt, so entsteht ein Torus, d.h. der Zylinder geht in einen Torus über.
Wie aus der Beschreibung ersichtlich ist ausschließlich der ZylinderMANTEL Gegenstand der Betrachtungen - Er wird fälschlicherweise als Zylinder bezeichnet.

Uni Hannover: Seite 13 - Bezeichnung ebenfalls "unsauber" als Zylinder, es wurde aber eine korrekte grafische Darstellung verwendet (http://www.stud.uni-hannover.de/~fmodler/Doughnuts%20und%20Mathematik.pdf)
Kugel, Ellipsoid, Zylinder und Quader gehören derselben topologischen Klasse an, weil man sie durch Drücken und Verbiegen ineinander überführen kann.

zeitgenosse
09.11.09, 08:11
Wie aus der Beschreibung ersichtlich ist ausschließlich der Zylinder-MANTEL Gegenstand der Betrachtungen - Er wird fälschlicherweise als Zylinder bezeichnet.

Es geht im Kontext (Krümmungsfrage) eben nur um die Oberfläche!

Im vorliegenden Beispiel handelt es sich simpel gesagt um einen Hohlzylinder ohne Boden und Deckel. Am Kern der betreffenden Aussage verändert sich dadurch nichts.

Gr. zg

SCR
09.11.09, 08:52
Hallo zg,

jetzt verwirrst Du mich: Worin besteht der konkrete Unterschied zwischen einem Hohlzylinder ohne Boden und Deckel und einem Zylindermantel?
Und auf welches Beispiel bezieht sich Deine letzte Aussage - auf "meines", auf "Deines" oder auf den Link?
Und wie lautet die Kernaussage - Dass der Zylindermantel ungekrümmt ist? Darüber brauchen wir nicht zu disuktieren.
:confused:

Deshalb frage ich am Besten wohl noch einmal ganz simpel: Welche Krümmung weißt denn jetzt ein Vollzylinder auf?
Wobei Vollzylinder = Unten und oben eine Kreisfläche, beide mit einem Zylindermantel verbunden und - damit es keine Missverständnisse bezüglich der Oberfläche gibt - vollständig gefüllt: Eben ein Batzen der über eine Wurst / ein Brot zu einem Vollzylinder geformt wurde.
Das war schließlich der Ausgangspunkt der Diskussion.

Und darauf gibt es nur drei Antwortmöglichkeiten:
a) ungekrümmt
b) positiv gekrümmt
c) negativ gekrümmt

c) wird wohl ausscheiden -> Also a) oder b), zg? :rolleyes:

zeitgenosse
09.11.09, 09:28
Deshalb frage ich am Besten wohl noch einmal ganz simpel: Welche Krümmung weißt denn jetzt ein Vollzylinder auf?

Ob Vollzylinder oder Hohlzylinder ändert nichts an dessen 'innerer' Krümmung. Diese wird im vorliegenden Beispiel anhand der Mantelfläche bestimmt; demzufolge ist K = 0. Allenfalls vorliegende Stirnflächen sind bereits euklidisch flach und müssen daher nicht gesondert berücksichtigt werden.

Gr. zg

SCR
09.11.09, 09:57
Hallo zg,
Also lautet Deine Antwort a): "Ein Vollzylinder (mit Vollzylinder-Oberfläche = 2*Pi*r *h + 2*Pi*r²) ist tutto completto ungekrümmt"
- Oder was? :confused:

Nur zur Erinnerung - Es geht hier schließlich um die Wurst bezüglich der Batzen-Topologie ;):
In Bezug auf die Metrik ist es allerdings so, dass - wenn die Wurst wie ein Zylinder aussähe -, in diesem Fall eine euklidische Geometrie existent wäre. Denn Zylinderoberflächen lassen sich besonders einfach auf einer Ebene abrollen und umgekehrt aus einem Rechteck herstellen. Sie erben damit die euklidische Metrik und besitzen demzufolge die Krümmung k = 0.

Bauhof
09.11.09, 09:59
Ob Vollzylinder oder Hohlzylinder ändert nichts an dessen 'innerer' Krümmung. Diese wird im vorliegenden Beispiel anhand der Mantelfläche bestimmt; demzufolge ist K = 0. Allenfalls vorliegende Stirnflächen sind bereits euklidisch flach und müssen daher nicht gesondert berücksichtigt werden. Gr. zg
Hallo zeitgenosse,

volle Zustimmung.
Vielleich noch folgendes zum allgemeinen Verständnis: Die äußere Krümmung eines Raumes kann nur mit Hilfe eines höherdimensionalen Raumes bestimmt werden, in dem der betrachtete Raum 'eingebettet' ist.

Den Unterschied zwischen innerer und äußerer Krümmung kann man sich anhand dieses Beitrags

http://de.wikipedia.org/wiki/Raumkr%C3%BCmmung#Innere_und_.C3.A4u.C3.9Fere_Kr.C 3.BCmmung

klarmachen. Zur Bestimmung der inneren Krümmung benötigt man keinen einbettenden höherdimensionalen Raum.

M.f.G. Eugen Bauhof

SCR
09.11.09, 10:45
Sagt einmal - man sollte glaube ich "Werken" als Pflichtfach im Physikunterricht vorschreiben.

Wie rollt Ihr denn einen Vollzylinder (inkl. Boden und Deckel) aus Papier in eine Ebene ab?

Ihr macht einen Schnitt längs der Mantelfläche (sozusagen "von oben nach unten aufschlitzen") - O.K. Damit kann man den Zylinder aber noch lange nicht abrollen.
Hierzu muß man zuerst noch die Kreisflächen "bearbeiten":
a) Entweder "abschneiden / einen Verbindungs-Punkt stehen lassen" oder
b) - und das macht die positive Krümmung besonders deutlich - man schneidet die beiden Kreisflächen in Tortenstücke (Je mehr Stückchen das sind, umso geringer wölbt sich dann am Ende noch was beim flachen Hinlegen nach oben).
Was liegt im Fall b) zum Schluß dann vor einem? Ein Rechteck mit Sägezähnen an beiden Seiten. Und Sägezähne "mit Luft dazwischen" sind nun einmal gleichbedeutend mit positiver Krümmung - Wie bei einer Apfelsine die man schält.

Bin ich denn hier allein im Wald? ;) :confused:

Bauhof
09.11.09, 10:51
Bin ich denn hier allein im Wald? ;) :confused:
Hallo SCR,

ja, das scheint so.
Verinnerliche mal die innere Krümung, die der Zeitgenosse nun schon oft erwähnt hat.

M.f.G. Eugen Bauhof

zeitgenosse
09.11.09, 11:00
Bin ich denn hier allein im Wald?

Wenn du dich absichtlich dumm stellst, verzichte ich fortan auf weitere Erklärungshilfen.

Gr. zg

zeitgenosse
09.11.09, 11:08
Zur Bestimmung der inneren Krümmung benötigt man keinen einbettenden höherdimensionalen Raum.

Ja, das ist Bestandteil des 'Theorema egregium' (= herausragender Satz) aus der Gaußschen Flächentheorie.

Für weitere Mitleser:

Präzise ausgedrückt lässt sich die Gausskrümmung durch die metrischen Koeffizienten g_ik und ihre Ableitungen bestimmen, d.h. die Krümmung ist nur von der inneren Geometrie des betreffenden Körpers und nicht vom umgebenden Raum abhängig.

Die Gaußsche Krümmung ist das Produkt der beiden extremalen Normalschnittkrümmungen. Man spricht auch von der inneren Krümmung, der totalen Krümmung oder einfach nur der Krümmung.

Ein paar Beispiele:

Die Einheitskugel (R = 1) hat die konstante Krümmung 1. Eine beliebige Kugel hat die Krümmung 1/r². Ein Zylinder oder ein Kegel haben die Krümmung 0 (deswegen, weil die Normalschnittkrümmung - in Richtung der Mantellinien - Null ist). Beide Körper sind daher längentreu in der Ebene abbildbar (was durch einfaches Abrollen demonstriert wird).

Das 'Theorema egregium' spricht auch davon, dass die Gaußsche Krümmung (also die innere Krümmung) bei isometrischen Abbildungen unverändert bleibt, auch dann, wenn eine Verbiegung nötig ist.

Ausgezeichnete Aspekte bezüglich dieser Thematik finden sich übrigens auch im Lehrbuch 'Anschauliche Geometrie' von Hilbert, Alexandroff und Cohn-Vossen.

In der ART - um eine Brücke zur Physik zu schlagen - impliziert das Nichtverschwinden des Krümmungstensors einen Raum mit intrinsischer Krümmung (bei vorauszusetzender Riemannscher Geometrie eine positive).

Gr. zg

Jogi
09.11.09, 11:13
Hi SCR.



Wie rollt Ihr denn einen Vollzylinder (inkl. Boden und Deckel) aus Papier in eine Ebene ab?

Genau so wie du es hier (Mitte) gemacht hast:
http://img4.imageshack.us/img4/9397/zylinder2p.jpg
Und da wird auch deutlich, daß alle Winkel erhalten bleiben, Parallelen bleiben parallel, die Winkelsumme im Dreieck bleibt 180°, die Geometrie ist euklidisch.
In der unteren Graphik hast du die Quadratur des Kreises versucht, was unnötig ist.


Ihr macht einen Schnitt längs der Mantelfläche (sozusagen "von oben nach unten aufschlitzen") - O.K. Damit kann man den Zylinder aber noch lange nicht abrollen.
Hierzu muß man zuerst noch die Kreisflächen "bearbeiten":
a) Entweder "abschneiden / einen Verbindungs-Punkt stehen lassen"
Genau.
Es ist kein Problem, dass sich die Flächen nur an einem Punkt berühren, man kann diesen Punkt durch Rollen der Kreisfläche entlang des Rechtecks beliebig verschieben, alle Linien, die vorher durch den jeweiligen Berührungspunkt liefen, tun dies auch nacher noch und zwar winkeltreu!
Hierbei wird nur die (euklidische) 2D-Oberfläche des Zylinders berücksichtigt, die Anordnung der Flächen im 3D-Raum spielt keine Rolle.


Gruß Jogi

SCR
09.11.09, 11:38
Hallo zg,
Wenn du dich absichtlich dumm stellst, verzichte ich fortan auf weitere Erklärungshilfen.
Nur zum Verständnis: Wenn dann stelle ich mich nicht dumm sondern bin es.

Hätte vor diesem Hintergrund vielleicht noch jemand die Güte den von mir geschilderten "Fall b)" des Zylinderabwickelns "krümmungsfrei" zu erklären?
Schließlich spricht nichts gegen dieses Vorgehen des Abwickelns - Oder?
Und dabei insbesondere: Wo liegt dabei der Unterschied zur "abgerollten Apfelsine"?
Vielleicht verstehe ich es ja dann: Vielen Dank!

Jogi
09.11.09, 11:41
Um auf die ursprüngliche Fragestellung (den Threadtitel) zurückzukommen:

Die Frage ist so alt wie Euklids "Elemente", wo sie als "Fünftes Postulat" erscheint.
Es besagt, daß sich zwei Geraden, die eine dritte unter Einschluß einer Winkelsumme <180° (auf einer Seite) schneiden, sich in endlicher Entfernung auf dieser Seite auch gegenseitig schneiden.
Euklid konnte es nicht beweisen, und durch die Jahrhunderte scheiterten auch die genialsten Köpfe an dem Beweis.
Von Anfang an gab es die Vermutung, daß es Geometrien gibt, in denen das fünfte Postulat nicht gilt, aber auch das konnte niemand beweisen.
Ich meine, mich erinnern zu können daß das fünfte Postulat ein zentrales Problem der Poincare`schen Vermutung ist, und erst Grischa (Perelman) die Lösung gelang. (Man möge mich korrigieren, wenn ich hier falsch liege.)


Gruß Jogi

Jogi
09.11.09, 11:57
Hi SCR.



Hätte vor diesem Hintergrund vielleicht noch jemand die Güte den von mir geschilderten "Fall b)" des Zylinderabwickelns "krümmungsfrei" zu erklären?
Wie gesagt, dein "Fall b)" ist unnötig und geht an der Fragestellung vorbei, geradewegs in die Hose.
Es ist doch offensichtlich, dass da Geraden nicht gerade bleiben, Winkel verändert werden, etc.


Schließlich spricht nichts gegen dieses Vorgehen des Abwickelns - Oder?
Doch, siehe oben.


Wo liegt dabei der Unterschied zur "abgerollten Apfelsine"?
Die Oberfläche der Apfelsine ist sphärisch in den 3D Raum gekrümmt, die Oberfläche des Zylinders nicht.
Das Merkmal hierfür ist die Winkeltreue, die bei der 2D-Darstellung einer Sphäre nur durch ein spezielles Verfahren (Mercator-Projektion) erhalten bleibt.
Der Zylindermantel weist keine sphärische Krümmung auf, deshalb kann man ihn einfach flach auslegen, und alle Proportionen darauf bleiben erhalten.


Gruß Jogi

zeitgenosse
09.11.09, 12:30
Von Anfang an gab es die Vermutung, daß es Geometrien gibt, in denen das fünfte Postulat nicht gilt, aber auch das konnte niemand beweisen.

Bolyai und unabhängig Lobatschewski haben die hyperbolische Geometrie begründet und damit das Parallelenpostulat auf die euklidische Geometrie verwiesen.

http://www-history.mcs.st-andrews.ac.uk/Biographies/Bolyai.html

http://www-history.mcs.st-andrews.ac.uk/Biographies/Lobachevsky.html

Gauß wusste bereits zuvor davon, publizierte aber nicht, weil er das 'Geschrei der Böotier' fürchtete.

Riemann hat die Gaußsche Flächentheorie auf beliebige Mannigfaltigkeiten erweitert und auch die nach ihm benannte elliptische Geometrie vorangetrieben, wo das 5-Postulat bekanntlich auch nicht gilt.

Die grundlegende Arbeit war folglich bereits im 19. Jh. getan.

Wenn es dich interessiert siehe unter:

- Wußing: 6000 Jahre Mathematik, Bd. 2 (Springer)

- Scriba, Schreiber: 5000 Jahre Geometrie (Springer)

Gr. zg

möbius
09.11.09, 13:38
Ich aber nicht...:D
Mögen Deine Fahrgäste immer nett und großzügig sein...;)
Gruß, möbius

zeitgenosse
09.11.09, 13:59
Ich bin AUCH Taxi-Fahrer ...

Ich dachte eher an einen Bäcker (wegen dem Kneten und den Batzen).

Gr. zg

Jogi
09.11.09, 14:49
Bolyai und unabhängig Lobatschewski haben die hyperbolische Geometrie begründet und damit das Parallelenpostulat auf die euklidische Geometrie verwiesen.
Aah, ja, danke für den Hinweis.
Jetzt weiß ich auch wieder, was mich an der Geschichte erstaunte:
Wieso formulierte man nicht schon viel früher eine hyperbolische Geometrie?
Hyperbolische Krümmungen sind ja nun keine abstrakten Kopfgeburten, es gibt sie in unserer Umwelt zu Hauf.

Gruß Jogi

zeitgenosse
09.11.09, 15:58
Wieso formulierte man nicht schon viel früher eine hyperbolische Geometrie?

Die Zeit dafür war wohl nicht früher reif. Während Jahrhunderten beherrschten Platons gerade Linien und Aristoteles Kreisfiguren die Köpfe der Geometer.

Dazu kam, dass die uns vertraute Welt dem Menschen beständig die Gültigkeit der euklidischen Geometrie vor Augen führte. Mit dem Lot erstellte senkrechte Wände, mit der Libelle ausgerichtete Böden, rechte Winkel und der Satz des Pythagoras bekräftigten immerzu Euklids Postulate. Die euklidische Geometrie war dermassen tief im menschlichen Bewusstsein verankert, dass es einzelne Geometer - welche die universelle Gültigkeit des Parallelenaxioms anzweifelten - schwer hatten, damit ihre Zeitgenossen zu überzeugen. Vor Bolyai bspw. drang bereits Lambert nahe an die hyperbolische Geometrie heran. Gauß selbst war ohnehin bereits im Besitz dieses Wissens um die 'anti-euklidische Geometrie'.

Aber warum der Durchbruch erst im 19. Jh. gelang, ist letztlich nicht eindeutig zu beantworten.

Nachdem dann Riemann die Fundamente der Geometrie in einem umfassenderen Sinne als je zuvor neu ausrichtete, ging es leidlich rasch voran. Beltrami, Klein, Poincaré und Hilbert blieben nicht untätig. Es entstanden die bekannten hyperbolischen Ebenenmodelle sowie die ersten Bezüge zur Physik. Namentlich seien Helmholtz erwähnt und auch Clifford. Als durch Levi-Civita und Ricci auch die Differentialgeometrie ihren mächtigen Calculus erhielt, war der Weg frei für Einsteins Vorstoss in die von Tensoren geprägte gekrümmte Raumzeit mit ihrer Pseudo-Riemannschen Geometrie.

Lehrreich für den historischen Abriss ist z.B.:

- Pfeiffer, Dahan-Dalmedico: Wege und Irrwege - Eine Geschichte der Mathematik (Birkhäuser)

- Trudeau: Die geometrische Revolution (Birkhäuser)

Vermutlich muss man sich diese Titel über eine Tauschbörse beschaffen, weil sie vergriffen sind.

Wer sich umfassender in die Historiographie der Wissenschaften einlesen möchte, greife zu:

- Wußing: Die Grobe Erneuerung - Zur Geschichte der Wissenschaftlichen Revolution (Birkhäuser)

- Russo: Die vergessene Revolution oder die Wiedergeburt des antiken Wissens (Springer)

p.s.
Ich nenne solche Bücher - als nützliche Quellen des Wissens - ab und zu deswegen, damit sich der am Thema Interessierte ein eigenes Bild von der Geschichte machen kann. Denn ohne Fleiss kein Preis!

Gr. zg

SCR
09.11.09, 22:17
Ich dachte eher an einen Bäcker (wegen dem Kneten und den Batzen).
:D Jetzt ohne Flacks: Meine Nussecken waren früher einmal sehr begehrt.
Aber ich muß leider enttäuschen: Da ich sowieso nix richtig kann bin ich weder Bäcker noch Taxifahrer (War nur des böswilligen Scherzes zuliebe - Ich hoffe, möbius verzeiht mir ;)).

Zum Thema:
Also ein Batzen muß wenigstens eine Wölbung (im Sinne einer Kuppel oder eine entsprechend nach innen gerichtete Delle) aufweisen damit er positiv gekrümmt ist. Lasst Ihr das wenigstens durchgehen? :rolleyes:

Jogi
10.11.09, 00:41
:D Jetzt ohne Flacks: Meine Nussecken waren früher einmal sehr begehrt.
Aha, dann bist du also Lotti Köhler, die Mutter des Meisters (http://www.guildo-horn.com/pages/guildos_ecke.php?#ge3)?:D



Also ein Batzen muß wenigstens eine Wölbung (im Sinne einer Kuppel oder eine entsprechend nach innen gerichtete Delle) aufweisen damit er positiv gekrümmt ist.
Lies doch wenigstens mal die Wiki-Einträge zu positiven und negativen, inneren und äußeren Krümmungen.
Wenn dein Batzen stellenweise nach außen (sphärisch) und an anderen Stellen nach innen (hyperbolisch) gedellt ist, ist seine Oberfläche mal positiv, mal negativ gekrümmt.
An anderen Stellen kann die Delle auch wie eine Halfpipe aussehen, dann hat sie dort keine innere Krümmung.

Deine Ursprungsfrage zielte doch auf die Krümmung(en) des Universums ab, oder?
Die Frage nach der globalen Krümmung des Universums ist nach wie vor nicht geklärt.
Die Tatsache daß das Parallelenpostulat im hyperbolischen Universum keine Gültigkeit hat, würde für deine beiden Raumschiffe bedeuten, daß sie im hyperbolisch, also negativ gekrümmten Universum nicht kollidieren werden (und im euklidisch Flachen auch nicht).
Nur das sphärische, also positiv gekrümmte Universum lässt die Raumschiffe in endlicher Zeit kollidieren, vorausgesetzt die Sphäre ist randlos.
Ansonsten kann es einem oder beiden Raumschiffen passieren, daß sie vor der Kollision über den Rand hinausschießen, und was dann passiert, steht nicht einmal in den Sternen...


Gruß Jogi

PS:
Kleine Aufgabe:
Der Batzen unten rechts (http://www.heinlenews.de/muenzverschie.htm#batzen), weist der eine innere oder äußere Krümmung auf?

SCR
10.11.09, 06:59
Aha, dann bist du also Lotti Köhler, die Mutter des Meisters (http://www.guildo-horn.com/pages/guildos_ecke.php?#ge3)?:D
Nein: Aber Lotti hat das Rezept von mir :p
Lies doch wenigstens mal die Wiki-Einträge zu positiven und negativen, inneren und äußeren Krümmungen.
Ich habe neben wikipedia jetzt schon Etliches gelesen - und/aber in wikipedia steht z.B. bei der Gaußschen Krümmung ...
In einem beliebigen Punkt auf der gekrümmten Fläche eines geraden Kreiszylinders ist die gaußsche Krümmung gleich 0.
... was mir wenig bezüglich des Verständnisses Eurer Argumentation weiterhilft (da es ja eher meine Ansicht untermauert).

Ich nehme eben z.B. eine Kugel, ziehe sie am Äquator auseinander (so bilde ich den Zylindermantel) und delle / ebne die Halbkugeln an beiden Enden ein - Fertig ist ein zerreissfrei hergestellter Zylinder: Ein Zylinder ist in meinen Augen dementsprechend nichts anderes wie eine "gerichtete" Kugel mit einem "breiten Äquator", seine Kreismittelpunkte an beiden Enden bilden weiterhin die (Kugel-)Pole. Naja, diese verformte Kugel hat ansonsten "ein paar Kanten" - Aber die stören nicht weiter.
Starten nun nämlich zwei Ameisen vom Äquator aus in Richtung Zylinder-Nordpol, verläuft ihr Weg zunächst parallel (da der "breite Äquotor" = Zylindermantel ungekrümmt ist). Kommen sie über die Kante aber auf den Kreis ändert sich das schlagartig: Sie bewegen sich immer mehr aufeinander zu und treffen sich am Kreismittelpunkt (= Pol).
Insgesamt ist da in meinen Augen die positive Krümmung der Ausgangskugel dabei nicht verschwunden - Sie hat sich lediglich an den Kreisflächen des Zylinders konzentriert.

Bei einer Kugeloberfläche ist das anders. Man denke dabei nur einmal an die Winkelsumme im Dreieck oder an Parallelen. Auf einem Zylinder ist das Parallenaxiom erfüllt, auf der Sphäre hingegen nicht. Ein anderer Test, um herauszufinden, ob es sich um eine euklidische Metrik handelt oder nicht, ist der Paralleltransport eines Vektors entlang eines geschlossenen Weges.
Der Vektor bleibt dabei immer tangential zur Oberfläche:
http://sites.google.com/site/futurephysics/Home/paralleltransport.jpg
Auf einer Kugeloberfläche gehen wir bspw. vom Nordpol entlang des Nullmeridians bis zum Aequator, wandern dort eine zeitlang in östlicher Richtung, um schliesslich auf einem beliebigen Meridian zurück zum Nordpol zu gelangen; dabei entsteht eine Drehung des Vektors. Die Sphäre ist somit nichteuklidisch. Misst man zudem die Winkelsumme eines Dreiecks, wird schnell klar, dass es sich um eine positiv gekrümmte Fläche handeln muss.
Wenn du den Paralleltransport auf einem Zylinder durchführst, wirst du verstehen, worin der Unterschied liegt. ;)
Auch hier gleiches Spiel:
Ich gehe von einem Pol/Kreismittelpunkt los, sobald ich ein Stückchen auf dem Zylindermantel (= "breiter Äquator") gelaufen bin wende ich mich 90° nach links. Nach einem viertel des Zylinder-Umfangs wende ich mich abermals um 90° nach links.
Irgendwann treffe ich wieder auf meinen Ausgangspunkt, dem Kreismittelpunkt.
Das ist identisch mit dem Beispiel an einer Kugel: Die Winkelsumme dieses beschrittenen Dreiecks beträgt auch beim Zylinder 270°.
Und wikipedia schreibt ja auch zur inneren Krümmung:
Die innere Krümmung lässt sich anhand der Geometrie im gekrümmten Raum selbst feststellen. Beispielsweise können Dreiecke auf der Kugeloberfläche eine Innenwinkelsumme von mehr als 180 Grad (bis zu 540 Grad) haben [...]
Womit ein Zylinder als positiv gekrümmt gelten müsste.

PS: Kleine Aufgabe:
Der Batzen unten rechts (http://www.heinlenews.de/muenzverschie.htm#batzen), weist der eine innere oder äußere Krümmung auf?
Gemäß der Topologie des langen Quadrats ist jeder Batzen ohne Löcher positiv gekrümmt (= innere Krümmung).
Er scheint zudem verbogen -> äußere Krümmung vorhanden.

-> Aber lasst's einfach gut sein: Ich blick's anscheinend nicht und werde es wohl nie blicken.

zeitgenosse
10.11.09, 07:59
Ein Zylinder ist in meinen Augen dementsprechend nichts anderes wie eine 'gerichtete' Kugel mit einem 'breiten Äquator';, seine Kreismittelpunkte bilden die (Kugel-)Pole.

Du musst lernen, um zu unterscheiden!

Ein Zylinder ist topologisch homöomorph zu einer Kugel. Etwas unnatürlich sind nur seine Stirnflächen, weil wir beim Übergang auf den Zylindermantel einen rechtwinkligen Knick vollziehen müssen; doch bezüglich seines Geschlechts ist er mit der Kugel eng verwandt.

Geometrisch sind die Objekte hingegen grundverschieden, weil sie nicht dieselbe Metrik besitzen. Die Kugel ist sphärisch, der Zylinder euklidisch.

Lass' mich offen zu dir sein: Irgendwie ist dein räumliches Vorstellungsvermögen ungenügend entwickelt (nimm es mir nicht übel) - denn:

Die 'Kugelpole' befinden sich doch nicht an den Stirnflächen, sondern an gegenüberliegenden Punkten inmitten der Mantelfläche! Denn gemäss deiner Vorgehensweise sind die Stirnflächen (zumindest entlang der Mittellinie) Teil des Aequators. Somit werden sich die Ameisen auf ihrer parallelen Wanderung zum 'Nordpol' nie begegnen. Nach einer Umrundung werden sie aber wieder den Ausgangspunkt erreichen. Doch selbst dann, wenn sie sich über die Zylinderkante begeben müssten, konvergieren sie nicht; denn Parallelen bleiben auf der gesamten Zylinderoberfläche parallel. Wenn du davon nicht überzeugt bist, spanne meinetwegen Fäden über den Zylinder oder zeichne Linien ein. Danach wirst du mir beipflichten müssen.

Der Rest deines Beitrages ist mehr oder weniger Bull**** (deine Lernresistenz zwingt mich zu solchen Ausdrücken; glücklicherweise setzt das System automatisch *** als Platzhalter ein).

Fazit:

Deine geistige Schwerfälligkeit in dieser Sache hat m.E. mit min. zwei Aspekten zu tun. Erstens fehlen dir gewisse elementare Kenntnisse der Geometrie, zweitens liegt es an deinem Vorstellungsvermögen, das dich ständig in die Irre führt. Für das Zweite kannst du nichts dafür (räumliches Sehen lässt sich zwar bis zu einem gewissen Grad trainieren); doch bezüglich dem ersten Punkt liegt es nun an dir, diese Defizite endlich wett zu machen. Soviel zumindest erwarte ich von dir.

Gr. zg

SCR
10.11.09, 09:42
Du musst lernen, um zu unterscheiden!
Versuche ich.
Lass' mich offen zu dir sein: Irgendwie ist dein räumliches Vorstellungsvermögen ungenügend entwickelt (nimm es mir nicht übel)
Kein Problem.
- denn: Die 'Kugelpole' befinden sich doch nicht an den Stirnflächen, sondern an gegenüberliegenden Punkten inmitten der Mantelfläche!
Nein, dieser Fall auf der Mantelfläche ist doch klar und darüber erübrigt sich jegliche Diskussion. Ist spreche konkret hiervon:
http://img690.imageshack.us/img690/7457/kugelzylinder.jpg
Und da komme ich eben in beiden Fällen bei einem Dreieck "1/4 Umfang Äquator - Nordpol" auf Winkelsummen von 270°:
http://img200.imageshack.us/img200/9130/winkelsumme.jpg
Und bei Innenwinkelsummen > 180° hätte ich doch eine positive Innenkrümmung zu unterstellen.

Jetzt sage mir doch bitte einmal konkret in diesem Beispiel wo hier der Betrachtungsfehler liegt (Ich denke da evtl. an die Kanten - Liegen da vielleicht zwei zusätzliche Innenwinkel von je 135° vor? :rolleyes:) - Danke!

Jogi
10.11.09, 10:00
Ich nehme eben z.B. eine Kugel, ziehe sie am Äquator auseinander (so bilde ich den Zylindermantel) und delle / ebne die Halbkugeln an beiden Enden ein - Fertig ist ein zerreissfrei hergestellter Zylinder
Ich versteh' schon, wie du das meinst, und bezüglich der Topologie ist das auch okay.
Dir muß aber klar werden, daß du durch die Verformung der Flächen deren Metrik veränderst.
Die zwei Ameisen, die auf der Kugel den Äquator jeweils im rechten Winkel verlassen, sind schon im nächsten Moment nicht mehr auf Parallelkurs, eigentlich sind sie es nie, sie bewegen sich von Anfang an auf den Kollisionspunkt, den Pol zu.
Auf sphärischen/hyperbolischen Flächen gibt es keine geraden Parallelen.
Diese Nichtparallelität verschiebst du durch die Verformung auf die Kreisflächen.

Geh' doch mal den umgekehrten Weg, wie zg es beschrieben hat:
Ziehe zwei stets parallele Linien über alle drei Flächen des Zylinders und verforme ihn dann zur Kugel.


Gruß Jogi

SCR
10.11.09, 10:47
Hallo Jogi,
Geh' doch mal den umgekehrten Weg, wie zg es beschrieben hat: Ziehe zwei stets parallele Linien über alle drei Flächen des Zylinders und verforme ihn dann zur Kugel.
Ein Zylinder mit drei Flächen http://www.elvisnachrichten.de/images/smilies/gruebel.gif - Das wird ja immer anspruchsvoller ;).

Aber ich versuche mein Bestes :):
- Linien in Krümmungsrichtung des Zylindermantels schrumpfen auf den Äquator zusammen.
- Linien quer zur Krümmungsrichtung auf dem Zylindermantel schrumpfen auf 0.
- Konzentrische Kreise auf Deckel und Boden des Zylinders bilden später die Breitengrade der Kugel.
- Strahlenförmig vom Mittelpunkt zum Kreisrand gezeichnete Linien ("Tortenstücke" auf Deckel und Boden des Zylinders) bilden die Längengrade der Kugel.

(Ich denke da evtl. an die Kanten - Liegen da vielleicht zwei zusätzliche Innenwinkel von je 135° vor? :rolleyes:) - Danke!
Ich denke Nein: Denn die Ameisen laufen ja über die Kante einfach weiter geradeaus -> Für sie gibt es keinen zusätzlichen Winkel an der Kante.

Aber warum komme ich auf eine Innenwinkelsumme > 180° wenn da Eurer Meinung nach nichts gekrümmt sein soll -
Eure Argumente in allen Ehren, aber das konkret z.B. verstehe ich eben einfach nicht. :rolleyes:

möbius
10.11.09, 11:17
(War nur des böswilligen Scherzes zuliebe - Ich hoffe, möbius verzeiht mir ;)).
...


Da möbius absolut humorlos :D ist, versteht er weder Spaß noch "böswillige Scherze" und verzeiht nichts ...;)
Gruß, möbius

Jogi
10.11.09, 11:36
Hi SCR.



Aber warum komme ich auf eine Innenwinkelsumme > 180° wenn da Eurer Meinung nach nichts gekrümmt sein soll -

Na, vielleicht, weil das, was du da auf den Zylinder draufgemalt hast, gar kein Dreieck ist.
Den Übergang von der Mantel- auf die Deckelfläche kann man jeweils nur für eine Linie durch Projektion wegkonstruieren, die andere weist dann einen Winkel auf.
Erst durch entsprechende Verformung des Zylinders wird das Fünfeck zum Dreieck mit Winkelsumme >180°.


Gruß Jogi

zeitgenosse
10.11.09, 11:56
Nein, dieser Fall auf der Mantelfläche ist doch klar und darüber erübrigt sich jegliche Diskussion.

Wenn du es so verstehst, ist es anders, als ich dachte; denn du schriebst zuvor:

nehme eben z.B. eine Kugel, ziehe sie am Äquator auseinander... Also, wenn du am Aequator ziehst, wird dieser m.E. länger und länger, bis der 'Matzen' reisst.

Wenn du stattdessen schreiben würdest:

ziehe an beiden Polen...Nun, gut, es ist jetzt offenkundig, was du mir sagen wolltest. Dabei begehst du jedoch einen Denkfehler; denn du führst an der Zylinderkante eine willkürliche (!) und durch nichts begründbare Richtungsänderung gegen das Kreiszentrum durch. Doch es geht hier nicht um 'Tortenstücke', wo solches durchaus angebracht wäre. Richtig wären zwei parallele und ebene Schnitte durch den Zylinder. So bliebe auch der Tangentialvektor beim Paralleltransport ungedreht erhalten.

p.s.
Leider kann ich vom betrieblichen Arbeitsplatz aus keine Grafik hochladen, um den Sachverhalt zu verdeutlichen (das System verweigert mir dies).

Gr. zg

SCR
10.11.09, 12:05
Na, vielleicht, weil das, was du da auf den Zylinder draufgemalt hast, gar kein Dreieck ist.
Ja, berechtigter Einwand. :rolleyes:
Aber da frage ich mich gleichzeitig: Ist es denn überhaupt ein Dreieck auf der Kugel?

Denn ich kann ja beide "Dreiecke" vektoriell (= auf Basis der Wege der Ameisen) identisch beschreiben:
Vektor 1: Gehe geradeaus vom Nordpol bis Du den Äquator erreichst
Vektor 2: Wende Dich um 90° nach links und gehe 1/4 des Umfangs auf dem Äquator
Vektor 3: Wende Dich um 90° nach links und gehe immer geradeaus zurück zum Nordpol

Und ich dachte eben, die Sicht der Ameise wäre die Entscheidende: Die merkt dabei nichts von der Oberflächenkrümmung einer Kugel.
Aber auch nichts von einem "Knick" (?) ... Das ist doch nichts anderes als wenn ich ein Blatt Papier auf 90° falte:
Das ist die äußere Krümmung. Die Innere des Blattes bleibt davon unberührt ... :rolleyes:

SCR
10.11.09, 12:11
Wenn du es so verstehst, ist es anders, als ich dachte; denn du schriebst zuvor: Also, wenn du am Aequator ziehst, wird dieser m.E. länger und länger, bis der 'Batzen' reisst.
Upps, ja - Stimmt: Mein Beschreibungsfehler. Du hast das völlig richtig erkannt: An den Polen ziehen damit die Kugel am Äquator auf einen Zylinder gedehnt wird.
Ich bitte vielmals um Entschuldigung :o.
Da möbius absolut humorlos :D ist, versteht er weder Spaß noch "böswillige Scherze" und verzeiht nichts ...;)
Gruß, möbius
Dann ziehe ich hiermit meine Entschuldigung auch offiziell zurück. ;)

Jogi
10.11.09, 14:56
Hi SCR.


Ist es denn überhaupt ein Dreieck auf der Kugel?
Ja.
Mit krummen Seiten, deshalb die Winkelabweichungen.


Denn ich kann ja beide "Dreiecke" vektoriell (= auf Basis der Wege der Ameisen) identisch beschreiben:
Vektor 1: Gehe geradeaus vom Nordpol bis Du den Äquator erreichst
Vektor 2: Wende Dich um 90° nach links und gehe 1/4 des Umfangs auf dem Äquator
Vektor 3: Wende Dich um 90° nach links und gehe immer geradeaus zurück zum Nordpol
Die Ameise ist vielleicht nicht der geeignete Beobachter (weil 3-dimensional).
Ein Beobachter innerhalb einer Metrik kann maximal diesselbe Metrik aufweisen wie sein Lebensraum, eine zusätzliche Dimension erhöbe ihn ja in den Stand eines aussenstehenden Beobachters.
Deshalb kann der Flachweltler nur eine innere Krümmung (anhand der Winkelsumme) feststellen, eine äußere Krümmung bleibt ihm verborgen.
Gelangt der Flachweltler an die Kante, ist für ihn da die Welt zu Ende, er kann nicht um die Ecke, weder gehen noch kucken. (Die Mannigfaltigkeit hat einen Rand.)
Das entspricht dem Problem, das Euklid mit dem Beweis des Parallelenpostulates hatte: Er konnte nicht sehen, ob sich die Linien irgendwo schneiden oder nicht.


Und ich dachte eben, die Sicht der Ameise wäre die Entscheidende: Die merkt dabei nichts von der Oberflächenkrümmung einer Kugel.
Ja, wenn sie selbst auch nur zweidimensional, aber genauso gekrümmt ist wie die Oberfläche.

Aber auch nichts von einem "Knick" (?) ... Das ist doch nichts anderes als wenn ich ein Blatt Papier auf 90° falte:
Das ist die äußere Krümmung....der der Flachweltler nicht mehr folgen kann.


Die Innere des Blattes bleibt davon unberührt ... :rolleyes:
Beim Zylinder kommt noch etwas hinzu:
Die Deckelfläche ist nicht in den 3Raum hineingekrümmt, die Mantelfläche hingegen sehr wohl, zusätzlich zu ihrer rechtwinkligen Anordnung zur Deckelfläche.
Beides, die rechtwinklige Anordnung wie auch die (äußere) Krümmung existieren nur im 3Raum.
Im 2Raum existiert nicht mal die andere Fläche, es gibt sie schlicht nicht.


Gruß Jogi

JoAx
10.11.09, 15:03
Im 2Raum existiert nicht mal die andere Fläche, es gibt sie schlicht nicht.


Jetzt müsste es aber langsam Klick machen.


Gruss, Johann

EMI
10.11.09, 16:29
Deshalb kann der Flachweltler...
So ein Flachweltler ist durch sein Nahrungsaufnahme- und Verdauungstrackt zweigeteilt.
Armer Kerl so ein Flachweltler, der muss immer aufpassen das seine 2. (bessere) Hälfte ihm immer folgt.:D

Gruß EMI

zeitgenosse
10.11.09, 16:48
So ein Flachweltler ist durch sein Nahrungsaufnahme- und Verdauungstrackt zweigeteilt.

Die Flachweltler oder Beltramischen besitzen einen ganz anderen Metabolismus als wir. Sie entnehmen die benötigte Energie direkt den Sonnenstrahlen. Deshalb leben sie auch auf Oberflächen, wo es immer hell und licht ist. Die Unterwelt hingegen gehört den Morlocks.

Gr. zg

Marco Polo
10.11.09, 18:58
Die Unterwelt hingegen gehört den Morlocks.

Im Flachland gibts ne Unterwelt? :rolleyes:

Gruss, Marco Polo

zeitgenosse
10.11.09, 20:07
Im Flachland gibts ne Unterwelt?

Aber sicher, nur, das wissen die Flatlander nicht; denn ihnen ist die dritte Dimension bekanntlich unbekannt.

So leben sie denn glücklich und zufrieden auf der Oberfläche der Weltkugel während tief drinnen in der Erde die Morlocks ihr Unwesen treiben.

Gr. zg

SCR
10.11.09, 21:25
Jetzt müsste es aber langsam Klick machen.
Bei wem? http://satanskruemel.sa.funpic.de/smiley/gruebel.gif Etwa bei MIR? Das ist doch nicht Deine ernsthafte Erwartung, JoAx - oder? ;)

Was wäre im Vergleich dazu zu dieser Form zu sagen (abgerundete Kanten):

http://img690.imageshack.us/img690/2973/zylinderabgerundet.jpg

Jogi
10.11.09, 23:06
Was wäre im Vergleich dazu zu dieser Form zu sagen (abgerundete Kanten):

Eigentlich nix mehr, das artet hier zur Realsatire aus.

Aber weil ich so gutmütig bin:
@SCR:
Es ist schnurzpiepegal, wie stark sich die Krümmung ändert, am ersten Punkt der nicht mehr in der Ebene liegt, scheitert der Plattfüßer.
Wenn er sich der Krümmumg anpassen könnte, wäre er kein Flachländer mehr, sondern dreidimensional.

Es gäbe hierzu noch einiges zu sagen, vor allem bezüglich der Erweiterung dieser Thematik auf den 3- und 4-Raum (und den n-Raum).
Aber erst muß es hier mal Klick machen.
Oder SCR gesteht, daß er uns auf den Arm nimmt.:D


Gruß Jogi

JoAx
10.11.09, 23:09
Bei wem? http://satanskruemel.sa.funpic.de/smiley/gruebel.gif Etwa bei MIR? Das ist doch nicht Deine ernsthafte Erwartung, JoAx - oder? ;)

Was wäre im Vergleich dazu zu dieser Form zu sagen (abgerundete Kanten):

http://img690.imageshack.us/img690/2973/zylinderabgerundet.jpg

Dass das kein Zylinder ist, von dem man redet. Das wäre eine 2Sphäre.

Nicht die Bilder überinterpretieren. Etwas mehr abstrakt denken.


Gruss, Johann

SCR
10.11.09, 23:57
Das wäre eine 2Sphäre.
Ja: Eben noch ein ungekrümmter Zylinder - plötzlich eine positiv gekrümmte 2Sphäre ... Keine fließende Grenze ... Hmmm :rolleyes:
Und grundsätzlich: Wann ist ein Knick eine äußere Krümmung - Wann eine andere Dimension?
Einem 'Beltramischen Flächenwesen' mit elastischen Körperteilen wäre dies zwar weitestgehend egal. Es würde sich zwar wundern, dass eine Wanderung in seiner Welt zuweilen einen unliebsamen Knick beinhaltet; doch ob es daraus die richtigen Schlüsse ziehen könnte, bleibt eine offene Frage.
Ein Beobachter innerhalb einer Metrik kann maximal diesselbe Metrik aufweisen wie sein Lebensraum, eine zusätzliche Dimension erhöbe ihn ja in den Stand eines aussenstehenden Beobachters.
Dabei stellt die Zylinderkante doch alle Punkte dar, die beiden Ebenen zugehören. Hätten zgs Flächenwesen oder vielleicht besser noch ein eindimensionaler "Super-"Flatländer Probleme beim Überwinden der Kante?
Krümmungen misst man entlang einer geschlossenen Fläche (Kugel, Torus etc.) oder auch bei Flächen mit Rand (näherungsweise ein Reitsattel oder eine Zeltbespannung).
Ist ein Zylinder eine geschlossene Fläche oder drei Flächen mit Rand?
Und misst / gibt man für die Kante eigentlich zwei unterschiedliche Gaußsche Krümmungen (an)?

möbius
11.11.09, 00:26
....
So leben sie denn glücklich und zufrieden auf der Oberfläche der Weltkugel während tief drinnen in der Erde die Morlocks ihr Unwesen treiben.

Gr. zg

Sind das die Verwandten vom Max Morlock, dem Mitglied der Fuballweltmeisterschaftsmannschaft von 1954...:confused:

Uli
11.11.09, 00:38
Sind das die Verwandten vom Max Morlock, dem Mitglied der Fuballweltmeisterschaftsmannschaft von 1954...:confused:

Wohl nie H.G. Wells gelesen ("Time Machine") ?
Die Morlocks werden in ferner Zukunft die Unterwelt bewohnen:
http://de.wikipedia.org/wiki/The_Time_Machine_(2002)

Gruß,
Uli

JoAx
11.11.09, 00:59
Ja: Eben noch ein ungekrümmter Zylinder - plötzlich eine positiv gekrümmte 2Sphäre ... Keine fließende Grenze ... Hmmm :rolleyes:

Nichts ist plötzlich, SCR. Wenn man sich gedanken über die Geometrie des Raumes macht, dann interessiert nicht das Modell, dass eine Art der Krümmung bildhaft darstellt - der physisch real herstellbare Zylinder in diesem Fall.

Die Sphäre umschliesst die Kugel komplett, von allen Seiten, deswegen kannst du ihr inneres nicht sehen. Beim Zylinder ist es nicht so. Der Deckel und der Boden von diesem gehöhren zum Inneren, wenn man so will. Der Übergang vom Mantel zu diesen erfolgt nicht stetig (wie von dir gezeichnet), man könnte die Kante zwischen diesen vielleicht sogar als singulär bezeichnen.


Gruss, Johann

JoAx
11.11.09, 01:05
Ist ein Zylinder eine geschlossene Fläche oder drei Flächen mit Rand?


AHA!

Ich freue mich dies sagen zu können ;) - richtige Richtung.


Gruss, Johann

zeitgenosse
11.11.09, 06:36
Wohl nie H.G. Wells gelesen ("Time Machine")? Die Morlocks werden in ferner Zukunft die Unterwelt bewohnen.

http://de.wikipedia.org/wiki/The_Time_Machine_(2002) (http://de.wikipedia.org/wiki/The_Time_Machine_%282002%29)

Gr. zg

SCR
11.11.09, 08:49
Hallo JoAx,
das würde mich jetzt schon interessieren: Was hattest Du denn für eine Note in Werken? - Deine gebastelten Zylinder müssen zwangsläufig beim Abgeben ja immer auseinandergefallen sein.:D
Es gibt eben nur "PHYSIKALISCHE" und keine "MATHEMATISCHEN" Papierblätter zu kaufen.
Ernsthaft: Ist das hier ein rein mathematisches Definitionsproblem oder steckt da mehr dahinter?
Der Deckel und der Boden von diesem gehören zum Inneren, wenn man so will.
Wenn Du an dieser Stelle den Zylindermantel abrollst - Was hast Du dann vor Dir? "Babuschkas"?
Jede einfach zusammenhängende kompakte unberandete 3-dimensionale Mannigfaltigkeit ist homöomorph zur 3-Sphäre.
Poincaré hatte gefragt, ob jede einfach zusammenhängende dreidimensionale Mannigfaltigkeit, die keinen Rand hat und die nicht unendlich ist, eine dreidimensionale Sphäre ist.
Berechtigte Frage - Und das Schöne ist: Nur eine Antwort kann eigentlich richtig sein.:D
Oder muß man eventuell nach "mathematischer" und "physikalischer" Antwort unterscheiden? :rolleyes:

Schließlich kann man aus einer Kugel genauso gut einen Quader formen.
Und dann die Betrachtungsweise, wie ich sie am Zylinder dargestellt habe, auch auf ihn anwenden - Mit dem Ergebnis "positive Krümmung".

"Ein (physikalischer) Quader ist eine eckige Kugel" :rolleyes:

P.S.: Ich bedaure es schon ein wenig dass EMIs langes Quadrat als Ur-Mutter derartiger Bezeichnungen "abhanden gekommen" ist ;).

möbius
11.11.09, 09:11
Wohl nie H.G. Wells gelesen ("Time Machine") ?
Die Morlocks werden in ferner Zukunft die Unterwelt bewohnen:
http://de.wikipedia.org/wiki/The_Time_Machine_(2002)

Gruß,
Uli
Sehr interessant! :D
Und wer wird in "ferner Zukunft" die ERDE bewohnen...:confused:
"Schau' mer mol!";)
Gruß, möbius

SCR
11.11.09, 09:15
Möbianten.

zeitgenosse
11.11.09, 09:28
Möbianten.

SCR-Transformers

Gr. zg

SCR
11.11.09, 09:42
SCR-Transformers
Dann wäre diese Welt aber zu bedauern! :D

JoAx
11.11.09, 10:02
Hallo SCR,


Ernsthaft: Ist das hier ein rein mathematisches Definitionsproblem oder steckt da mehr dahinter?


ich denke, das Problem liegt daran, dass du dich weigerst abstrakt zu denken. Eine matematische Linie ist z.B. eindimensional. Sie hat keine "Dicke". Real, beim Heimwerken oder Zeichnen, ist so etwas aber nicht herstellbar. Dennoch benutzt du die Linie ohne ein Problem damit zu haben (hoffe ich). Warum? Warum hast du ein Problem mit einer 2D Zylinderoberfläche (Dicke=NULL)?


Wenn Du an dieser Stelle den Zylindermantel abrollst - Was hast Du dann vor Dir? "Babuschkas"?


Nein, keine Babuschkas, sondern Matröschkas. ;)
Weisst du wie (woraus) Furniere hergestellt werden?


Berechtigte Frage - Und das Schöne ist: Nur eine Antwort kann eigentlich richtig sein.:D
Oder muß man eventuell nach "mathematischer" und "physikalischer" Antwort unterscheiden? :rolleyes:


Eine 3Sphäre ist die Oberfläche einer vierdimensionalen Kugel. Lass uns vorerst bei der 2Sphäre und 2Zylinderoberfläche bleiben.


Gruss, Johann

SCR
11.11.09, 10:18
Hallo JoAx,
Warum? Warum hast du ein Problem mit einer 2D Zylinderoberfläche (Dicke=NULL)?
Ich habe ein Problem mit den Kanten:
- Ich falte ein Blatt Papier in der Mitte. Die eine Hälfte liegt bündig auf dem Tisch, die andere Hälfte steht jetzt 90° nach oben ab - Dadurch ändert sich nur die äußere und nicht die innere Krümmung des Blattes.
Und - wenn ich da jetzt "tiefer reingehe" - gilt das doch für jeden Punkt dieser Faltkante.
- Was unterscheidet hiervon der Übergang Zylindermantel - Kreisfläche wenn ich mir genau einen Punkt dieser "Rund-Kante" herausgreife?
DAS verstehe ich nicht.
Weisst du wie (woraus) Furniere hergestellt werden?
Jepp:
Ich kann ein Blatt Papier "richtig eng" einrollen, die Blattkanten bilden dann die Stirnflächen - völlig rissfrei.
(nur eben andersherum: "vom Stamm abrollen").
Nein, keine Babuschkas, sondern Matröschkas. ;)
Richtig :D http://de.wikipedia.org/wiki/Matrjoschka - Wie komme ich denn auf Babuschkas? http://www.elvisnachrichten.de/images/smilies/gruebel.gif
EDIT: Aah - Der wiki-Artikel klärt diesbezüglich auf!

Jogi
11.11.09, 10:27
...\\switch to "off topic modus"\\...

Wohl nie H.G. Wells gelesen ("Time Machine") ?
Die Morlocks werden in ferner Zukunft die Unterwelt bewohnen:
http://de.wikipedia.org/wiki/The_Time_Machine_(2002)


Ich kenne nur die Erstverfilmung aus dem Jahr 1960, die mich als jungen Menschen faszinierte.

Zur "modernen" Version schreibt das Lexikon des internationalen Films (Hervorhebung von mir):
„Das Remake erreicht weder die Leichtigkeit des Vorgängerfilms noch die philosophische Tiefe des Romans und beschränkt sich auf vordergründige Spannung und einige Gags.“

"Je weiter sich die Handlung in die Zukunft tastet, desto blödsinniger wird sie. ... Was bei H.G. Wells noch die Nachkommenschaft einer unterirdisch (über)lebenden Arbeiterklasse war, mutiert hier zum Horrorfilmpersonal. Jegliche Gesellschaftskritik und alles, was vielleicht zum Nachdenken anregen könnte, wird aus dieser Neuversion fürs Popcorn-Zeitalter sauber getilgt."
Und somit kann man sich das sparen, wie die meisten Zweit- und Drittverfilmungen.

OT-Modus aus.


Gruß Jogi

JoAx
11.11.09, 11:00
Hallo SCR,


- Ich falte ein Blatt Papier in der Mitte. Die eine Hälfte liegt bündig auf dem Tisch, die andere Hälfte steht jetzt 90° nach oben ab - Dadurch ändert sich nur die äußere und nicht die innere Krümmung des Blattes.
Und - wenn ich da jetzt "tiefer reingehe" - gilt das doch für jeden Punkt dieser Faltkante.
- Was unterscheidet hiervon der Übergang Zylindermantel - Kreisfläche wenn ich mir genau einen Punkt dieser "Rund-Kante" herausgreife?
DAS verstehe ich nicht.


du musst nicht das abgeknickte Papierblatt mit dem Übergang Zylindermantel-Deckel vergleichen, sondern den Verlauf der Zylinderoberfläche in der Querrichtung und den Übergang Zylindermantel-Deckel. Beim Durchschreiten der Zylinderoberfläche in der Querrichtung ändert sich die Orientierung stetig (weich, wenn man so will), im zweiten Fall passiert es abruppt. Es müssen nicht ein Mal 90° oder mehr sein. Der Deckel kann auch wie ein Konus aussehen.

Wenn die Richtungsänderung abruppt erfolgt (= es gibt keine "Zwischenstellungen") dann ist es eine (mathematische) Kante, und diese stellt ein Hinderniss dar (topologisch gesehen).

Gehe von der anderen Seite heran. Falte nicht das Blatt in der Mitte, sondern setzte ein anderes an der Kante des ersten an. Erst dann hast du tatsächlich eine abrupte Richtungsänderung.

IMHO


Gruss, Johann

PS: Stell dir auch ein Rohr mit Wanddicke=NULL vor.

EMI
11.11.09, 11:34
Hallo SCR,
PS: Stell dir auch ein Rohr mit Wanddicke=NULL vor.
Genau SCR,

Du kannst dir das auch leicht selbst herstellen.
Nimm dazu ein Ofenrohr und mach das Blech ab.

Gruß EMI

SCR
11.11.09, 12:23
Nimm dazu ein Ofenrohr und mach das Blech ab.
http://www.elvisnachrichten.de/images/smilies/gruebel.gif - Hmm.
Also hier habe ich das Ofenrohr.
Blechschere. Ich brauche eine Blechschere. - Wo ist die verdammte Blechschere? Ah, hier.
Also los geht's - Mann, geht das schwer.
So, jetzt hab ich's - Nur noch das Blech abnehmen.

Booah - HEUREKA! - Das muss ich Euch zeigen, hier schaut 'mal ...
Mist, jetzt ist es mir das blöde Ding runtergefallen.

Hmm - Wo ist es denn nur hin? Ich muß erst einmal suchen gehen.
Melde mich ERST wieder wenn ich es gefunden habe. ;)

SCR
11.11.09, 12:49
Falte nicht das Blatt in der Mitte, sondern setzte ein anderes an der Kante des ersten an. Erst dann hast du tatsächlich eine abrupte Richtungsänderung.
Das hatte ich auch schon so verstanden: MATHEMATISCH gibt es keine Punkte die gleichzeitig beiden "Dimensionen" angehören (können) - Es fehlt damit sozusagen die Brücke zwischen beiden Flächen damit irgendwelche Flatlander hinübergelangen könnten (Obwohl ich das ehrlicherweise auch noch nie so konkret irgendwo gelesen/gehört habe wie so ein Zylinder mathematisch zusammengesetzt sein soll).

Aber ein solcher Zylinder würde in der PHYSIKALISCHEN Realität doch einfach auseinanderfallen ... Von daher frage ich mich ob hier das mathematische Modell die physikalische Realität tatsächlich korrekt abbildet:
"Es gibt in der Realität/Physik eben doch nur Spheren" ... :rolleyes:

EDIT:
du musst nicht das abgeknickte Papierblatt mit dem Übergang Zylindermantel-Deckel vergleichen, sondern den Verlauf der Zylinderoberfläche in der Querrichtung und den Übergang Zylindermantel-Deckel.
Selbst bei punktueller Betrachtung kämen wir sonst zu Widersprüchen: Ein und derselbe Punkt hätte sonst unterschiedliche Innenkrümmungen - Je nachdem aus welcher "Richtung" wir ihn betrachten (?) :rolleyes:

SCR
11.11.09, 12:56
Melde mich ERST wieder wenn ich es gefunden habe. ;)
Tut mir für Euch leid: War gelogen. :D

EMI
11.11.09, 13:19
MATHEMATISCH gibt es keine Punkte die gleichzeitig beiden "Dimensionen" angehören...
Ein Punkt hat keine Dimension SCR,

auch mathematisch nicht!

Dimension Punkt = 0
Dimension Linie = 1
Dimension Fläche = 2
Dimension Raum = 3
Dimension Raumzeit = 4

In einem Spiegel "sieht" man immer eine Dimension weniger.;)

Gruß EMI

SCR
11.11.09, 13:45
Ein Punkt hat keine Dimension SCR, auch mathematisch nicht!
Ja, habe ich aber auch nicht gesagt - Oder doch? :rolleyes:

Ich versuch's einmal so:
Eine Linie besteht aus "nebeneinanderliegenden Punkten, die miteinander in Beziehung stehen", eine Fläche aus "nebeneinanderliegenden Linien (und damit "ausgezeichnete" Punkte), die miteinander in Beziehung stehen", ...

(Das reicht erst einmal).

Und jetzt lassen wir zwei Flächen Kante auf Kante (oder auch eine Kante auf Fläche) aneinander stoßen.
Und da komme ich wieder auf das gleiche Problem/die gleiche Lösung wie bei unserer "Raumquanten-Abstands-Diskussion" im Jenseits:
Die Punkte sind eindeutig einer Fläche zugeordnet (-> Krümmung eindeutig etc.) - BEZIEHUNGEN zwischen Punkten können aber mehrfach /flächenübergreifend auftreten ... (?)

Aber irgendwie ist das selbst in meinen Augen (und das heißt schon was!) doch "mathematisch bekloppt" - (alternativ meine Wenigkeit) ;)

In einem Spiegel "sieht" man immer eine Dimension weniger.;)
Der ist gut!

EDIT: Anderseits denke ich lässt sich der Sachverhalt doch konkret auf folgende Fragestellung "eindampfen":
Zwei Punkte liegen nebeneinander. Wann gehören sie jetzt der gleichen "Dimension" (= der gleichen Linie, der gleichen Fläche, ...) an und wann nicht?
... Geht IMHO "nur" über Beziehungen ... :rolleyes:

Timm
11.11.09, 14:17
Und wer wird in "ferner Zukunft" die ERDE bewohnen...:confused:


Vielleicht nur noch die Phil_ist_er.

Gruß, Timm

möbius
11.11.09, 16:53
Na besser diese Phil-ist-er als diese Fies-olofen...ääähhhh....Phisolophen...äähhhh....o der wie hiessen die noch mal...:confused: :eek:
Gruß, möbius

möbius
11.11.09, 16:57
Möbianten.

Sind das die neuesten Elementar-Teilchen...:confused:
Und wenn ja, wie vertragen die sich ab heute (11.11.!!! Helau) mit den Komödianten :confused: ...

Und jetzt soll auch noch ein Vogel auf CERN geschissen haben ...:eek:

In allergrößter Sorge...
möbius

SCR
12.11.09, 08:27
Also ich habe noch einmal ein wenig recherchiert (leider erfolglos) und nachgedacht.

Ergebnis:
- Jeder Batzen ohne Loch ist in Summe positiv gekrümmt.
- Jedes Loch ohne Batzen ist in Summe negativ gekrümmt.
- Die Gesamtkrümmung eines Batzens mit Löchern (bzw. von Löchern mit Batzen) ergibt sich aus der Anzahl der Löcher im Batzen.

Lokal kann ein Batzen / ein Loch davon abweichend ein anderes Krümmungsmaß aufweisen.

Euklidik ist eine Vereinfachungsform, die lokal in Näherung überall angewendet werden kann, die es aber in der Natur eigentlich gar nicht gibt.

Ein Batzen = (ab)geschlossene(s) Objekt/Oberfläche/Dimension.
Ein Loch = offene(s) Objekt/Oberfläche/Dimension (= Kein Batzen).

Ergänzend:
Endlich: Das/Die Objekt/Oberfläche/Dimension hat einen Rand
Unendlich: Das/Die Objekt/Oberfläche/Dimension hat keinen Rand

Und zur Klarstellung - Ich meine es tatsächlich so wie ich es schreibe:
Batzen = Kugel, Eiförmig, Wurst, ZYLINDER, QUADER, ...

In Erwartung entsprechender Züchtigung ... http://www.smileyparadies.de/girls/smilie_girl_170.gif

Bauhof
12.11.09, 10:35
In Erwartung entsprechender Züchtigung ...
Hallo SCR,

du scheinst tatsächlich masochistisch veranlagt zu sein. :D
Der Zeitgenosse hat dich doch bereits ─ mit Recht ─ ausreichend 'gezüchtig'. Reicht dir das denn immer noch nicht? :D Mache dir keine Hoffnung, ich greife nicht zur Rute! :D

M.f.G. Eugen Bauhof

SCR
12.11.09, 11:00
Der soll nur kommen: http://www.smileyparadies.de/girls/smilie_girl_224.gif
;)

möbius
12.11.09, 11:15
Aber der NIKOLAUS kommt selten allein...:D (Und zwar "in der Regel" am 5. 12.!)
Und was ist, wenn sein Knecht Ruprecht Dich in den Sack steckt ....:confused: :eek:
Gruß, möbius ;)

Jogi
12.11.09, 11:18
Moin SCR.



- Jeder Batzen ohne Loch ist in Summe positiv gekrümmt.
Du mußt hier unterscheiden:
Der Körper (3D) ist im 3Raum erst mal gar nicht gekrümmt, er hat die gleiche (im Idealfall euklidische) Geometrie wie der Raum.
Seine Oberfläche kann im 3Raum alle möglichen Krümmungen haben, auch negative.
Was du wahrscheinlich meinst:
Man kann aus einer endlichen Menge Knetmasse keinen Körper ohne Loch formen, dessen Oberfläche in Summe eine äußere negative Krümmung im 3Raum aufweist.
Das schließt aber mathematisch eine komplett negative innere Krümmung der Flächen, die den Körper umschließen, nicht aus.



- Jedes Loch ohne Batzen ist in Summe negativ gekrümmt.
Kann ech chein, dach er Chapernack treipt?

- Die Gesamtkrümmung eines Batzens mit Löchern (bzw. von Löchern mit Batzen) ergibt sich aus der Anzahl der Löcher im Batzen.
Wie das?
Spielt die Größe der Löcher keine Rolle?


Lokal kann ein Batzen / ein Loch davon abweichend ein anderes Krümmungsmaß aufweisen.
Eben.

Euklidik ist eine Vereinfachungsform, die lokal in Näherung überall angewendet werden kann, die es aber in der Natur eigentlich gar nicht gibt.
Das gleiche könntest du für jede konstante Krümmung sagen.
Und in letzter Konsequenz könnte ich dir in diesem Punkt sogar zustimmen, aber das gehört nicht hierher...


Ein Batzen = (ab)geschlossene(s) Objekt/Oberfläche/Dimension.
Ein Loch = offene(s) Objekt/Oberfläche/Dimension (= Kein Batzen).
Tut mir leid, damit kann ich gar nichts anfangen.


Ergänzend:
Endlich: Das/Die Objekt/Oberfläche/Dimension hat einen Rand
Das ist definitiv falsch.
Eine Kugeloberfläche ist endlich, aber randlos.


Gruß Jogi

zeitgenosse
12.11.09, 11:27
Jeder Batzen ohne Loch ist in Summe positiv gekrümmt.

Ein Quader besitzt meines Wissens ziemlich flache Flächen. Ein Würfel auch. Wie soll ich die Krümmung einer flachen Welt nach deinem Gusto bestimmen?

Jedes Loch ohne Batzen ist in Summe negativ gekrümmt.Ein Loch ohne gleichzeitig anwesenden Körper macht wenig Sinn. Selbst beim p-dotierten Halbleiter benötigt man ab und zu Elektronen. Eine Welt aus lauter Löchern wäre körperlos.

Gr. zg

SCR
12.11.09, 12:05
Hallo Ihr zwei,
Das ist definitiv falsch. Eine Kugeloberfläche ist endlich, aber randlos.
Ja, ich und meine schludrigen Definitionen - Die brechen mir noch einmal das Genick :D.
Ja, Du hast Recht, ich hatte die Sphären nicht berücksichtigt -> Korrektur
Endlich: Das/Die Objekt/Oberfläche/Dimension hat einen Rand oder ist abgeschlossen
Unendlich: Das/Die Objekt/Oberfläche/Dimension hat keinen Rand und ist offen
Besser? :rolleyes: Deshalb bleibt aber trotzdem jeder Batzen positiv gekrümmt (s.u.). ;)
Ein Quader besitzt meines Wissens ziemlich flache Flächen. Ein Würfel auch. Wie soll ich die Krümmung einer flachen Welt nach deinem Gusto bestimmen?
Das liegt weder an mir noch an Dir sondern ausschließlich an der Winkelsumme:
http://img690.imageshack.us/img690/2976/quadergekrmmt.jpg
(Anmerkung: Das Rote ist "aus Sicht des Quaders" ein DREIeck)
Kann ech chein, dach er Chapernack treipt?
Wer? ICH? Niemals, ich bin doch ein http://www.smileyparadies.de/girls/smilie_girl_025.gif (Ernsthaft: Zumindest in diesem Falle "Nein").
Eine Welt aus lauter Löchern wäre körperlos.
Ja - Was übrig bliebe wäre so etwas wie "leerer Raum". :rolleyes:

JoAx
12.11.09, 12:45
Hallo SCR,


Das liegt weder an mir noch an Dir sondern ausschließlich an der Winkelsumme:
http://img690.imageshack.us/img690/2976/quadergekrmmt.jpg
(Anmerkung: Das Rote ist "aus Sicht des Quaders" ein DREIeck)


so, wie es hier gezeichnet ist, kann es nur ein 3d-Weltler, aus seiner 3d-Perspektive gemacht haben. (Was im Übrigen nicht die Einzige Möglichkeit ist.) Für ein Flattländler (wenn er die Kante - imho = Singularität - überschreiten könnte) wäre jede Richtung auf der "neuen" Oberfläche gleich möglich/wahrscheinlich. Denn die einzige Bedingung für die Richtung des neuen Kurses ist, dass es in der "neuen" Fläche liegt. Darauf hat dich zg. mit dem "Tortenstück" aber schon mal hingewiesen.

Nachtrag:
Mit einer runden Kante wäre es kein Quader.


Gruss, Johann

zeitgenosse
12.11.09, 13:02
Das Rote ist aus Sicht des Quaders ein DREIeck

Blödsinn, du hast ein Siebeneck eingezeichnet!

Ich rate dringlichst zu einem Besuch beim Augenarzt.

Gr. zg

EMI
12.11.09, 13:51
Aber der NIKOLAUS kommt selten allein...:D (Und zwar "in der Regel" am 5. 12.!)
Seltsam:confused: , bei mir kam der solange wie ich denken kann immer am 6.12.
Na ja, Ausnahmen bestätigen die Regel wie man sieht.

Jogi
12.11.09, 14:34
Hallo Ihr zwei,drei,vier...



Ja, Du hast Recht, ich hatte die Sphären nicht berücksichtigt -> Korrektur
Endlich: Das/Die Objekt/Oberfläche/Dimension hat einen Rand oder ist abgeschlossen
Unendlich: Das/Die Objekt/Oberfläche/Dimension hat keinen Rand und ist offen
Besser? :rolleyes: Deshalb bleibt aber trotzdem jeder Batzen positiv gekrümmt (s.u.). ;)
Solange du nicht zwischen dem Batzen und seiner Oberfläche(n) unterscheidest, kannst du über die Krümmung(en) überhaupt keine sinnvollen Aussagen machen.
Und was soll denn eine gekrümmte/geschlossene/offene Dimension sein?
Sowenig der Flachweltler eine äußere Krümmung seiner 2D-Welt feststellen kann, sowenig sind wir als 3D-ler in der Lage, eine evtl. Krümmung unseres Universums (Batzen) im 4Raum wahrzunehmen.
Lediglich eine innere Krümmung kann über die Dreieckswinkelsumme erkannt werden.

Ich ahne schon, wie du's meinst, aber du stehst dir selbst im Weg.

Du denkst imho:
Der Batzen beansprucht Raum (im Raum), also muß er positiv gekrümmt sein.
So einfach ist das aber nicht.

Nochmal zu Räumen und ihren Rändern:
Begib' dich mal ins Innere deines Batzens.
Hier hast du 3 räumliche Freiheitsgrade, entlang derer du dich bewegen kannst, bis du an den Rand (die Oberfläche) stößt. Der Batzen hat also einen Rand.
Jetzt wirst du zum Flachweltler und bewegst dich auf dieser Fläche, die nun dein Raum ist.
Ist der Batzen z. B. eine Kugel, dann hat sein Rand keinen Rand, du kannst ewig geradeaus gehen, kommst aber von Zeit zu Zeit wieder an deinen Ausgangspunkt, was du als Hinweis auf die Endlichkeit deiner Welt interpretieren kannst.

Nochmal zurück ins Innere des Batzens:
Es kann passieren, daß du trotz ewiger Geradeausbewegung den Rand des Batzens nie erreichst, weil sich dieser schneller von dir wegbewegt als du ihm folgen kannst.
Deshalb kannst du in diesem Falle nicht sagen, ob deine Welt einen Rand hat, oder ob sie gar unendlich ist.


Gruß Jogi

zeitgenosse
12.11.09, 16:20
Es kann passieren, daß du trotz ewiger Geradeausbewegung den Rand des Batzens nie erreichst, weil sich dieser schneller von dir wegbewegt als du ihm folgen kannst.

Im Beispiele der Poincaréschen Kreisscheibe ist dies z.B. so. Obwohl es sich im 3-Raum um eine geschlossene Kreisfläche handelt, erreicht ein Flachweltler den Rand dieser seiner Welt nie; denn die Längenskala variiert lt. Konvention mit dem Ort:

l = (R² - r²)/R²
(r = Abstand vom Mittelpunkt)

Dies hat zur Folge, dass der Scheibenrand in unendliche Ferne rückt, je näher ein Wanderer diesem kommen möchte. Für den Flachtweltler ist seine Welt unbegrenzt und randlos.

Was die Bewohner dieser Welt aber tun können ist: Vermessung der Geometrie, indem sie das Verhältnis von Kreisumfang und Durchmesser ermitteln; dabei stellen sie überrascht fest, dass ihrer Welt eine Bolyai-Lobatschewski Geometrie zugrunde liegt.

Gr. zg

möbius
12.11.09, 16:26
Warum soll's beim Kommen des Nikolauses anders sein als in der Physik...solange er nicht zu früh kommt...:D (also etwa schon am 4., 3. oder 2. Dezember!);)
Gruß, möbius

SCR
12.11.09, 22:21
Hallo zusammen,
eigentlich bin ich etwas genervt (nicht wegen Euch): Aber ich habe hier ja "provoziert" - und da habt Ihr auch ein Anrecht auf Antwort.
Vorab: Danke erst einmal für Euer Feedback!
Für ein Flattländler (wenn er die Kante - imho = Singularität - überschreiten könnte) wäre jede Richtung auf der "neuen" Oberfläche gleich möglich/wahrscheinlich.
Zwei angrenzende Seiten eines Quaders bilden eine Kante.
Was trifft nun zu:
a) Der Rand der einen Fläche stößt an den Rand der anderen Fläche. Die Quaderkante wird damit durch zwei Reihen von Punkten gebildet, wobei die eine Reihe eindeutig zur einen, die andere Reihe eindeutig zur anderen Fläche gehört.
b) Die Kante stellt eine Reihe von Punkten dar, die sowohl zur einen als auch zur anderen Fläche gehören.
Ich denke "b)" - Eure Argumentation sagt aber "a)". Nur wären doch dann alle derart zusammengesetzten Objekte nichts anderes als Kartenhäuser ohne Zusammenhalt ... :rolleyes:
Solange du nicht zwischen dem Batzen und seiner Oberfläche(n) unterscheidest, kannst du über die Krümmung(en) überhaupt keine sinnvollen Aussagen machen.
Ja: Ich rede von der Oberfläche von Körpern. Bzw. wollte ich das zumindest. Sorry wenn ich das nicht richtig rübergebracht habe - Da fehlt's mir sicher noch an Übung und Präzision.
Wenn wir schon dabei sind: Mit Krümmung meine ich immer innere Krümmung (sofern ich nichts anderes schreibe).
Der Batzen beansprucht Raum (im Raum), also muß er positiv gekrümmt sein.
Nein, das hat meines Erachtens nichts damit zu tun. Ich denke ein Batzen ist deshalb positiv gekrümmt da seine Oberfläche stets eine 2Sphäre bildet. Und das steht in direktem Zusammenhang mit obigen b): Ich habe ein Problem (mit dem Verständnis) der Kante.

Zwei ungekrümmte Ebenen sollen sich schneiden (-> in einer Linie). Ich schneide links und unter der Linie die "überstehenden" Teile beider Ebenen ab. Dann gehört nach meinem Verständnis diese Linie (bzw. alle Punkte, die diese Linie bilden) zu beiden Ebenen. Und genauso sieht für mich eine Kante z.B. an einem Quader aus: An diesem Punkt befinde ich mich sowohl in der einen als auch in der anderen Ebene. Deshalb verstehe ich das Singularitätsproblem hier nicht.

Nehmen wir noch einmal einen Punkt, der genau an der Kante eines Zylinders liegt:
In Richtung der äußeren Krümmung der Mantelfläche (Also einfach die Zylinderkante entlang) stößt er an einen weiteren Punkt - In diese Richtung "darf ich mich" nach Eurer Aussage "bewegen".
In Richtung des Zylinderpols stößt der Punkt ebenfalls an einen anderen Punkt - In diese Richtung "darf ich mich" nach Eurer Aussage "nicht bewegen".
Woher weiß das der Punkt (um nicht wieder - vielleicht diesmal null-dimensionale - Flatlander zu bemühen) - Er gehört schließlich zu beiden Flächen?

Aber das Verständnisproblem liegt augenscheinlich einzig bei mir ... :rolleyes:

JoAx
13.11.09, 00:28
Hallo SCR,

alles IMHO.

Vorab über uns 3d-Weltler. Wir haben zwar bildliches Verständnis von einer Fläche, im Gegensatz zu einem 4d-Objekt, sind aber dennoch nicht im Stande eine echte 2d-Figur mit der Dicke=0 zu erschaffen, da wir nur 3d-Körper zur Verfügung haben. Ok? Deswegen fallen unsere zusammengesetzte "Flächen" nicht auseinander.

Unsere Punkte, Dimension=0, sind trotzdem "3d" (zumindestens im Kopf), da wir sie von 3 Seiten anschauen können. Das muss man abstellen!, wenn man sich in 2d begibt. Gell?


Ich denke "b)" - Eure Argumentation sagt aber "a)". Nur wären doch dann alle derart zusammengesetzten Objekte nichts anderes als Kartenhäuser ohne Zusammenhalt ...


Im Bild siehst du 2 "echte" 2d-Flächen die aus "echten" 2d-Punkten zusammengesetzt sind, und von der Kante angeschaut werden. Echte 2d-Flächen würden keine Schnittmenge (Linie) ergeben (imho). (?) Die Kanten sind angedeutet. Das Rote Bereich soll Krümmung darstellen.


172


Gruss, Johann

PS: Das Bild ist ein Anhang. Beim Anklicken sieht man es in besserer Auflösung, wer das noch nicht weiss.

SCR
13.11.09, 08:32
Beim Anklicken sieht man es in besserer Auflösung, wer das noch nicht weiss.
Die Original-Auflösung direkt im Beitrag ist ja auch nicht so dolle http://www.smileyparadies.de/girls/smilie_girl_095.gif ;)
Echte 2d-Flächen würden keine Schnittmenge (Linie) ergeben (imho).
Das ist die Kernfrage (IMHO).
So könnte ich mir das noch vorstellen: Ich habe einen "echten" 3D-Quader vor mir. Alle seine Flächen decke ich mit exakt zugeschnittenen Fliesflächen der Dicke 0 ab.
Hinterher sähe eine Ecke etwa so aus - Man sieht nur noch die Flies-Stücke (wohlgemerkt: Stärke=0):
http://img402.imageshack.us/img402/571/abdecken.jpg
Bei dieser Vorstellung würde es keine Schnittmengen der Fliesflächen geben - und die von Dir so benannte "Kanten-Singularität" wäre für mich grob nachvollziehbar.
Ist es das? :rolleyes:

Nebenbei: Eine unendlich große Kugel - Ihre Oberfläche wäre euklidisch? :rolleyes:

JoAx
13.11.09, 10:19
Hallo SCR,


und die von Dir so benannte "Kanten-Singularität" wäre für mich grob nachvollziehbar.
Ist es das?


mir scheint, du übertreibst es wieder mit den Interpretationen der Bilder. Nicht alles, was es in der Mathe gibt, hat eine Entsprechung in der Natur, sonst würde sie Physik heissen. Dennoch sind diese "irrealen" Betrachtungen von realer, physikalischer Bedetung, denn spätestens wenn wir bei einer 3Fläche angelangt sind, sind wir in unserer räumlicher, physikalischer Welt, wo die herausgarbeiteten Gesetzmässigkeiten angewendet werden können.

Hier noch eine Möglichkeit sich die Kante zu überlegen. Stell dir eine unendlich hohe (positive) Krümmung vor. Egal wie nah du dich den "Knick" reinzoomst, siehst du immer eine scharfe Ecke. Unendlich hohe Krümmung = Singularität. Es kann keine Information über die Bewegungsrichtung in einer Oberfläche in die andere gelangen.


Nebenbei: Eine unendlich große Kugel - Ihre Oberfläche wäre euklidisch?

Das wird wohl stimmen.

Mir scheint, dass du dich deswegen so streubst, weil du so etwas wie: "Eine n-Fläche erfordert einen (n+1)-Körper", höhren willst. Dann wäre dein "Wasserplanet" zwingend notwendig. Ist es so?


Gruss, Johann

SCR
13.11.09, 10:20
Bei dieser Vorstellung würde es keine Schnittmengen der Fliesflächen geben - und die von Dir so benannte "Kanten-Singularität" wäre für mich grob nachvollziehbar.
Ist es das? :rolleyes:
Andererseits wäre das letztendlich aber doch "nur" eine Betrachtung der "allernächsten Umgebung" der Oberfläche eines Objekts - und/aber eben nicht der Oberfläche selbst. :rolleyes:
Stell dir eine unendlich hohe (positive) Krümmung vor.
Jetzt bin ich vermutlich wieder nur zu blöd:
Ich dachte K=1/r1 * 1/r2. r1 und r2 müssten dann doch unendlich klein sein (und auf der gleichen Seite liegen). Eine unendlich hohe (positive) Krümmung könnte demnach (nur) ein einzelner Punkt haben ... Ein Punkt hat aber keine "Kante" ... :rolleyes:
Deswegen fallen unsere zusammengesetzte "Flächen" nicht auseinander.
Stelle ich ein Blatt senkrecht auf ein anderes (= 90°) können beide Blätter auseinanderfallen.
Knicke ich ein Blatt in der Mitte und klappe es auf (= 90°) kann es nicht auseinanderfallen.
Und nun ersetze ich hier "Blatt" durch "2D-Fläche (= Dicke 0)" - Dadurch ändert sich doch meines Erachtens nach nichts an dem Paradoxon: Ich blick's nicht.
Mir scheint, dass du dich deswegen so streubst, weil du so etwas wie: "Eine n-Fläche erfordert einen (n+1)-Körper", hören willst. Dann wäre dein "Wasserplanet" zwingend notwendig. Ist es so?
Nein - Hier in diesem Thread habe ich eigentlich nur "Dinge, die auf meinem Schreibtisch stehen (könnten)", vor Augen.
Deshalb kann ich adhoc auch Deine Aussage für "Eine n-Fläche erfordert einen (n+1)-Körper" nicht nachvollziehen.
Aber anscheinend wäre diese für mich schon interessant - Deshalb werde ich einmal darüber nachdenken. ;)
Mir scheint, dass du dich deswegen so streubst,
Ich "streube mich" deshalb, weil ich etwas nicht verstehe.
In einem solchen Fall nehme ich (häufig/gerne) bewußt eine "provozierende" Gegenposition ein um Argumente dagegen zu hören / mit Argumenten widerlegt zu werden.
Mir kommt es dabei eigentlich nur darauf an, etwas zu verstehen was ich vorher nicht verstanden habe ... Ob ich dann letztendlich Recht behalte oder nicht interessiert mich dabei ehrlich gesagt weniger (bis gar nicht ;)): Von daher trifft der Begriff "streuben" meine Motivation in meinen Augen nicht so ganz.
Meine "Reputation" juckt mich dabei im Übrigen gar nicht, ihr Ankratzen nehme ich dabei gerne in Kauf: Du weißt doch "Ist der Ruf erst ..." ;).
Ich möchte eben nur - sofern möglich - hinterher immer (etwas) schlauer als vorher sein.

SCR
13.11.09, 14:39
Hallo JoAx,
da fällt mir etwas auf:
Unendlich hohe Krümmung = Singularität. Es kann keine Information über die Bewegungsrichtung in einer Oberfläche in die andere gelangen.

Eine unendlich große Kugel ist euklidisch. Warum? Weil sich ihre positive Gesamtkrümmung auf eine unendlich große Fläche verteilt: Lokale Krümmung -> 0.

Ich nehme eine endlich große Kugel (Gesamtkrümmung > 0) und verforme diese zu einem Quader. Dabei "wandert" / konzentriert sich die positive Gesamtkrümmung in die / den Kanten. Dadurch erhalte ich lokal auf den Quader-Flächen ein K=0, und an den im Verhältnis zur Gesamtoberfläche unendlich kleinen "Oberfläche" der Kanten eine lokale Krümmung -> ∞.

In Summe wäre der Quader (Krümmung Flächen + Krümmung Kanten!) positiv gekrümmt - Womit wir aber (leider) wieder bei meiner These wären. :rolleyes:

JoAx
13.11.09, 17:08
Hallo SCR,

ich denke, in deinem Fall kanst nur du selbst den Weg finden.

Die anderen können nur Hilfestellung geben, überzeugen musst du dich selbst.

Frage: Wie viele gerade Linien sind einer anderen Geraden senkrecht?
Oder: Wie viele Ebenen kann man durch eine Gerade "führen"?


Gruss, Johann

SCR
13.11.09, 18:46
ich denke, in deinem Fall kanst nur du selbst den Weg finden. Die anderen können nur Hilfestellung geben, überzeugen musst du dich selbst.
Aber http://1.1.1.2/bmi/www.smileyparadies.de/girls/smilie_girl_168.gif ... Schnief ... Soll das heißen, ich werde mit meinem Problem ganz alleine gelassen? http://www.smileyparadies.de/girls/smilie_girl_237.gif
Frage: Wie viele gerade Linien sind einer anderen Geraden senkrecht?
Oder: Wie viele Ebenen kann man durch eine Gerade "führen"?
Kommt IMHO darauf an: Hat die Gerade einen Stärke von 1,616252*10^−35 m oder von 0?

JoAx
13.11.09, 18:57
Aber http://1.1.1.2/bmi/www.smileyparadies.de/girls/smilie_girl_168.gif ... Schnief ... Soll das heißen, ich werde mit meinem Problem ganz alleine gelassen? http://www.smileyparadies.de/girls/smilie_girl_237.gif


Nein. Das soll heissen, dass du auch selbst nach Argumenten suchen sollst, die gegen die von dir eingenommene Position sprechen. Sie sind dann in der Regel (für dich) die überzeugendsten.


Kommt IMHO darauf an: Hat die Gerade einen Stärke von 1,616252*10^−35 m oder von 0?


Nein. Darauf kommt es nicht an.


Gruss, Johann

SCR
13.11.09, 19:38
Nein. Das soll heissen, dass du auch selbst nach Argumenten suchen sollst, die gegen die von dir eingenommene Position sprechen.
Blöde Idee, s**blöde Idee ... ;)
Nein. Darauf kommt es nicht an.
Auch nicht bei den Ebenen/Geraden, die ich durchstecken soll? http://www.electronicattack.de/images/smilies/gruebel.gif


P.S.: Kleiner Hinweis: Der "Political Correctnes"-Filter funktioniert nicht richtig - Bei s** musste ich die Sternchen selber einfügen.

SCR
13.11.09, 23:51
Nein. Das soll heissen, dass du auch selbst nach Argumenten suchen sollst, die gegen die von dir eingenommene Position sprechen.
Der beste Ansatz war (IMHO) noch der mit dem Flies ... Aber der passt nicht, das wäre obwohl Dicke=0 trotzdem AUF der Oberfläche und eben nicht DIE Oberfläche.
Und dann musst Du auch noch mit Deinen Singularitäten und unendlich großen Krümmungen kommen - Das passt(e) doch einfach super-prima zusammen. :)
So eine schöne Lösung - aber nee, jetzt darf man wieder Gegenargumente dafür (oder heißt das jetzt hier dagegen? http://www.electronicattack.de/images/smilies/gruebel.gif) suchen.
Also gut: 'Mal sehen, ob man evtl. den Flies-Ansatz noch etwas verbessern kann ... Ansonsten eben was anderes. ;)

JoAx
14.11.09, 05:21
Hallo SCR,


Blöde Idee, s**blöde Idee ...


ich habe schon oft versucht eigene Erklärungen für bestimte einzelne Phänomene zu finden. Hin und wieder war ich sehr euphorisch, wenn ich meinte, ich hätte eine widerspruchsfreie Beschreibung gefunden (meistens morgens, bei der ersten Zigarette). Nachdem sich die Euphorie legte (so gegen Mittag), habe ich angefangen eigene Schlussfolgerungen zu hinterfragen, und es dauerte in der Regel 15 min. bis meine "Theorie" vom Tisch war (ziemlich kurze Halbwertszeit :D). Die längste (vor diesem Forum) hat eine Woche gehalten. :cool:

Ich spreche also aus eigener Erfahrung in der Selbstwiderlegung. Das ist immer überzeugender, als von anderen widerlegt zu werden. Weil man erst dann es wirklich kapiert hat. ;)

Frag dich einfach, was jemand anderes gegen deine Argumente anführen könnte. Hin und wieder zumindestens.

Was könnte man z.B. darauf:


Auch nicht bei den Ebenen/Geraden, die ich durchstecken soll? http://www.electronicattack.de/images/smilies/gruebel.gif


erwidern?


Gruss, Johann

SCR
14.11.09, 07:40
Was könnte man z.B. darauf [...] erwidern?
Man könnte erwidern: Ja. Dann wäre es eine mathematische Betrachtung. Dann wäre die korrekte Antwort ∞. Wenn man die durchzusteckenden Geraden/Ebenen überall "etwas abschneidet" erhält man einen Vollzylinder.
Man könnte erwidern: Nein. Dann wäre es eine physikalische Betrachtung. Dann käme in etwa so etwas heraus:
http://www.mosmann-kaminoefen.business.t-online.de/shops/heiztechnik/Media/Shop/spezialbuerste_viessmann_rund120mm_2558068_om_gr.j pg
Deshalb meine Frage.

Ansonsten hast Du absolut Recht. ;)

JoAx
14.11.09, 12:41
Hi SCR,


Man könnte erwidern: Nein. Dann wäre es eine physikalische Betrachtung. Dann käme in etwa so etwas heraus:


du denkst immer noch 3d über 2d-Dinger. Ein 2d-ler befindet sich in der Oberfläche, und nicht auf ihr, so wie wir uns im 3d-Raum befinden, und nicht auf ihm. So würde ein 2d-ler die andere Fläche einfach nicht wahrnemen können, auch wenn einer der Punkte zu beiden gehöhren würde.

173


Gruss, Johann

Jogi
14.11.09, 18:23
Wenn's sonst Keiner tut, mach ich's hier mal:


Johann, hab Dank für deine Geduld!

SCR
14.11.09, 20:44
Hallo JoAx,

ich kann mich Jogi natürlich nur anschließen: Auch von meiner Seite vielen Dank!
Anderseits weiß ich aber auch nicht was an meiner Antwort falsch gewesen sein sollte: Du hattest nach Anzahlen gefragt - Mathematisch unendlich / Physikalisch endlich. :confused:

Aber möglicherweise verstehe ich es, falls mir jemand diese Frage beantworten könnte: Erreicht der Flatlander sein Ziel oder nicht (und kurz angerissen warum)?

http://img339.imageshack.us/img339/609/einblattoderzwei.jpg
(Blick eines 3D-Betrachters auf ein 2D-Objekt)

JoAx
14.11.09, 22:05
Hi SCR,


Aber möglicherweise verstehe ich es, falls mir jemand diese Frage beantworten könnte: Erreicht der Flatlander sein Ziel oder nicht (und kurz angerissen warum)?

http://img339.imageshack.us/img339/609/einblattoderzwei.jpg
(Blick eines 3D-Betrachters auf ein 2D-Objekt)

http://www.smileyparadies.de/crazy/14.gif (http://www.smileyparadies.de) http://www.smileyparadies.de/crazy/30.gif (http://www.smileyparadies.de)

Kannst du die Wände hochlaufen? Oder auf der Decke deines Zimmers?

Der Flachländer kann seine Fläche nicht verlassen. Und eine Kante bedeutet das ENDE einer Fläche. Der hat keine Ahnung, dass es noch weiter geht. Er kann nur umkehren und zurück, oder sonst wohin laufen. Wenn er in die andere Fläche gelangen könnte, dann hätte die Bewegungsrichtung in der neuen Fläche aber gaaaar nichts mit der Bewegungsrichtung in der alten zu tun. Er könnte in der neuen Fläche jedes Ziel erreichen können. Die Bewegungsrichtung wäre zufällig/frei wählbar.

Bring die beiden gezeichneten Flächen weit weit auseinander und in beliebiger Ausrichtung, und stell sich die Frage neu. Für dich gibt's einen Unterschied, für die Flachländer - NULL.
Jetzt mach schon!http://www.smileyparadies.de/crazy/50.gif (http://www.smileyparadies.de)


Grüssi :D

Eyk van Bommel
14.11.09, 23:17
Vielleicht redet ihr "SCR und JoAx" auch nur aneinander vorbei?
Wenn es eine (um 90°) gekrümmte Ebene ist, dann erreicht er das Ziel. Er kann es sogar sehen, da das Licht der Krümmung folgt. Wenn es aber zwei Senkrecht aufeinander stehende Ebenen sind, dann benötigt sowohl der Flachländler wie auch das Photon an der Kante einen Impuls (der aus dem „Nichts kommt) um eine Richtungsänderung zu erfahren.

Der Flachländler kann jedes Ziel erreichen, wenn dieser mit 2 Ortskoordinaten beschrieben werden kann.

Gruß
EVB

SCR
14.11.09, 23:41
Hallo JoAx,
das Wichtigste (!) zuerst: Deine Smilies finde ich super!http://wgc-bs.de/smilies/friendly/smilie.gif

Ansonsten gebe ich EVB (Wo hast Du Dich denn rumgetrieben?) völlig Recht: Du hättest mich ruhig fragen können ob es sich um eine geknickte Fläche oder um zwei handelt :p ;).

Jetzt einmal Flachs beiseite: Darum geht es doch im Kern, oder?
Stellen die beiden Flächen, die wir betrachten, eigentlich eine dar, die lediglich eine äußere Krümmung aufweist.
Oder handelt es sich um zwei getrennt zu betrachtende Flächen, die nichts miteinander zu tun haben, außer "auf Kante" zueinander stehen.
(Und darauf spielte doch auch Dein Beispiel an - oder? Am Anfang habe ich nicht verstanden warum Du mich das fragst.)

Ich mach hier erst einmal einen Break.

P.S.:
Vielleicht redet ihr "SCR und JoAx" auch nur aneinander vorbei?
Das tun wir doch NIE - oder JoAx? ;)

JoAx
15.11.09, 00:40
Das tun wir doch NIE - oder JoAx?


Ich sag's Mal so:

Manchmal hat man das Gefühl:

SCR -> http://www.smileyparadies.de/crazy/9.gif (http://www.smileyparadies.de)
Jemand anderes beim Versuch SCR etwas zu erklären -> http://www.smileyparadies.de/crazy/41.gif (http://www.smileyparadies.de)


Gruss, Johann ;)

SCR
15.11.09, 07:08
http://www.greensmilies.com/smile/smiley_emoticons_ugly_gruebel.gif Fasst man beide Zeilen als Gleichungen auf ... lässt sich SCR rauskürzen ... und das Problem wäre erledigt.
Damit könnte man auch die in Zeile 2 aufgezeigten Kollateralschäden dieses Forum betreffend vermeiden.
Im Sinne des Gemeinwohls also ein durchaus überlegenswerter Ansatz ;).

SCR
15.11.09, 07:12
Hallo zg,
Blödsinn, du hast ein Siebeneck eingezeichnet!
vielleicht ist das ja noch ein Ansatzpunkt - Wie meintest Du das: Als Scherz oder hatte das einen ernsthaften Hintergrund?
Im zweiten Fall: Wie kommst Du auf Sieben? http://forum.gleitz.info/images/smilies/gruebel.gif

zeitgenosse
15.11.09, 12:18
Wie meintest Du das: Als Scherz oder hatte das einen ernsthaften Hintergrund?
Es ist so, wie ich's sagte.

Im zweiten Fall: Wie kommst Du auf Sieben?Zähle einmal die Ecken deiner Figur, die du in den 3-Raum eingezeichnet bzw. die du auf den Quader gelegt hast.

Drei Ebenen, die aufeinander senkrecht stehen!

http://img690.imageshack.us/img690/2976/quadergekrmmt.jpg

Diese Figur ist übrigens ähnlich der Ansicht des Würfels aus der Perspektive. Um ein Dreieck handelt es sich dabei nicht.

Gr. zg

SCR
15.11.09, 19:58
Hallo zg,
Drei Ebenen, die aufeinander senkrecht stehen!
[1] Ja, für einen 3D-Betrachter drei quadratische Flächen, die in 90°-Winkeln zueinander gestellt sind ("Bierdeckel-Kartenhaus").
-> Jede für sich ist euklidisch, es besteht keine Verbindung zwischen den Flächen.
Um ein Dreieck handelt es sich dabei nicht.
[2] Oder ich nehme eine euklidische Fläche in Form eines gleichschenkligen Dreiecks (-> jeweils 60°), drücke diese in der Mitte nach oben und forme von der dadurch entstehenden zentralen Wölbung/Spitze ausgehend drei Kanten genau auf die Mitten der Seitenflächen des Dreiecks zulaufend.
Dabei dehne ich die gesamte Fläche so, dass die Kanten jeweils "von außen betrachtet" einen 90°-Winkel bilden.
Dadurch kann für einen 3D-Betrachter optisch das gleiche Ergebnis wie mit drei senkrecht zueinanderstehenden Flächen erzielt werden (= [1]). Die hier gedehnte, abgeschlossene Oberfläche wäre aber im Gegensatz zur ersten Lösung gekrümmt (Da die Innenwinkel des Ausgangsdreiecks jetzt jeweils 90° betragen).

Ist die Oberfläche eines Quaders geschlossen oder offen? :rolleyes:
Bzw. ist das überhaupt die richtige Frage (bezüglich der Verwendung der Bezeichnung "offen")? http://www.regner-support.net/images/regner/smilies/gruebel.gif)

JoAx
16.11.09, 06:32
http://www.greensmilies.com/smile/smiley_emoticons_ugly_gruebel.gif Fasst man beide Zeilen als Gleichungen auf ... lässt sich SCR rauskürzen ... und das Problem wäre erledigt.
Damit könnte man auch die in Zeile 2 aufgezeigten Kollateralschäden dieses Forum betreffend vermeiden.
Im Sinne des Gemeinwohls also ein durchaus überlegenswerter Ansatz ;).

:D:D:D

Aber so leicht machst du es uns nicht, stimmt's?


Gruss mit Spässle, Johann

SCR
16.11.09, 07:41
Aber so leicht machst du es uns nicht, stimmt's?
1. Denken und Handeln sind zwei paar Stiefel. :p
2. Mir ist ein Schludrigkeitsfehler bei der Elimination der Variable SCR unterlaufen: Die Blutlache bliebe in den "Gleichungen" so oder so zurück -> Somit liegt's gar nicht an mir. :D

Und deswegen auch gleich noch einen oben drauf: Jede endliche Fläche ist (positiv?) gekrümmt.
(Kein Scherz meinerseits - Denn es war eigentlich Deine Idee, JoAx! ;))

zeitgenosse
16.11.09, 11:13
Ist die Oberfläche eines Quaders geschlossen oder offen?

Definition: Eine Fläche ohne Rand heißt geschlossen, wenn sie kompakt ist.

Frage: Ist ein Kubus resp. dessen Fläche aber auch kompakt?

Antwort: Kompakt ist eine Fläche dann, wenn sie abgeschlossen und beschränkt ist.

Fazit: Topologisch sind Kugel und Würfel einander somit verwandt. Es besteht aber ein differentialgeometrischer Unterschied zwischen beiden. Der Rand eines Würfels ist nicht überall glatt und somit auch nicht überall differenzierbar. Trotzdem ist ein Hexaeder zur Kugel homöomorph. Beweis durch stetige Deformation.

Gr. zg

SCR
16.11.09, 12:22
Antwort: Kompakt ist eine Fläche dann, wenn sie abgeschlossen und beschränkt ist.
Und das trifft für einen Quader/Würfel zu - Richtig? :rolleyes:
[...] und somit auch nicht überall differenzierbar.
Um eine innere Krümmung an einem Punkt einer Fläche zu bestimmen benötige ich mindestens 4 umliegende (= jeweils zwei sich gegenüberliegende) Referenzpunkte - korrekt? :rolleyes:

Jogi
17.11.09, 02:07
Hi SCR.


Jede endliche Fläche ist (positiv?) gekrümmt.

Wie kommst 'n da drauf?

zg versucht, die Verschiedenheit von Topologie und Geometrie klar zu machen, und du schmeißt beides wieder in einen Topf.


Antwort: Kompakt ist eine Fläche dann, wenn sie abgeschlossen und beschränkt ist.
Und das trifft für einen Quader/Würfel zu - Richtig?
Nicht wirklich.
Kompakt kann nur eine Fläche sein.
Der Sechsflächner wird aber von deren sechs an der Zahl begrenzt, daher der Name.


Um eine innere Krümmung an einem Punkt einer Fläche zu bestimmen benötige ich mindestens 4 umliegende (= jeweils zwei sich gegenüberliegende) Referenzpunkte
Du kannst doch nicht zur Bestimmung der inneren Krümmung einer Fläche Referenzpunkte von einer anderen Fläche verwenden.
Das ist imho nämlich das was du die ganze Zeit machen willst.
Du willst damit der Gesamtheit der Flächen eine summarische Krümmung zuweisen, die der sphärischen Krümmung der Kugel (dem runden Würfel) entspricht.
Wozu?
Um zu zeigen, daß der Würfel der Kugel homöomorph ist? - Das ist trivial.

Ich muß immer wieder auf deine Eingangsfrage (paralleler Kollisionskurs) zurückkommen:
Das ist primär keine Frage der Topologie, sondern der Geometrie.
Und nichteuklidische Geometrien sind auf den verschiedensten Topologien denkbar, egal ob kompakt, berandet, einfach zusammenhängend...

Zwischenfazit:
Wenn wir uns in die zwei Raumschiffe setzen, mit parallelem Kurs starten, den Joystick nicht anfassen und nach elfundachtzig Quintilliarden von Jahren immer noch den gleichen Abstand voneinander messen, dann können wir ziemlich sicher sein, in einem euklidischen 3Raum zu Leben.
Ob dieser aber einen Rand hat, und wie dieser aussieht, wissen wir dann immer noch nicht.

In allen anderen Fällen (Kollision/zunehmender Abstand) können wir vom nichteuklidischen 3Raum ausgehen.
Dabei müssen wir aber alternativ zu einer weiteren räumlichen Dimension auch über die zeitliche Dimension nachdenken, die Abstandsveränderung könnte ja auch der Expansion/Kontraktion und damit der zeitlichen Inhomogenität geschuldet sein.


Gruß Jogi

möbius
17.11.09, 17:44
Das Problem könnte nach wie vor in der Frage liegen:
Warum existiert das Universum:confused:
Aber die Physiker/Kosmologen/String-Theoretiker/Loop-Quantengravitationstheoretiker usw. sind ja erst einmal damit beschäftigt, die Frage nach dem WIE zu beantworten ...:D ;)
Gruß, möbius

SCR
17.11.09, 18:38
Hallo Jogi,
Wie kommst 'n da drauf?
Lass' uns das bitte hinten anstellen - Das klärt sich vermutlich von alleine (und Du wirst am Ende wahrscheinlich dabei Recht behalten ;)).
Nicht wirklich.
Aha (Ein einfaches "Ja" oder "Nein" kennen Physiker wohl nicht ;)).
Du kannst doch nicht zur Bestimmung der inneren Krümmung einer Fläche Referenzpunkte von einer anderen Fläche verwenden. Das ist imho nämlich das was du die ganze Zeit machen willst. [...]

Das mag IYHO sein ist aber nicht IMHO:
Ich wollte mich versichern wieviele Umgebungs-Punkte man braucht um die innere Krümmung an einem Punkt einer Fläche zu bestimmen.
Wer spricht jetzt hier von anderen Flächen? Also ich nicht. :confused:
(So kommen wir dann zwangsläufig zu einer differenzierten Betrachtung der Punkte innerhalb der Fläche und der an der Kante - Oder etwa nicht?).
Zwischenfazit:
Wenn wir uns in die zwei Raumschiffe setzen, mit parallelem Kurs starten, den Joystick nicht anfassen und nach elfundachtzig Quintilliarden von Jahren immer noch den gleichen Abstand voneinander messen, dann können wir ziemlich sicher sein, in einem euklidischen 3Raum zu Leben.
Ob dieser aber einen Rand hat, und wie dieser aussieht, wissen wir dann immer noch nicht.
In allen anderen Fällen (Kollision/zunehmender Abstand) können wir vom nichteuklidischen 3Raum ausgehen.
Dabei müssen wir aber alternativ zu einer weiteren räumlichen Dimension auch über die zeitliche Dimension nachdenken, die Abstandsveränderung könnte ja auch der Expansion/Kontraktion und damit der zeitlichen Inhomogenität geschuldet sein.
Kein Widerspruch (find' ich sogar recht adrett :D).

SCR
17.11.09, 18:49
Hallo zg,
Topologisch sind Kugel und Würfel einander somit verwandt.
Topologisch sind eine abgeschlossene Kugel und ein abgeschlossener Quader identisch.
Falls Nein: Worin bestehen die topologischen Unterschiede? :rolleyes:

Eyk van Bommel
17.11.09, 22:42
Hi Jogi,
schön dass ich mal wieder mit dir Diskutieren kann – wie geht’s:)
Zwischenfazit:
Wenn wir uns in die zwei Raumschiffe setzen, mit parallelem Kurs starten, den Joystick nicht anfassen und nach elfundachtzig Quintilliarden von Jahren immer noch den gleichen Abstand voneinander messen, dann können wir ziemlich sicher sein, in einem euklidischen 3Raum zu Leben.
In allen anderen Fällen (Kollision/zunehmender Abstand) können wir vom nichteuklidischen 3Raum ausgehen
Das sehe ich nicht ganz so. Man spricht ja gerne von der Raumzeit.
Daher (wie immer IMHO)
Würde Abstand größer, dann vergeht die Zeit schneller.
Wird der Abstand kleiner, dann vergeht die Zeit langsamer.
Abstandmessung erfolgt über z.b. Photonen = Abstand bleibt (lokal) konstant
Meine Erfahrung/Fazit:
Die Welt erlaubt uns nicht lokal Änderungen in der Raumzeit zu messen.

Gruß
EVB

Jogi
18.11.09, 00:58
Ich wollte mich versichern wieviele Umgebungs-Punkte man braucht um die innere Krümmung an einem Punkt einer Fläche zu bestimmen.
Ich bin kein Geometer, aber ich würde sogar sagen, sechs.
Soweit ich es verstanden habe, muß man die Winkelabweichungen eines Dreiecks messen, das man vergrößert oder verkleinert.
In hyperbolischen Geometrien verkleinern sich die Winkel beim Vergrößern des Dreiecks, in quantitativer Abhängigkeit vom Krümmungsgrad (et vice versa).

Wer spricht jetzt hier von anderen Flächen? Also ich nicht. :confused:
Sorry, ich dachte, es geht dir um die Gesamtkrümmung der kompletten Körperoberfläche, einer Näherung zur äquivalenten Sphäre.

(So kommen wir dann zwangsläufig zu einer differenzierten Betrachtung der Punkte innerhalb der Fläche und der an der Kante - Oder etwa nicht?).
Brauchen wir das?
Der letzte Punkt einer Fläche gehört nunmal zu ihrem Rand (und damit zur Fläche).
Ob dieser Punkt gleichzeitig zu einer anderen Fläche gehört ist doch schnurz, ein Punkt ist nulldimensional, davon kannst du beliebig viele zusammenpacken, die brauchen immer noch keinen Raum (nicht mal im 1Raum).

Ein Randpunkt unterscheidet sich selber nicht von einem Punkt in der Fläche, nur seine Umgebung (in diesem, und nur in diesem 2Raum) ist eine andere.
Der Punkt ist nicht ringsum von anderen Punkten umgeben.
Deshalb ist dieser 2Raum auch hier zu Ende, es geht einfach nicht weiter.


Gruß Jogi

Jogi
18.11.09, 01:41
Hi Eyk.

Hi Jogi,
schön dass ich mal wieder mit dir Diskutieren kann – wie geht’s:)
Es ging schon mal besser, aber das wird schon wieder...:)


Die Welt erlaubt uns nicht lokal Änderungen in der Raumzeit zu messen.
Gut aufgepasst.
Was schließen wir daraus?
- Mit dieser Methode lässt sich das fünfte Postulat also auch nicht beweisen/widerlegen.
Zumindest dann nicht, wenn die Krümmung allein zeitlicher Natur ist.


Gruß Jogi

zeitgenosse
18.11.09, 07:05
Jede endliche Fläche ist (positiv?) gekrümmt.

In Bezugnahme auf die elementare Differentialgeometrie (Gaußsche Flächentheorie) gilt:

Du bestimmst zwei Schmiegekreise (diese verkörpern die beiden Hauptkrümmungen einer Fläche):

http://de.wikipedia.org/wiki/Kr%C3%BCmmungskreis

http://www.ottmarlabonde.de/L1/Kruemmung01.html

Die Schmiegekreise befinden sich auf zueinander senkrecht stehenden (Hilfs)-Flächen, welche die zu untersuchende Fläche in der am Ort der Krümmung anliegenden Tangentialebene lotrecht schneiden:

http://upload.wikimedia.org/wikipedia/commons/e/eb/Minimal_surface_curvature_planes-en.svg

Daraus lässt sich die lokale Krümmung berechnen:

K = k1k2 = 1/(R1R2)

Für eine ebene Kurvenkrümmung reicht bereits ein einziger Schmiegekreis. Für eine Fläche benötigst du deren zwei (weil die Fläche bekanntlich mehr Freiheitsgrade besitzt).

Für eine 2-Sphäre (Kugeloberfläche) ergibt sich somit eine Gaußsche Krümmung von:

K = 1/R²

Beim Kubus (Würfel, Quader) ist es schwieriger, weil die Krümmung an den pathologischen Zonen (Ecken) nicht ermittelbar ist. Ansonsten sind seine sechs Flächen so eben wie die Euklidische Ebene.

Die totale Krümmung eines Körpers ergibt sich aus dem Oberflächenintegral über den Bereich einer Oberfläche. Um tiefer in diese Materie einzudringen, muss der Riemannsche Krümmungstensor bemüht werden. Umfassende Kenntnisse aus der Differentialgeometrie sind dazu meist unerlässlich. Ansonsten stochert man bloss im Trüben herum.

Zur Vertiefung ist der Griff zur Fachliteratur somit angesagt, z.B. mittels folgender Auswahl:

- Kühnel, Differentialgeometrie (Vieweg+Teubner)

- Bär, Elementare Differentialgeometrie (de Gruyter)

- Klotzek, Einführung in die Differentialgeometrie (Verlag Harri Deutsch)

Gr. zg

Timm
18.11.09, 10:21
Zwischenfazit:
Wenn wir uns in die zwei Raumschiffe setzen, mit parallelem Kurs starten, den Joystick nicht anfassen und nach elfundachtzig Quintilliarden von Jahren immer noch den gleichen Abstand voneinander messen, dann können wir ziemlich sicher sein, in einem euklidischen 3Raum zu Leben.


Einspruch, Jogi,

das kann - schon der mitlesenden Leute zuliebe - so nicht stehen bleiben.

Ein euklidisches Universum (K=0) ist nicht statisch! Es expandiert für alle Zeiten. Die Raumschiffe entfernen sich voneinander.

Gruß, Timm

Jogi
18.11.09, 11:17
Hi Timm.



Ein euklidisches Universum (K=0) ist nicht statisch! Es expandiert für alle Zeiten. Die Raumschiffe entfernen sich voneinander.



Wenn Parallelen nicht parallel bleiben, haben wir es nicht mit einer euklidischen Geometrie zu tun, oder?

Bauhof
18.11.09, 11:31
Daraus lässt sich die lokale Krümmung berechnen:

K = k1k2 = 1/(R1R2) Gr. zg

1. Du hast einen Link mit der Extension "svg" angegeben. Welches Format ist das? Kann das der AdobeReader verarbeiten?

2. Ich habe noch nie in der Literatur die Formel für die Krümmung einer n-dimensionalen Kugeloberfläche gefunden. Ist dir da ein (deutschsprachiger) Link bekannt?

M.f.G Eugen Bauhof

Bauhof
18.11.09, 11:36
Ein euklidisches Universum (K=0) ist nicht statisch! Es expandiert für alle Zeiten. Die Raumschiffe entfernen sich voneinander. Gruß, Timm
Hallo Timm,

Ich meine, ein Universum expandiert (aus irgendeinem finsteren Grund) oder es expandiert nicht. In welcher Weise sollte die dei Existenz der Expansion davon abhängen, ob das Universum euklidisch, elliptisch oder hyperbolisch gestaltet ist?

M.f.G. Eugen Bauhof

zeitgenosse
18.11.09, 12:17
Wenn Parallelen nicht parallel bleiben, haben wir es nicht mit einer euklidischen Geometrie zu tun, oder?

In der Projektiven Geometrie berühren sich Parallelen in einem Fluchtpunkt.

Solches entspricht auch der menschlichen Erfahrung:

http://sites.google.com/site/futurephysics/Home/schienen_eukl.jpg

Gr. zg

Jogi
18.11.09, 12:38
Hi zg.

In der Projektiven Geometrie berühren sich Parallelen in einem Fluchtpunkt.

Is schon klar, geht aber an unserer Betrachtung vorbei.
Die beiden Parallelen beobachten sich nur gegenseitig, und auch nur im rechten Winkel zueinander.
Und da fragen wir uns, ob in einem expandierenden, ansonsten aber euklidischen 3Raum die Beiden eine Zunahme ihres Abstandes beobachten könnten.
Das ist jetzt eine Vermengung von allgemeinen Aussagen der Geometrie und beobachtbarer Physik, und da sehen wir schon die Probleme die sich daraus ergeben können.


Gruß Jogi

SCR
18.11.09, 13:17
Hallo Bauhof,
In welcher Weise sollte die Existenz der Expansion davon abhängen, ob das Universum euklidisch, elliptisch oder hyperbolisch gestaltet ist?
IMHO: Man muß die Frage andersherum stellen (So wie es nach meiner Einschätzung auch Jogi tut ;)).

EMI
18.11.09, 14:09
In welcher Weise sollte die Existenz der Expansion davon abhängen, ob das Universum euklidisch, elliptisch oder hyperbolisch gestaltet ist?
Hallo Bauhof,

Ein expandierendes (kein statisches) Universum folgt aus der ART, in folgender Weise:

Materiedichte < kritischer Wert
-> offenes Universum -> global negative Krümmung -> unbegrenzt weiter expandierendes Universum.

Materiedichte = kritischer Wert
-> offenes Universum -> global ungekrümmt/euklidisch -> unbegrenzt weiter(immer langsamer werdend ohne "stehen zu bleiben") expandierendes Universum.

Materiedichte > kritischer Wert
-> geschlossenes Universum -> global positive Krümmung -> immer langsamer(bis zu Stillstand) werdende Expansion, dann Umkehrung in eine immer schneller werdende Kontraktion. (Zykloidenbogen)

Gruß EMI

PS: das hat auch EINSTEIN überrascht, er wollte es bis zum Nachweis(Hubble) auch nicht glauben.

Bauhof
18.11.09, 16:19
Hallo Bauhof, Ein expandierendes (kein statisches) Universum folgt aus der ART, in folgender Weise:

Materiedichte < kritischer Wert
-> offenes Universum -> global negative Krümmung -> unbegrenzt weiter expandierendes Universum.

Materiedichte = kritischer Wert
-> offenes Universum -> global ungekrümmt/euklidisch -> unbegrenzt weiter(immer langsamer werdend ohne "stehen zu bleiben") expandierendes Universum.

Materiedichte > kritischer Wert
-> geschlossenes Universum -> global positive Krümmung -> immer langsamer(bis zu Stillstand) werdende Expansion, dann Umkehrung in eine immer schneller werdende Kontraktion. (Zykloidenbogen)
Hallo EMI,

du hast recht, das alles folgt aus der ART, so habe ich es auch aus der Literatur in Erinnerung.

Aber das alles ist nur dann zutreffend, wenn die ART auf das Universum als Ganzes anwendbar ist. Das ist noch nicht experimentell ausreichend belegt.

Wir wissen bisher nur, dass die ART die gravitativen Effekte in unserer näheren Umgebung mit hinreichender Genauigkeit beschreibt. Bei den Abweichungen der Rotationskurven der Galaxien muss man bereits die "Dunkle Materie" [1] mit zur Erklärung heranziehen. Hier liefert die ART keine Erklärung. Deshalb ist es fraglich, ob die ART auf das Universum als Ganzes anwendbar ist, wenn sie bereits bei kleineren Objekten wie den Galaxien die Beobachtungs-Ergebnisse nicht vorhersagen kann.

M.f.G Eugen Bauhof

[1] Bei der (angeblich) beobachteten beschleunigten Expansion muss man die sog. "Dunkle Energie" zur Erklärung heranziehen. Da liefert die ART auch nichts.

Timm
18.11.09, 16:24
Hi Jogi und Eugen,


Wenn Parallelen nicht parallel bleiben, haben wir es nicht mit einer euklidischen Geometrie zu tun, oder?

Da stimme ich Dir zu, verstehe allerdings den Einwand nicht.

Zwischenfazit:
Wenn wir uns in die zwei Raumschiffe setzen, mit parallelem Kurs starten, den Joystick nicht anfassen und nach elfundachtzig Quintilliarden von Jahren immer noch den gleichen Abstand voneinander messen, dann können wir ziemlich sicher sein, in einem euklidischen 3Raum zu Leben.


Ich wollte hier lediglich kundtun, daß ein statisches Universum in den Friedmann-Gleichungen http://de.wikipedia.org/wiki/Friedmann-Gleichungen nicht enthalten ist. Ein solches Universum müßte kollabieren. Deshalb wurde ja eine abstoßend wirkende Vakuumenergie eingeführt.

Zitat von Bauhof
Ich meine, ein Universum expandiert (aus irgendeinem finsteren Grund) oder es expandiert nicht. In welcher Weise sollte die dei Existenz der Expansion davon abhängen, ob das Universum euklidisch, elliptisch oder hyperbolisch gestaltet ist?


"es expandiert nicht" kann nur bedeuten, es kollabiert. Das Schicksal des Universums hängt von seiner Krümmung ab. Hier ein Ausschnitt aus dem Wiki Link:

1. Modell: Die Energiedichte des Universums ist größer als die kritische Energiedichte (siehe unten). Dann ist die Krümmung der Raumzeit positiv (K = 1), das Universum „sphärisch“ (ein zweidimensionales Analogon wäre die Oberfläche einer Kugel). Ein solches sphärisches Universum ist übrigens auch geschlossen: Obwohl unbegrenzt wäre es nur endlich groß. Wer lange genug in eine Richtung läuft, kommt irgendwann zu seinem Ausgangspunkt zurück.
2. Modell: Die Energiedichte ist genau so groß wie die kritische Energiedichte. Die Raumzeit hat verschwindende Krümmung (K = 0), das Universum ist „flach“ (entsprechend einer Ebene in zwei Dimensionen).
3. Modell: Die Energiedichte ist kleiner als der kritische Wert. Die Krümmung der Raumzeit ist negativ (K = − 1), das Universum „hyperbolisch“ (eine solche Raumzeit lässt sich nicht in zwei Dimensionen visualisieren).



Je nach Zustandsgleichung der im Universum enthaltenen Materie ergeben sich auch drei verschiedene Möglichkeiten für die weitere Entwicklung des Universums:

1. Möglichkeit: Die Gravitation ist in der Lage, die Expansion soweit abzubremsen, dass sie zum Stillstand kommt und sich umkehrt. Das Universum zieht sich auf einen einzigen Punkt zusammen (Big Crunch). Über die weitere Entwicklung „nach“ diesem Ereignis kann nur spekuliert werden. Einige Szenarien sehen die Möglichkeit eines „pulsierenden“ Universums vor.
2. Möglichkeit: Die Gravitation verlangsamt die Expansion immer weiter, bringt sie jedoch nicht zum Stillstand.
3. Möglichkeit: Die Expansion beschleunigt sich und die gewöhnliche Materie im Universum wird immer weiter ausgedünnt.


So unterschiedlich Kosmologen die Urknall Szenarien diskutieren, so einig sind sie sich über die grundsätzliche Richtigkeit der Friedmann-Gleichungen. Auch neuere Erkenntnisse, wie dunkle Materie und dunkle Energie geben keinerlei Spielraum für ein statisches Universum.

Gruß, Timm

Bauhof
18.11.09, 16:47
"es expandiert nicht" kann nur bedeuten, es kollabiert. Das Schicksal des Universums hängt von seiner Krümmung ab.
Hallo Timm,

warum soll "es expandiert nicht" gleichbedeutend mit "es kollabiert" sein? Was sollte ein unendlich großes, euklidisches Universum zum Kollabieren bringen? Bisher war ich der Meinung, dass es von "Lambda" [1] abhängt, ob das Universum expandiert oder statisch ist. Ich dachte, wenn Lambda = Null, dann ist es statisch. Vielleicht habe ich da etwas nicht richtig in Erinnerung.

M.f.G. Eugen Bauhof

[1] Hatte Einstein sein "Lambda" nicht deswegen eingeführt, damit seine ART auch die Expansion voraussagt?

EMI
18.11.09, 17:43
Ich dachte, wenn Lambda = Null, dann ist es statisch. Vielleicht habe ich da etwas nicht richtig in Erinnerung.
Hatte Einstein sein "Lambda" nicht deswegen eingeführt, damit seine ART auch die Expansion voraussagt?
Hallo Bauhof,

so ist es nicht.
Wie ich oben erwähnte war EINSTEIN "erschrocken" als ihm bekannt wurde, dass die Gleichungen der ART KEIN statisches Universum erlauben.
Er war aber Anhänger eines statischen Universum und machte sich sofort daran seine Gleichungen zu korrigieren.
Er führte deshalb künstlich sein Lambda ein und "hielt" damit das Universum an.
Kurz darauf wurde er von Friedmann darauf aufmerksam gemacht, das auch ein "Lambda Universum" äußerst instabil ist und in eine dynamische Phase übergehen MUSS!
EINSTEIN gab erst auf an seinen Gleichungen "rumzubasteln" als Hubble die Expansion durch Beobachtung bestätigte.
EINSTEIN strich sofort das künstliche Lambda aus seinen Gleichungen und nannte die Einführung des Lambda seine größte Eselei.

Gruß EMI

PS: Das mit dem Spruch der Eselei ist nicht gesichert, kann auch eine Anekdote sein.

zeitgenosse
18.11.09, 18:23
Und da fragen wir uns, ob in einem expandierenden, ansonsten aber euklidischen 3Raum die Beiden eine Zunahme ihres Abstandes beobachten könnten.

Rotverschiebung wäre ein Indiz dafür.

Gr. zg

Timm
18.11.09, 18:48
Hallo Eugen,


warum soll "es expandiert nicht" gleichbedeutend mit "es kollabiert" sein? Was sollte ein unendlich großes, euklidisches Universum zum Kollabieren bringen?

Bisher war ich der Meinung, dass es von "Lambda" [1] abhängt, ob das Universum expandiert oder statisch ist. Ich dachte, wenn Lambda = Null, dann ist es statisch. Vielleicht habe ich da etwas nicht richtig in Erinnerung.

M.f.G. Eugen Bauhof

[1] Hatte Einstein sein "Lambda" nicht deswegen eingeführt, damit seine ART auch die Expansion voraussagt?

warum soll "es expandiert nicht" gleichbedeutend mit "es kollabiert" sein? Was sollte ein unendlich großes, euklidisches Universum zum Kollabieren bringen?


Dieser Meinung war Einstein ursprünglich auch. Dann stellte sich aber schnell heraus, daß seine Gleichungen ein statisches Universum nicht zuließen, es konnte nur expandieren oder kollabieren. Um einen Ausweg zu finden, führte er die kosmologische Konstante ein. Diese wurde aber schnell wieder hinfällig, als de Sitter allgemeingültigere Gleichungen fand. Der Durchbruch kam durch Friedmann und Lemaitre. Nur in diesem Kontext gibt es kein statisches Universum.

Gruß, Timm

P.S. Ich sehe gerade, daß EMI Dir schon geantwortet hat

zeitgenosse
18.11.09, 19:27
Du hast einen Link mit der Extension "svg" angegeben. Welches Format ist das?

Scalable Vector Graphics:
http://de.wikipedia.org/wiki/Scalable_Vector_Graphics

Ich habe noch nie in der Literatur die Formel für die Krümmung einer n-dimensionalen Kugeloberfläche gefunden.Im Moment weiss ich auch keinen Rat.

Gr. zg

Jogi
18.11.09, 20:17
Hi Timm & zg.



Wenn Parallelen nicht parallel bleiben, haben wir es nicht mit einer euklidischen Geometrie zu tun, oder?

Da stimme ich Dir zu, verstehe allerdings den Einwand nicht.
Don't worry, ich inzwischen auch nicht mehr.:o

Im homogen expandierenden 3Raum bleiben Parallelen natürlich auch dann parallel, wenn ihr Abstand wächst.
Rotverschiebung wäre ein Indiz dafür.
Sehr richtig, ein Indiz.


Gruß Jogi

SCR
19.11.09, 09:17
Don't worry, ich inzwischen auch nicht mehr.:o
Im homogen expandierenden 3Raum bleiben Parallelen natürlich auch dann parallel, wenn ihr Abstand wächst.
Betrachtet man keine parallelen Linien (mit denen der Raum leider nicht ausgestattet ist ;)) sondern Flugbahnen :rolleyes: ... das hatten wir schon einmal ziemlich am Anfang dieses Threads (und auch - zumindest ansatzweise - wie die Raumkrümmung gemessen wird: Stichwort WMAP).

Bauhof
19.11.09, 18:35
Er führte deshalb künstlich sein Lambda ein und "hielt" damit das Universum an.
Hallo EMI,

Bewahrte Einstein damit das euklidische unendliche Universum vor dem Zusammensturz oder vor der Expansion?

M.f.G. Eugen Bauhof

P.S.
Das mit der Eselei Einsteins stimmt. Ich kenne es so:
"Das war die größte Eselei meines Lebens!"

Durch diese Eselei hat er übersehen, dass das Univesum auch expandieren könnte.

zeitgenosse
20.11.09, 06:22
Bewahrte Einstein damit das euklidische unendliche Universum vor dem Zusammensturz oder vor der Expansion?

Einstein ging nicht von einem offenen euklidischen Raum aus. Das von ihm 1917 versuchsweise in die Physik eingeführte kosmologische Glied (auch Λ-Term genannt) sollte bewirken, dass die Feldgleichungen einen statischen Kugelraum zulassen. Als durch Friedmann ein expandierendes Weltall nahegelegt und durch Hubble der empirische Beweis dazu erbracht wurde, liess Einstein das kosmologische Glied fallen.

In jüngerer Zeit erhielt die kosmologische Konstante erneuten Auftrieb durch die Quintessenz-Modelle, wo sie als Energiedichte des Vakuums interpretiert wird.

Oft kommt im Kontext ein Name zu kurz, dessen Träger viel zur modernen Kosmologie beigetragen hat, nämlich Georges Lemaitre (1894-1966). Er war Priester und Mathematiker zugleich.

Die ursprüngliche Idee des expandierenden Universums stammt von Lemaitre (1927) und nicht von Hubble. Die Entdeckung der kosmologischen Rotverschiebung stammt ebenfalls nicht von Hubble, sondern von Slipher (1912). Hubble hat dafür andere Erkenntnisse beigesteuert, darunter das Hubble-Gesetz (1929): v = Hr

Die Idee eines Urknalls stammt von Lemaitre. Dieser sprach von einem Uratom, „ein kosmisches Ei, das im Moment der Entstehung des Universums explodierte“. Später hat Gamow den populärwissenschaftlichen Ausdruck "Big bang" geprägt. Heute spricht man vom Friedmann-Lemaitre-Kosmos.

Gr. zg

SCR
20.11.09, 07:56
Georges Lemaitre
möbius müsste ihn auf jeden Fall schon kennen. ;)

Bauhof
20.11.09, 09:22
Einstein ging nicht von einem offenen euklidischen Raum aus. Das von ihm 1917 versuchsweise in die Physik eingeführte kosmologische Glied (auch Λ-Term genannt) sollte bewirken, dass die Feldgleichungen einen statischen Kugelraum zulassen.

Hallo Zeitgenosse,

was ist ein statischer Kugelraum im Hinblick auf das Universum als Ganzes?

M.f.G. Eugen Bauhof

möbius
20.11.09, 12:14
möbius müsste ihn auf jeden Fall schon kennen. ;)

Möbius fragt sich natürlich, wer denn dieses sog. "kosmische Ei" befruchtet hat ...:confused: :eek:
Na, die "Henne" muss es ja wissen ...:D
Grüsse, möbius

Timm
20.11.09, 12:53
In jüngerer Zeit erhielt die kosmologische Konstante erneuten Auftrieb durch die Quintessenz-Modelle, wo sie als Energiedichte des Vakuums interpretiert wird.


Man muß da ein bißchen aufpassen. Der Wert der Kosmologischen Konstante ist, wie der Name schon sagt, zeitunabhängig, also auch unabhängig von der Größe des Universums. Konkurrierend dazu gibt es die Quintessenzmodelle, die von einer unbekannten Energieform ausgehen. Mit solche Modellen (Sammelbegriff "Dunkle Energie") versucht man die seit einigen Jahren festgestellte beschleunigte Expansion des Universums zu erklären. Die Quintessenz ist im Unterschied zur Kosmologischen Konstante zeitabhängig.

Gruß, Timm

möbius
20.11.09, 15:15
1. Man muß da ein bißchen aufpassen. Der Wert der Kosmologischen Konstante ist, wie der Name schon sagt, zeitunabhängig, also auch unabhängig von der Größe des Universums.
2. Die Quintessenz ist im Unterschied zur Kosmologischen Konstante zeitabhängig.

Gruß, Timm

Hallo Timm!
Zu 1.:
Wenn die "Kosmologische Konstante" eine empirische Größe ist, (Wahrscheinlich ist sie das nicht...:eek: ) musste sie aber doch irgendwann einmal gemessen worden sein, oder:confused: Was heisst da "zeitunabhängig":confused:
Zu 2.:
Wie kann das verstanden werden:
"Die Qintessenz ist ...zeitabhängig." Was heisst da "zeitabhängig":confused:
Gruß, möbius

zeitgenosse
20.11.09, 16:58
was ist ein statischer Kugelraum im Hinblick auf das Universum als Ganzes?

So wie ich das z.Z. sehe, handelt es sich beim statischen Kugelraum-Universum Einsteins um ein sphärisches Weltall mit homogenen und isotropen Eigenschaften. Ein solches Weltall ist endlich. Es könnte sich topologisch um eine 3-Sphäre handeln.

Gr. zg

Timm
20.11.09, 18:00
Hallo Timm!
Zu 1.:
Wenn die "Kosmologische Konstante" eine empirische Größe ist, (Wahrscheinlich ist sie das nicht...:eek: ) musste sie aber doch irgendwann einmal gemessen worden sein, oder:confused: Was heisst da "zeitunabhängig":confused:
Zu 2.:
Wie kann das verstanden werden:
"Die Qintessenz ist ...zeitabhängig." Was heisst da "zeitabhängig":confused:
Gruß, möbius

Hallo Möbius,

die Kosmologische Konstante ist Bestandteil mathematischer Modelle, die das Schicksal des Universums beschreiben. Ihre Wirkung ist über die Zeit konstant. Das paßt aber nicht zu den neueren Beobachtungen. Deswegen postulieren Kosmologen stattdessen eine dunkle Energie, "dunkel", um ihe Unwissen über deren Natur anzudeuten. Sind Philosophen auch so ehrlich?

Gruß, Timm

EMI
20.11.09, 18:12
was ist ein statischer Kugelraum im Hinblick auf das Universum als Ganzes?
Hallo Bauhof,

ein Jahr nach seiner Veröffentlichung "Grundlage der allgemeinen Relativitätstheorie"(1916) veröffentlichte Einstein seine Arbeit "Kosmologische Betrachtungen zur allgemeinen Relativitätstheorie"(1917).
Diese ist die ERSTE wissentschaftliche Begründung der Kosmologie!

Das in dieser Arbeit entwickelte mathematische Modell des Kosmos ergab einen zeitlich unveränderten(statischen) unbegrenzten, bezüglich des Volumens positiv gekrümmten endlichen Raum.

Zwischen dem Krümmungradius R, der mittleren Massedichte ρ und der Einsteinschen grav.Konstante χ besteht in dieser (Einsteinschen Zylinderwelt) der Zusammenhang:

R = √(2/ρχc²)

Gruß EMI

möbius
20.11.09, 19:57
Hallo Timm!
Das kommt auf den/die jeweiligen Philosophen an..;)
möbius jedenfalls, falls er überhaupt ein sog. "Philosoph" ist:confused: , weiss nichts, aber das möglicherweise auf relativ hohem Niveau...(was immer das heissen mag...):D
Gruß, möbius

zeitgenosse
20.11.09, 22:17
Man muß da ein bißchen aufpassen. Der Wert der Kosmologischen Konstante ist, wie der Name schon sagt, zeitunabhängig, also auch unabhängig von der Größe des Universums.

Für eine konstante Vakuumenergiedichte müsste die kosmologische Konstante sehr gross sein. Beobachtungen verweisen dagegen auf eine verhältnismässig kleine Energiedichte. Die Unterschiede liegen in der Grössenordnung von etlichen Zehnerpotenzen. Aus diesem Grund geht man heute von einer zeitlich veränderlichen Energiedichte eines kosmologischen Skalarfeldes aus - eben der Quintessenz.

Gr. zg

Timm
20.11.09, 23:40
Für eine konstante Vakuumenergiedichte müsste die kosmologische Konstante sehr gross sein. ... Aus diesem Grund geht man heute von einer zeitlich veränderlichen Energiedichte eines kosmologischen Skalarfeldes aus - eben der Quintessenz.


Das ist ja richtig,

In jüngerer Zeit erhielt die kosmologische Konstante erneuten Auftrieb durch die Quintessenz-Modelle, wo sie als Energiedichte des Vakuums interpretiert wird.


und gleichzeitig der Grund, weshalb die Kosmologische Konstante keinen erneuten Auftrieb erhält.

Gruß, Timm

Marco Polo
21.11.09, 01:20
was ist ein statischer Kugelraum im Hinblick auf das Universum als Ganzes?

Hallo Eugen,

ich vermute, (hat nichts zu sagen) das ist ein sphärischer Raum, der seit allen Ewigkeiten unverändert (also statisch) existiert. So wie es Einstein zunächst vermutet hatte.

Zeitgenosse hat es ja bereits ähnlich formuliert.

Gruss, Marco Polo